Sei sulla pagina 1di 257

eroberogeday

lwm pln nig Esn Bisid

sRmab;fak;TI 11
i
KNkmkarniBn nig eroberog
lwm pln nig Esn Bisid
KNkmkarRtYtBinitbeckeTs
elak lwm qun elak Gwug sMNag
elak nn; suxNa elakRsI Tuy rINa
elak RBwm sunit elak pl bunqay
elak Tit em:g
KNkmkarRtYtBinitGkraviruT
elak lwm miKsir
karIkMuBTr
kBaa lI KuNaka elak Gwug sMNag

ii
Garmfa
esoePA sIVtcMnYnBit nig esrIfak;TI11 EdlGksikSakMBugkan;
enAkgdenH rYmmanemeronsegb lMhat;KMrU nig lMhat;Gnuvtn_ .
eKalbMNgnkareroberogcgRkgKWedIm,ITukCaksarCMnYysRmab;Gk
sikSakgRKb;mCdan nig ma:geTotedIm,IcUlrYmelIksyvisyKNitviTakg
RbeTskm<CaeyIg[kan;EtrIkceRmInqab;rhsRsbtamsmyviTasaRsTMenIb.
esovePAenHminlhYseKhYsgenaHeT . kMhusqgedayGectnaR)akd
CamanTaMgbeckeTs nig GkraviruT . ehtuenHeyIgxJMCaGkniBn rgcaMCanicnUv
mtiriHKn;EbbsabnaBIGksikSaRKb;mCdanedaykIrIkrayedIm,IEklMGesovenH
[kan;EtmansuRkitPaBRbesIreLIgEfmeTot .
CaTIbBab; xJM)aTGkniBnsUmeKarBCUnBrGksikSaTaMgGs;mansuxPaBl
manR)aCJaQasv nig TTYl)aneCaKCyCanickgkarsikSa .

)at;dMbg fTI 30 Exkkda qaM2011


GkniBn nig RsavRCav lwm pln
Tel : 017 768 246
Email: lim_phalkun@ymail.com
Website: www.mathtoday.wordpress.com

iii
sIVtncMnYnBit nig esrI

emeronTI1
sIVtncMnYnBit
1>sBaaNnsIVtcMnYnBit
sIVtncMnYnBit KWCaGnuKmn_elxEdlkMNt;BIsMNMu IN eTA IR .
]TahrN_
eKmanGnuKmn_ f : IN IR Edl f (n) n 2 n 1
cMeBaH n 1 enaH f (1) 12 1 1 3
cMeBaH n 2 enaH f ( 2) 2 2 2 1 7
cMeBaH n 3 enaH f ( 3) 3 2 3 1 13
cMeBaH n 4 enaH f (4) 4 2 4 1 21
cMeBaH n 5 enaH f (5) 5 2 5 1 31
>>>>>>>>>>>>>>>>>>>>>>>>>>>>>>>>>>>>>>>>>>>>>>>>>>>>>>>>>>>>>>>>>>>>>>>>>>>>>>>>>>>>>>>>>>>>>>>>>>>>>>>
>>>>>>>>>>>>>>>>>>>>>>>>>>>>>>>>>>>>>>>>>>>>>>>>>>>>>>>>>>>>>>>>>>>>>>>>>>>>>>>>>>>>>>>>>>>>>>>>>>>>>>>
eK)ancMnYnerobtamlMdab; 3 , 7 , 13 , 21 , 31 , ..... begIt)anCasIVt
ncMnYnBitmYy .cMnYnnImYynsIVtehAfatY .
cMnYn 3 ehAfatYTImYy cMnYn 7 ehAfatYTIBIr cMnYn 13 ehAfatYTIbI

eroberogeday lwm pln TMBr 1


sIVtncMnYnBit nig esrI

cMnYn 21 ehAfatYTIbYn >>>>>> erogKa .


eKkMNt;eRbIGkSr a1 , a 2 , a 3 , ....., a n sRmab;tag[tYnsIVt .
a1 tag[tYTImYy a 2 tag[tYTIBIr >>>>> nig a n tag[tYTI n .
eKtagsIVtedaynimitsBa (a n ) Edl n IN .
ebI n 0 , 1 , 2 , .... enaHsIVt (a n ) epImedaytY a 0 , a1 , a 2 ,... .
]TahrN_ eK[sIVt a n 2n 1 Edl n 1 , 2 , 3 , .... .
cUrsresrR)aMtYdMbUgnsIVt (a n ) ?
cMeBaH n 1 enaH a1 21 1 3
cMeBaH n 2 enaH a 2 2 2 1 5
cMeBaH n 3 enaH a 3 2 3 1 9
cMeBaH n 4 enaH a 4 2 4 1 17
cMeBaH n 5 enaH a 5 2 5 1 33
dUcenH 3 , 5 , 9 , 17 , 33 CaR)aMtYdMbUgnsIVt (a n ) .
2>tYTI n nsIVtcMnYnBit
eK[sIVtncMnYnBit (a n ) mantY f (1) , f ( 2) , f (3) , .... .
tYTI n nsIVtenHKW a n f (n) .

eroberogeday lwm pln TMBr 2


sIVtncMnYnBit nig esrI

]TahrN_1 eK[sIVt (a n ) : 1 , 3 , 5 , 7 ,9,11,....


eKman a1 1 2 1 1
a2 3 2 2 1
a3 5 2 3 1
a4 7 2 4 1


tYTI n nsIVtenHKW a n 2n 1 Edl n 1 , 2 , 3 ,.... .
]TahrN_2 eK[sIVt (a n ) : 2 , 5 , 10 , 17 , 26 , .... .
rktYTI n nsIVtenH ?
eKman a1 2 12 1
a 2 5 22 1
a 3 10 32 1
a4 17 42 1
a5 26 52 1


dUcenH a n n 2 1 CatYTI n nsIVt (a n ) .

eroberogeday lwm pln TMBr 3


sIVtncMnYnBit nig esrI

]TahrN_3 eK[sIVt (a n ) : 3 , 5 , 9 , 17 , 33 , .... .


rktYTI n nsIVtenH ?
eKman a1 3 21 1
a2 5 22 1
a3 9 23 1
a 4 17 2 4 1
a 5 33 2 5 1

dUcenH a n 2n 1 CatYTI n nsIVt (a n ) .
3>GefrPaBnsIVtcMnYnBit
k-sIVtekIn nig sIVtcuH
niymny
-sIVt (a n ) CasIVtekInluHRtaEtRKb;cMnYnKt; n IN :a n 1 a n .
-sIVt (a n ) CasIVtcuHluHRtaEtRKb;cMnYnKt; n IN :a n 1 a n .
]TahrN_1 Rsayfa a n 2n 1 CasIVtekIn ?
n1
2n 1
eKman a n
n1
naM[ a n 1 2(n 1) 1 2n 3
(n 1) 1 n2

eroberogeday lwm pln TMBr 4


sIVtncMnYnBit nig esrI
2n 3 2n 1
eK)an a n 1 a n
n2 n1
( 2n 3)(n 1) ( 2n 1)(n 2)

(n 1)(n 2)
2n 2 5n 3 2n 2 5n 2

(n 1)(n 2)
1
0
(n 1)(n 2)
eKTaj a n 1 a n naM[ (a n ) CasIVtekIn .
]TahrN_2 Rsayfa a n n CasIVtcuH ?
2n 1
n n1 n1
eKman a n naM[ a n 1
2n 1 2(n 1) 1 2n 1
eK)an a n 1 a n n 1 n
2n 1 2n 1
(n 1)( 2n 1) n( 2n 1)

( 2n 1)( 2n 1)
2n 2 n 1 2n 2 n

4n 2 1
1
2 0
4n 1
eKTaj a n 1 a n naM[ (a n ) CasIVtcuH .

eroberogeday lwm pln TMBr 5


sIVtncMnYnBit nig esrI

x-sIVtm:UNUtUn
niymny
sIVt (a n ) CasIVtm:UNUtUnluHRtaEt (a n ) CasIVtekInCanic b (a n )
CasIVtcuHCanic .
)annyfa a1 a 2 a 3 ..... a n a n 1 ....
b a1 a 2 a 3 ..... a n a n 1 ..... .
]TahrN_ bgajfa a n 3n 1 CasIVtm:UNUtUn ?
eKman a n 1 a n (3n 1 1) (3n 1) 2 3n 0
enaH a n 1 a n naM[ (a n ) CasIVtm:UNUtUn .
4>sIVtTal;
k-sIVtTal;elI
niymny
sIVt (a n ) CasIVtTal;elIluHRtaEtmancMnYnBit M mYyEdlcMeBaH
n IN epgpat; a n M . cMnYn M ehAfaeKalelInsIVt (a n )
x-sIVtTal;eRkam
niymny
sIVt (a n ) CasIVtTal;eRkamluHRtaEtmancMnYnBit m mYyEdlcMeBaH
n IN epgpat; a n m .cMnYn m ehAfaeKaleRkamnsIVt (a n ) .

eroberogeday lwm pln TMBr 6


sIVtncMnYnBit nig esrI

K-sIVtTal;
niymny
sIVt (a n ) CasIVtTal;luHRtaEtmancMnYnBit m nig M EdlcMeBaH
n IN epgpat; m a n M .
cMnYn m ehAfaeKaleRkam nig M ehAfaeKalelInsIVt (a n ) .
)annyfa (a n ) CasIVtTal;elIpg nig Tal;eRkampg .
]TahrN_1 eK[sIVt a n 2 9 Edl n 1 , 2 , 3 ,...
n n1
cUrRsayfa (a n ) CasIVtTal;elI ?
RKb; n 1 , 2 , 3 ,... eKman n 2 n 1 3
eK)an 2 1 1 naM[ a n 2 9 9 3 .
n n1 3 n n1 3
dUcenH (a n ) CasIVtTal;elI Edl 3 CaeKalelInsIVt .
n n
]TahrN_2 eK[sIVt a n 5 1 5 1
2 2
Edl n 0 , 1 , 2 , 3 ,... .
cUrRsayfa (a n ) CasIVtTal;eRkam ?
tamvismPaBmFmnBVn nig mFmFrNImaRt a b ab
2

eroberogeday lwm pln TMBr 7


sIVtncMnYnBit nig esrI
n n
eK)an a n 2 5 1 5 1 2
2 2
dUcenH (a n ) CasIVtTal;eRkam Edl 2 CaeKaleRkamnsIVt .
2n 2 5n 4
]TahrN_3 eK[sIVt an 2 Edl n 1 , 2 , 3 ,...
n 3n 3
KNna a n 1 nig a n 7 rYcTajfa (a n ) CasIVtTal;EdleKnig
3
bBaak;eKalelI nig eKaleRkamrbs;va .
KNna a n 1 nig a n 7
3
2n 2 5n 4 n 2 2n 1
eK)an a n 1 2 1 2
n 3n 3 n 3n 3
(n 1) 2
dUcenH a n 1 2 .
n 3n 3
7 2n 2 5n 4 7 n 2 6n 9
ehIy a n 2 2
3 n 3n 3 3 n 3n 3
7 (n 3) 2
dUcenH a n 2 .
3 n 3n 3
Tajfa (a n ) CasIVtTal;
eKman n 2 3n 3 (n 3 ) 2 3 0
2 4

eroberogeday lwm pln TMBr 8


sIVtncMnYnBit nig esrI

(n 1) 2
tamsRmayxagelIeK)an a n 1 2 0
n 3n 3
7 (n 3 ) 2
naM[ a n 1 ehIy a n 2 0
3 n 3n 3
naM[ a n 7 .
3
dUcenH 1 a n 7 naM[ (a n ) CasIVtTal;Edl 1 CaeKaleRkam
3
nig 7 CaeKalelInsIVt .
3

eroberogeday lwm pln TMBr 9


sIVtncMnYnBit nig esrI

emeronTI2
sIVtnBVn
1-niymny
sIVtnBVn KWCasIVtncMnYnBitEdlmantYnImYyeRkABItYTImYy
esInwgtYmunbnab;bUkcMnYnefr d mYyehAfaplsgrYmnsIVt .
ebI (u n ) CasIVtnBVnmanplsgrYm d enaH d kMNt;eday
d u 2 u 1 u 3 u 2 .... u n 1 u n .
]TahrN_ (u n ) : 4 , 7 , 10 , 13 , 16 , 19 , .... CasIVtnBVnEdlman
plsgrYm d 7 4 3 .
2-tYTI n nsIVtnBVn
-ebI (u n ) CasIVtnBVnEdlmantYTImYy u1 nig plsgrYm d enaH
tYTI n rbs;vakMNt;eday u n u1 (n 1)d .
-ebI (u n ) CasIVtnBVnEdlmantYTImYy u 0 nig plsgrYm d enaH
tYTI n rbs;vakMNt;eday u n u 0 n.d .

eroberogeday lwm pln TMBr 10


sIVtncMnYnBit nig esrI

]TahrN_1 eK[ (u n ) CasIVtnBVnmantY u1 5 nigplsgrYm


d 4 . KNnatY u n CaGnuKmn_n n ?
tamrUbmn u n u1 (n 1)d eday u1 5 , d 4
eK)an u n 5 (n 1)(4) 4n 1 .
]TahrN_2 eK[ (u n ) CasIVtnBVnmantY u 0 7 nigplsgrYm
d 9 . KNnatY u n CaGnuKmn_n n ?
tamrUbmn u n u 0 nd eday u 0 7 , d 9
eK)an u n 7 9n .
]TahrN_3 eKmansIVt (u n ) kMNt;eday
u 1 2 nig u n 1 3 u n 5 Edl n 1 , 2 , 3 , ....
3

k-tag Vn u n 3 . bgajfa (Vn ) CasIVtnBVn .


x-KNna Vn nig u n CaGnuKmn_n n .
dMeNaHRsay
k-bgajfa (Vn ) CasIVtnBVn
eKman Vn u n 3 naM[ Vn 1 u n 1 3
eday u n1 3 u n 3 5 enaH Vn1 u n 3 5
eK)an Vn1 Vn (u n 3 5) u n 3 5 efr .
dUcenH (Vn ) CasIVtnBVnmanplsgrYm d 5 .

eroberogeday lwm pln TMBr 11


sIVtncMnYnBit nig esrI

x-KNna Vn nig u n CaGnuKmn_n n


tamrUbmn Vn V1 (n 1)d eday V1 u1 3 8
dUcenH Vn 8 5(n 1) 5n 3 .
ehIy Vn u n 3 enaH u n 3 Vn 3 5n 3 .
K-plbUknsIVtnBVn
plbUk n tYdMbUgnsIVtnBVnEdlman u1 CatYTImYy nig u n CatYTI n
kMNt;eday S n u1 u 2 u 3 ... u n n(u1 u n ) .
2
sRmaybBaak;
eKman S n u1 u 2 ........ u n
nig S n u n u n 1 .... u1
eK)an 2S n (u1 u n ) (u 2 u n 1 ) ..... (u n u1 )
eday u1 u n u 2 u n 1 .... u k u n k
eK)an 2S n (u1u n) ( u 1 u n ) .... (u 1 u n )

( n tY )
n( u 1 u n )
2S n n(u 1 u n ) naM[ S n .
2
dUcenH S n u1 u 2 u 3 ... u n n(u1 u n ) .
2

eroberogeday lwm pln TMBr 12


sIVtncMnYnBit nig esrI

]TahrN_1 KNnaplbUk S n 1 2 3 .... n


eKman 1 , 2 , 3 , ....., n CasIVtnBVnmantY u1 1 nig u n n
eK)an S n n(u1 u n ) n(1 n) n(n 1) .
2 2 2
]TahrN_2 KNnaplbUk S n 1 3 5 .... ( 2n 1)
eKman 1, 3 , 5 , ...,2n 1 CasIVtnBVnman u1 1 nig u n 2n 1
eK)an S n n(u1 u n ) n(1 2n 1) n 2 .
2 2
lMhat;Gnuvtn_
!> eK[ a , b , c , d CacMnYnKt;viCmanbegIt)anCasIVtnBVnmYy .
cUrRsaybBaak;fa abcd (b a)4 CakaerR)akd .
@> eK[ a , b , c CabIcMnYnBitviCman .
ebI 1 , 1 , 1 CabItYbnKansIVtnBVnmYyenaHcUrRsay
ab ac bc
bBaak;fa a 2 , b 2 , c 2 kCabItYbnKansIVtnBVnEdr .
#> cUrkMNt;cMnYnBit x edIm,I[cMnYnxageRkamCabItYbnKansIVtnBVn
k> x 2 ; 2(x 1) 2 ; x 8
x> (2 x) 2 ; 2x ; x 2
K> (x 1) 2 ; (x 3) 2 ; (x 9) 2

eroberogeday lwm pln TMBr 13


sIVtncMnYnBit nig esrI
5
$> eK[sIVtnBVnmYyEdlmantYTImYy u1 nigplsgrYm d
3 6
k> KNnatYTI 168 nsIVtenH .
x> etItYTIb:unannsIVtenHEdlmantmesInwg 8 ?
3
%> etImanRtIekaNEkgNaEdlmanrgVas;nRCugTaMgbIrbs;vabegIt)an
CabItYbnKansIVtnBVn ?
^> eK[smIkar x 2 x 0 .
kMNt;tm nig edIm,I[plbUkkaernbsrbs;smIkarenHesInwg
14 ehIy 1 , , CabItYbnKansIVtnBVnmYyEdlcuHdac;xat .
&> etImanRtIekaNEdlRCugrbs;va nig brimaRtbegIt)anCasIVtnBVnbeT?
*> eK[RtIekaNmYyEdlrgVas;mMukgbegIt)anCasIVtnBVnmYy .
cUrKNnatYTIBIrnsIVtenH ?
g( 0) 8

(> eK[ g : n IR Edl 1
g ( n ) g ( n 1) g(n 1)
2
Edl n IN .
k> KNnatm g(1), g( 2), g(3), g(4) rYckMNt;kenSamTUeTA g(n)
CaGnuKmn_n n .
x> Rsayfa g(n) CaGnuKmn_ekIn .

eroberogeday lwm pln TMBr 14


sIVtncMnYnBit nig esrI
u 1 1
!0> eKmansIVt (u n ) kMNt;eday
u un ; n IN

n 1 1 168 u n2

1
k> tag Vn 2
. bgajfa (Vn ) CasIVtnBVnmYy .
un
n
x> KNna Vn , u n nig S n ( Vn ) CaGnuKmn_n n .
k 1
!!> eK[GnuKmn_BhuFa f ( x) px 2 qx .
k> kMNt;cMnYnBit p nig q edIm,I[ f ( x) f ( x 1) x RKb; x .
n
x> TajrkplbUk (k ) 1 2 3 .... n .
k 1
!@>eKmanBhuFa f ( x) px 3 qx 2 rx .
k>kMNt; p, q, r edIm,I[ f ( x) f ( x 1) x 2 RKb; x IR
x>TajbBaak;fa
n n(n 1)( 2n 1)
k 1 2 ... n
2 2 2 2
k 1 6
!#> eKmanBhuFa f ( x) px 4 qx 3 rx 2 sx .
k>kMNt; p, q, r, s edIm,I[ f ( x) f ( x 1) x 3 RKb; x IR
x>TajbBaak;fa

eroberogeday lwm pln TMBr 15


sIVtncMnYnBit nig esrI
n n 2 (n 1) 2
k 1 2 ... n
3 3 3 3
k 1 4
!$> kMNt;plsgrYm d nigtYTImYy u1 nsIVtnBVnekIn (u n ),nIN
u 1 2 u 2 2 u 3 2 98
ebIeKdwgfa
u 1 u 2 u 3 4
KNna u10 nig plbUk !0 tYdMbUgnsIVtenH .
!%> plbUk n tYdMbUgnsIVtmYyesI S n n 2 n . cUrKNnatYTI%
rYcbgajfasIVtenHCasIVtnBVn .
a1 2
!^> eK[sIVt a 5an 1
n 1 a 3
n

k-tag bn a 1 1 . bgajfa (bn ) CasIVtnBVn .


n
x-KNna bn nig an CaGnuKmn_n n .
!&> eKmansIVtnBVnBIr
(an ) : 1 , 6 , 11 , 16 , 21 , ..... nig (bn ) : 7 , 11 , 15 , 19 , 23 , ....
k> sresr an nig bn CaGnuKmn_n n .
x> etIkgcMeNam 2012 tYdMbUgnsIVt (an ) & (bn ) manb:unantYEdlman
tmesIKa ?

eroberogeday lwm pln TMBr 16


sIVtncMnYnBit nig esrI

emeronTI3
sIVtFrNImaRt
1-niymny
sIVtFrNImaRt KWCasIVtncMnYnBitEdltYnImYy eRkABItYTImYy
esInwgtYmunbnab;KuNnwgcMnYnefr q Edl q 0 . cMnYnefr q enH
ehAfaplrYmrYmnsIVtFrNImaRt behAfaersugnsIVt .
ebI (u n ) CasIVtFrNImaRtmanpleFobrYm q enaHeK)an
u u u u
q 2 3 4 ..... n 1 .
u1 u2 u3 un
CaTUeTARKb; n IN : u n 1 q. u n enaH (u n ) CasIVtFrNImaRt
manpleFobrYm q .
x-tYTI n nsIVtFrNImaRt
-ebI (u n ) sIVtFrNImaRt mantYTImYy u1 nigmanpleFobrYm q
enaH tYTI n rbs;vakMNt;eday u n u1 q n 1 .
-ebI (u n ) CasIVtFrNImaRt mantYTImYy u 0 nigmanpleFobrYm q
enaHtYTI n rbs;vakMNt;eday u n u 0 q n .

eroberogeday lwm pln TMBr 17


sIVtncMnYnBit nig esrI

K-plbUk n tYdMbUgnsIVtFrNImaRt
ebI (u n ) sIVtFrNImaRtmantYTImYy u1 nigpleFobrYm q 1 enaH
1 qn
eK)an S n u1 u 2 .... u n u1 .
1 q
]TahrN_1 KNnaplbUk S n 10 10 2 10 3 .... 10n
eKman 10 , 10 2 , 10 3 , ....,10n CasIVtFrNImaRtmanpleFobrYm
q 10 nigtYTImYy u 1 10 .
1 10 n 10 n
eK)an S n 10 (10 1) .
1 10 9
]TahrN_2 eKmansIVtncMnYnBit (u n ) kMNt;eday
u 1 2 nig u n 1 3 2 3(u n ) 3 Edl n 1 , 2 , 3, ...
k> Rsayfa Vn 1 u n 3 CasIVtFrNImaRt .
x> KNna Vn nig u n CaGnuKmn_n n .
K> KNnaplbUk S n V1 V2 .... Vn CaGnuKmn_n n .
dMeNaHRsay
k> Rsayfa Vn 1 u n 3 CasIVtFrNImaRt
eK)an Vn 1 1 u n3 1 eday u n 1 3 2 3u n3
Vn 1 1 2 3u n3 3(1 u n3 ) 3Vn
dUcenH (Vn ) CasIVtFrNImaRtmanersug q 3 .
eroberogeday lwm pln TMBr 18
sIVtncMnYnBit nig esrI

x> KNna Vn nig u n CaGnuKmn_n n


tamrUbmn Vn V1 .q n 1 Et V1 1 u1 3 1 2 3 9
dUcenH Vn 9 3n 1 3n 1 .
tam Vn 1 u n 3 eKTaj)an u n 3 Vn 1
dUcenH u n 3 3n 1 1 .
K> KNnaplbUk S n V1 V2 .... Vn CaGnuKmn_n n
qn 1 3n 1 9 n
eK)an S n V1 9 ( 3 1)
q1 31 2
dUcenH S n 9 (3n 1) .
2
]TahrN_3
KNnaplbUk S n 3 33 333 ..... 333
....
333

(n )

eK)an 3S n 9 99 999 .... 999


...999

(n )
3S n (10 1) (10 2 1) .... (10 n 1)
3S n (10 10 2 .... 10 n ) (1 1 .... 1)
10 n 1 10 n 1 9n 10
3S n 10 n
10 1 9
10 n 1 ( 9n 10)
dUcenH S n .
27

eroberogeday lwm pln TMBr 19


sIVtncMnYnBit nig esrI

lMhat;Gnuvtn_
!> eK[sIVtFrNImaRtmYyman & tY EdplbUkbItYdMbUgesI 7 nig plbUk
bItYcugeRkayesInwg 112 . cUrkMNt;sIVtFrNImaRtenH ?
@> cMeBaHRKb;cMnYnBit a nig b Edl | a || b | 0 cUrRsayfa
a3 b3
) CatYtKansIVtFrNImaRt .
1
; ; ( a 2
ab b 2 2
(a b ) 2
a b
2 2

#> eK[kaermYymanRCugesI a .
eKsg;kaermYyeTotEdlmankMBUlTaMgbYnCacMnuckNalnRCugkaermun
rYcehIyeKsg;kaermYyeTotcarwkkgkaerTIBIr EdlmankBUlCacMnuc
kNalnkaerTIBIr . eKsg;kaertamrebobenHrhUtdl;kaerTI n .
k> cUrKNnaplbUknpRkLarbs;kaerTaMgGs; .
x> KNnaplbUknbrimaRtrbs;kaerTaMgGs; .
$>eKmanbIcMnYnBit a , b , c xusBIsUn .
eKdwgfa a , b , c CatYtKansIVtFrNImaRt ehIy a , 2b , 3c
CatYtKansIVtnBVn . cUrrkersugnsIVtFrNImaRt ?
%> bgajfaebI 2 , 1 , 2 CatYtKansIVtnBVnenaH x , y , z
yx y yz
CatYtKansIVtFrNImaRt .

eroberogeday lwm pln TMBr 20


sIVtncMnYnBit nig esrI

^> kMNt;BIrcMnYnBit x , y ebI 1; x ; y CasIVtnBVn nig 1; x 2 ; y 2


CasIVtFrNImaRt .
&> KNnaplbUk
13 303 5003 ( 2n 1)00...003
Sn ....
10 100 1000 1000....000
*> eK[ S n (a) a aa aaa ..... aaa
.....
aaa

(n )
9
Edl a IN nig 1 a 9 . cUrKNnaplbUk S S n (a)
a 1
(> eKmankenSam Tn 3 33 333 .... 333
....
333

(n )

k> Rsayfa 3Tn T'n n Edl Tn ' CaplbUknsIVtFrNImaRt


mYyEdleKnwgkMNt;tYTImYy nig pleFobrYmrbs;va .
x> KNna Tn CaGnuKmn_n n .
!0> eKmansIVt u n 2 2 2 2..... 2 2 man n bskaer .
k> cUrKNna u n edaysresrlTplCarag 2 V Edl Vn CasIVt
n

ncMnYnBitmYyRtUvrk .
x> sikSaPaBekIn b cuHnsIVt (u n ) .
K> cUrkMNt;lImItnsIVt u n kalNa n .

eroberogeday lwm pln TMBr 21


sIVtncMnYnBit nig esrI

!!> eK[ a 1 CacMnYnBitviCman . eKBinitsIVt u 0 ; u1 ; u 2 ;...; u n


EdlkMNt;edayTMnak;TMng u 0 1 , u n a.u n2 1 .
k> eKtag u n a V b . bgajfaeKGackMNt;cMnYnBit b )an
n

edIm,I[ (Vn ) CasIVtFrNImaRt .


x> KNna u n CaGnuKmn_n n nig a .
u 1 a ; u 2 b

!@> eK[sIVtncMnYnBit (u n ) kMNt;eday u n 1 u n 2
n
u
2
k> cUrRsayfa Vn u n u n 1 CasIVtFrNImaRt .
x> KNna u n CaGnuKmn_n a , b , n .

a 0 2 , b 0 4

!#> eK[sIVtcMnYnBit a n1 1 (a n 3b n )
4
1
b n 1 4 ( 3a n b n ) , n IN
k> kMnt;cMnYnBit edIm,I[ Cn a n .b n CasIVtefr .
x> bgajfa U n a n b n CasIVtFrNImaRt .
K> Tajrk a n nig b n CaGnuKmn_n n rYcbBaak;lImItva.

eroberogeday lwm pln TMBr 22


sIVtncMnYnBit nig esrI

!$> eK[smIkardWeRkTIbIGBaatBit
(E) : x 3 mx 2 (m 2 9m 14)x m 3 30m 2 300m 1000 0
Edl m Ca)a:ra:Em:Rt .
k>kMnt;tmrbs; m edIm,I[smIkarenHmanbsbIbeg;ItCasIVFrNImaRtmYy .
x>cUredaHRsaysmIkar (E) cMeBaHtm m rkeXIjxagelI .
!%> eK[sVItnBVn U 1 , U 2 , U 3 , ........, U n EdlmanplsgrYm d
ehIy d 0 .
eKBinitsIVt (Vn ) mYykMnt;cMeBaHRKb; n IN * eday
Vn a U , a 0 , a 1 .
n

k> cUrbgajfa (Vn ) CasIVtFrNImaRt rYcKNna Vn CaGnuKmn_n


a , U 1 , d nig n .
x> cUrRsayfa S V V V .... V a . 1 a .
n 1 2 3 n
U1
nd

1a
d

K> cUrKNnaplKuN Pn V1 V2 V3 ..... Vn .


!^> eK[ f CaGnuKmn_ kMnt;BIsMNMu IN * eTA IR eday
f (1) 2

f (1) f ( 2) f ( 3) .... f (n ) n .f (n )
2

k> KNnatmn f ( 2) nig f ( 3) .


x> cUrKNna f (n) CaGnuKmn_n n .

eroberogeday lwm pln TMBr 23


sIVtncMnYnBit nig esrI
u 1 5 3 5

!&> eK[sIVtncMnYnBit un kMnt;elI IN * eday 1 5
n 1 2 n u
u u
n

k> eKtag n IN* : v n uun 5


5
.
n
cUrrkTMnak;TMngrvag v n1 nig v n .
x> KNna v n CaGnuKmn_n n rYcTajrk u n CaGnuKmn_n n .
K> eKtag w n 1 v n cMeBaH n IN * .
KNnaplKuN Pn w 1 .w 2 .w 3 .....w n .
u1 5
!*> eK[sIVtncMnYnBit un kMnt;RKb; n IN * eday
u n 1 u n 2 2
k> cUrbgajfa n IN : un 2 .
x> eKtag v n un un 2 4 RKb; n IN * .
cUrrkTMnak;TMngrvag v n1 nig v n .
K> KNna v n CaGnuKmn_n n rYcTajrk u n CaGnuKmn_n n .
!(> eK[ a, b, c, d CabYntYtKansIVtnBVnmYy ehIy m CacMnYnBitepgpat;
ad bc
m
2
.
bgajfa x ax b x cx d m 2 0 cMeBaHRKb; x IR

eroberogeday lwm pln TMBr 24


sIVtncMnYnBit nig esrI
1 2 3 n1 n
@0> eK[kenSam S n 2 3 ... n 1 n .
7 7 7 7 7
cUrKNnapldk S n 1 .S n rYcTajrk S n .
7
u 1 1
@!> eK[sIVtncMnYnBit (u n ),nIN* kMnt;eday
2 1
u n 1 u n , n IN *
3 2
k> eKtag n IN* : v n 2u n 3 .
cUrbgajfa ( v n ) CasIVtFrNImaRtmYyEdleKnwgbBak;ersug .
x> KNnaplbUk S n v 1 v 2 v 3 .... v n CaGnuKmn_n n .
K> cUrKNnatY v n rYcTajrk u n CaGnuKmn_n n .
u1 1

@@> eK[sIVt (u n ),nIN* kMnt;eday u 1u 1 ,nIN*
n1 n
2 3
k> bgajfaeKGackMnt;tm k edIm,I[ v n 3u n k CasIVtFrNImaRt
mYyRKb; n IN * .
x> cUrKNna v n cMeBaHtm n Edl)anrkeXIjxagelI rYcTajrk u n
CaGnuKmn_n n .

eroberogeday lwm pln TMBr 25


sIVtncMnYnBit nig esrI

@#> eK[sIVtncMnYnBit (u n ), nIN* kMnt;eday


u 1 4

2 5
n 1
u u 2( 2n ) , n IN *

n
3 3
k> eKtag 1
v n u n 3n 2 .
2
cUrbgajfasIVt v n , nIN* CasIVtFrNImaRt .
x> KNnaplbUk S n v 1 v 2 v 3 .... v n CaGnuKmn_n n
K> cUrKNnatY v n rYcTajrk u n CaGnuKmn_n n .
@$> eK[sIVt (u n ) kMnt;eday u1 1 , u 2 1
nig n IN* : u n 2 5u n1 6u n .
eKtag v n u n1 2u n nig w n u n1 3u n RKb; n IN *
k> cUrbgajfa v n nig w n CasVItFrNImaRt .
rYcKNna v n nig w n CaGnuKmn_n n .
x> TajrktY u n CaGnuKmn_n n .

eroberogeday lwm pln TMBr 26


sIVtncMnYnBit nig esrI

emeronTI4
plbUktYnsIVtepSg
1-nimitsBaa sRmabplbUknsIVt
plbUk n tYdMbUgnsIVt u , u , u ,...., u kMnt;tageday
1 2 3 n
n
S n uk u1 u2 u3 ..... un
k 1

2-lkNplbUktYnsIVt
n
1> ..... n
k 1
n n
2> u u CacMnYnefr
k k
k 1 k 1
n n n n
3> u k v k wk uk v k w k
k 1 k 1 k 1 k 1

4> u v u 2 u v v
n n n n
2 2 2
k k k k k k
k 1 k 1 k 1 k 1

3-rebobKNnaplbUksIVtEdlmanTRmg
S n 1p 2p 3p ..... n p Edl p 1 ; 2 ; 3 ; ...
edIm,IKNnaplbUkenHeKRtUvGnuvtn_tamCMhanxageRkam
KNna (k 1)p 1 k p

eroberogeday lwm pln TMBr 27


sIVtncMnYnBit nig esrI

[tm k 1 ; 2 ; 3 ; .... ; n
eFIVviFIbUk .
]TahrN_ KNnaplbUk Sn 12 22 32 .... n2
eKman (k 1)3 k 3 3k 2 3k 1
n n n
eK)an [(k 1) k ] 3 k (3k 1)
3 3 2
k 1 k 1 k 1
n(4 3n 1)
(n 1)3 1 3Sn
2
3( 3n 5)
3Sn (n 1)3 1
2
n(n 1)( 2n 1)
3Sn
2
dUcenH Sn n(n 1)(6 2n 1) .
4-rebobKNnaplbUksIVtEdlmanTRmg
1 1 1 1
Sn .....
a1a 2 a 2a 3 a 3a 4 anan 1
Edl an 1 an d efr ehIy d 0 .
edIm,IKNnaplbUkenHeKRtUv
1 1 a a 1 1 1
bEmgtY . k 1 k
a a k k 1d a a d a
k k 1 k ak 1
[tm k 1 ; 2 ; 3 ; .... ; n rYceFIVviFIbUk .

eroberogeday lwm pln TMBr 28


sIVtncMnYnBit nig esrI

]TahrN_ KNnaplbUk
1 1 1 1
Sn ....
1.4 4.7 7.10 ( 3n 2)( 3n 1)
man (3k 2)(1 3k 1) 13 ( 3k1 2 3k1 1)
eK)an Sn 3 3k 2 3k1 1 13 (1 3n1 1)
1 n 1
k 1
dUcenH Sn 3nn 1 .
5-rebobKNnaplbUksIVtEdlmanTRmg
1 1 1 1
Sn .....
a1a2a3 a2a3a4 a3a4a5 a n a n 1a n 2
Edl a a d efr ehIy d 0 .
n 2 n

edIm,IKNnaplbUkenHeKRtUv
ak 1 1
bEmgtY
1 1 a 1
. k2
a a a d a a a
k k 1 k 2 k k 1 k 2 d ak ak 1 ak 1ak 2
[tm k 1 ; 2 ; 3 ; .... ; n
eFIVviFIbUk
]TahrN_ KNnaplbUk
1 1 1 1
Sn
1.2.3 2.3.4 3.4.5
....
n(n 1)(n 2)
.

eroberogeday lwm pln TMBr 29


sIVtncMnYnBit nig esrI

6-rebobKNnaplbUksIVtEdlmanTRmg
S n a1b1 a2b2 a3b3 ...... an bn
Edl (a ) CasIVtnBVnmanplsgrYm d nig (b ) CasIVtFrNImaRt
n n

manersug q .
edIm,IKNnaplbUkenHeKRtUvKNna S q S rYcTajrk S .
n n n

]TahrN_1 KNnaplbUk Sn 15 22 33 ..... nn ?


5 5 5
1 2 3 n
eKman Sn 2 3 ..... n (1)
5 5 5 5
nig 15 Sn 12 23 .... n n 1 nn1 (2)
5 5 5 5
eFIVpldksmIkar (1) & (2) GgnigGgeK)an
1 1 1 1 1 n
Sn Sn ( 2 3 .... n ) n 1
5 5 5 5 5 5
1
1 n
4 1 5 n 1 (1 1 ) n
Sn
5 5 1 1 5n 1 4 5n 5n 1
5
dUcenH Sn 165 (1 1n ) n n .
5 4 5

eroberogeday lwm pln TMBr 30


sIVtncMnYnBit nig esrI

]TahrN_2
eK[sIVtncMnYnBit u kMnt;eday
n n1

1 1
u
1
1
3
ni g TM n ak; T M n gkM e nI n u 1 u n 1
3
n1
n
n

k>KNna u CaGnuKmn_n n .
n

x>tag S u u u .... u .
n 1 2 3 n

edayeRbIplsg S 13 S cUrKNna S CaGnuKmn_n n .


n n n

dMeNaHRsay
k>KNna u CaGnuKmn_n n
n

eKman u 13 1 n1 u n 1
n1 n

eKTaj uu 13 . n n 1
n1

n
n 1 uk 1 n 1 1 k 1
eK)an
k 1 u
.



k 3 k
k 1
n 1
u2 u3 un 1 2 3 n
. ... . . .....
u1 u2 un 1 3 1 2 n1
un n
n 1
u1 3

eroberogeday lwm pln TMBr 31


sIVtncMnYnBit nig esrI

eKTaj u 3n .u eday u 13
n n 1 1 1

dUcenH u 3n .
n n

x> KNna S CaGnuKmn_n n


n

eKman S u 13 32 33 ... n3 1 3n
n
n

k 1
k 2 3 n1 n

ehIy 13 S 31 32 33 .... n3 1 3n
n 2 3 4 n n1

eK)an S 13 S 13 31 31 ... 31 3n
n n 2 3 n n1

1
1
2 1 3 n n
Sn . n1
3 3 1 1 3
3
2 1 1 n
Sn 1 n n 1
3 2 3 3
3 1
dUcenH Sn 1 n
4
n
3 2.3n 1
.

eroberogeday lwm pln TMBr 32


sIVtncMnYnBit nig esrI
n
7-sMKal eK[ S u u u u .... u
n k 1 2 3 n
k 1

edIm,IKNnaplbUkxagelIenHeKRtUv
sresrtY u Carag u k t k 1 t k b u k t k t k 1 ebIGac
k

krNIeKGacsresr u k t k 1 t k enaHeK)an
n n
Sn uk t k 1 t k t n 1 t1
k 1 k 1
krNIeKGacsresr u k t k t k 1 enaHeK)an
n n
S n u k t k t k 1 t 1 t n 1 .
k 1 k 1
]TahrN_1
eK[sIVtncMnYnBit (u ) kMnt;eday
n

2n 1
un
( n 2 1) n2 2n 2 ( n2 2n 2) n 2 1
Edl n 0 ; 1 ; 2 ; .... .
k> cUrsresr u Carag nA 1 n B2n 2
n 2 2

Edl A nig B CaBIrcMnYnBitRtUvkMnt; .


x> KNnaplbUk S u u u u ... u
n
n

k 0
k 0 1 2 n

CaGnuKmn_n n rYcTajrklImItvakalNa n .

eroberogeday lwm pln TMBr 33


sIVtncMnYnBit nig esrI

dMeNaHRsay
k>sresr u Carag
n
A
n2 1

B
n 2 2n 2
eyIg)an
2n 1
un
( n 2 1) n2 2n 2 ( n2 2n 2) n 2 1
2n 1

n 2 1 n 2 2n 2 ( n 2 1 n 2 2n 2 )
( 2n 1)( n2 2n 2 n 2 1 )

n2 1 n 2 2n 2 ( 2n 1)
n 2 2n 2 n 2 1
un
n 2 1 n 2 2n 2
n 2 2n 2 n2 1

n 1 n 2n 2
2 2
n 2 1 n 2 2n 2
1 1

n2 1 n 2 2n 2
dUcenH un
1
n 1
2
Edl 1
n 2n 2
2
. A 1 ; B 1

x> KNnaplbUk S u u u u ... u


n
n

k 0
k 0 1 2 n

eKman u n1 1 n 12n 2 sRmayxagelI


n 2 2

eroberogeday lwm pln TMBr 34


sIVtncMnYnBit nig esrI
1
eK)an n
Sn 2
k 0 k 1

1

k 2 2k 2
1 1 1 1 1
1

...
2 2 5 5 10
1 1
... 2
n 1 n 2n 2
2

1 n 2 2n 2 1
1
n 2n 2
2
n 2 2n 2
n 2 2n 2 1
dUcenH Sn
n 2n 2
2
niglim Sn 1
n
.
]TahrN_2
eK[sIVtncMnYnBit (u ) kMnt;eday
n
n

u
n n n
6
( 2 3 )( 2 3 )
n1
Edl n 0 ; 1 ; 2 ; 3 ; ....
n1

k> cUrepgpat;smPaB
3n 1 3n
un n 1
2 3n 1 2n 3n
x> KNnaplbUk S u u u u ... u
n
n

k 0
k 0 1 2 n

CaGnuKmn_n n rYcTajrklImItvakalNa n .

eroberogeday lwm pln TMBr 35


sIVtncMnYnBit nig esrI

dMeNaHRsay
n1 n

k> epgpat;smPaB u 2 3 3 2 3 3
n n1 n1 n n

eK)an u 3 ((22 33 ))(23 (2 3 )3 )


n1 n n n n1 n1

n n n n1 n1

3.6n 32 n 1 2.6n 32 n 1

( 2n 3n )( 2n 1 3n 1 )
6n
n
( 2 3n )( 2n 1 3n 1 )
Bit
3n 1 3n
dUcenH un n 1
2 3 n1
.
n
2 3n
x> KNnaplbUk S u u u u ... u
n
k 0
n

k 0 1 2 n

n1 n

eKman u 2 3 3 2 3 3
n n1 n1 n n

eK)an S 2 3 3 2 3 3
n k 1 k

n k 1 k 1

k k
k 0

eroberogeday lwm pln TMBr 36


sIVtncMnYnBit nig esrI
3 1 9 3 27 9 81 27

5 2 13 5 35 13 97 35
3 n 1 3n
...... n1 n
2 3 n 1
2 3 n

1 3 n 1 2n1 3n1 2.3n1
n 1
2 2 3 n 1 2( 2n1 3n1 )
3 n 1 2 n 1

2( 3n1 2n1 )
1 3n 1 2n 1
dUcenH Sn . n 1
2 3 2n 1
ehIy lim S n
n
1
2
.
]TahrN_3
eK[sIVtncMnYnBit (u ) kMnt;eday
n

1
un log 2 1 . log 2 n 2 n
n
Edl n 1 ; 2 ; 3 ; .... .
k>cUrbgajfa u 0 cMeBaHRKb; n 1
n

x>KNnaplbUk S u u u .... u CaGnuKmn_n n


n 1 2 3 n

K>kMnt; n edIm,I[ S 100 .


n

eroberogeday lwm pln TMBr 37


sIVtncMnYnBit nig esrI

dMeNaHRsay
k>bgajfa u 0 cMeBaHRKb; n 1
n

eKman u log 1 n1 . log n n


n 2 2
2

eday log 1 n1 log n n 1 log n 1 log n


2 2 2 2

nig log n n log n(n 1) log (n) log n 1


2
2
2 2 2

eK)an u log n 1 log n


n 2
2
2
2

cMeBaHRKb; n 1 eKman log (n 1) log n 2 2

dUcenH u 0 cMeBaHRKb; n 1 .
n

x>KNnaplbUk S u u u .... u n 1 2 3 n

eKman u log n 1 log n


n 2
2
2
2

-ebI n 1 : u log 2 log 1


1 2
2
2
2

-ebI n 2 : u log 3 log 2


2 2
2
2
2

-ebI n 3 : u log 4 log 3


3 2
2
2
2

--------------------------------------
-ebI n n : u log (n 1) log n
n 2
2
2
2

eFIVviFIbUkGg nig GgnsmPaBTaMgenHeK)an


eroberogeday lwm pln TMBr 38
sIVtncMnYnBit nig esrI

dUcenH S log n 1 .
n 2
2

K>kMnt; n edIm,I[ S 100 n

eyIg)an log n 1 100


2
2

log 2 n 1 10

n 1 210 1024
dUcenH n 1023 .
8-rebobrktYTI n tamplsgtYnsIVt
k-plsgtYlMdabTImYy
eKmansIVt a : a ; a ; a ; ..... ; a
n 1 2 3 n

ehIy b a a ; b a a ; b a a ; ..... enaeKfasIVt


1 2 1 2 3 2 3 4 3

b : b ; b ; b ; .... ; b CaplsgtYlMdabTImYynsIVt a .
n 1 2 3 n n

rUbmnKNnatY a n

eKman b a a
n n 1 n
n 1 n 1
eKn b (a a ) k k 1 k
k 1 k 1
n 1
eday (a k 1 ak ) (a2 a1 ) (a3 a2 ) ... (an an1 )
k 1

an a1

eroberogeday lwm pln TMBr 39


sIVtncMnYnBit nig esrI
n 1
eKn bk an a1
k 1
n 1
dUcen a a (b ) .
n 1 k
k 1

x-plsgtYlMdabTIBIr
eKmansIVt a : a ; a ; a ; ..... ; a ehIy
n 1 2 3 n

b1 a2 a1 ; b2 a3 a2 ; b3 a4 a3 ; .....; bn an1 an
CaplsgtYlMdabTImYynsIVt a
bn : b1 ; b2 ; b3 ; .... ; bn n
n 1
rUbmnKNnatY a KW a a (b ) .
n n 1 k
k 1

sIVt ( c ) CaplsglMdabTIBIrnsIVt (a ) KWCaplsg


n n

lMdabTImYynsIVt (b ) Edl c b b ; n 1 , 2 , 3 , ...


n n n 1 n
n 1
rUbmnKNnatYTI n KW b c c ; n 2 .
n 1 i
i 1

eroberogeday lwm pln TMBr 40


sIVtncMnYnBit nig esrI

lMhat;Gnuvtn_
cUrKNnaplbUk S u u u u ... u
n
n

k 1
k 1 2 3 n

Edl u CasIVtcMnYnBitkgkrNInImYyxageRkam
n n1

k> u n(n 11)(n 2)


n

2n 1
x> u n
n2 ( 2n 1)
6n 1
K> u n 1 .
n

( 3n 2)( 3n 1)
X> un
n
( 2n2 2n 1)( 2n2 2n 1)
6n
g> un n
( 3 2n )( 3n 1 2n 1 )
2n
c> un n
( 2 1) 2n 1 1 ( 2n 1 1) 2n 1
n2 n 1
q> un
2n
2n
C> un 2n
Edl
x 1
1 x

eroberogeday lwm pln TMBr 41


sIVtncMnYnBit nig esrI

emeronTI5
TMnak;TMngtYnsIVt
1-kMNt;tYTI n edayeRbIsIVtCMnYy
k> TMnak;TMngkMeNInTRmg; an 1 pan q
viFan
cMeBaHsIVt (an ) lsal;TMnak;TMngkMeNInTRmg; an 1 pan q
edIm,IKNnatY an eKRtUvGnuvtn_dUcxageRkam
tagsVItCMnYy rn n Edl , CacMnYnefrRtUvrk
CMnYs rn kgTMnak;TMngnsIVt an 1 pan q
eK)an rn 1 prn q eday rn n enaH rn1 (n 1)
naM[ (n 1) p(n ) q
n ( ) pn (p q )
p
eKTaj p q naM[ 0 , 1 q p .

KNnaplsg an 1 rn 1 CaGnuKmn_n an rn
tagsIVt bn an rn . KNna bn rYcTajrk an .

eroberogeday lwm pln TMBr 42


sIVtncMnYnBit nig esrI

]TahrN_
eKmansIVtcMnYnBit (an ) kMNt;eday
a1 11 nig an 1 4an 9 Edl n 1 , 2 , 3 , ...
KNna an CaGnuKmn_n n .
dMeNaHRsay
KNna an CaGnuKmn_n n
tagsIVtCMnYy rn n CMnYskgTMnak;TMng an 1 4an 9
eK)an rn 1 4rn 9
(n 1) 4(n ) 9
n ( ) 4n (4 9)
4
eKTaj 4 9 naM[ 0 , 3 ehIy rn 3

KNna an 1 3 (4an 9) 3 4(an 3)
tag bn an 3 enaH bn 1 an 1 3 4(an 3) 4bn
naM[ (bn ) CasIVtFrNImaRtmanersug q 4 nig b1 a1 3 11 3 8
eK)an bn b1 qn 1 8 4n 1 22n 1
dUcenH an bn 3 22n 1 3 .

eroberogeday lwm pln TMBr 43


sIVtncMnYnBit nig esrI

x>TMnak;TMngkMeNInTRmg; an 1 pan qn r
cMeBaHsIVt (an ) Edlsal;TMnak;TMngkMeNInTRmg; an 1 pan qn r
edIm,IKNnatY an eKRtUvGnuvtn_dUcxageRkam
tagsVItCMnYy rn n Edl , CacMnYnefrRtUvrk
CMnYs rn kgTMnak;TMngnsIVt a n 1 pa n qn r
eK)an rn 1 prn qn r
eday rn n enaH rn1 (n 1)
naM[ (n 1) p(n ) qn r
n ( ) (p q )n (p r )
p q
eKTaj p r RtUvedaHRsayrk nig

KNnaplsg an 1 rn 1 CaGnuKmn_n an rn
tagsIVt bn an rn . KNna bn rYcTajrk an .
]TahrN_
eKmansIVtcMnYnBit (an ) kMNt;eday
a1 10 nig an 1 3an 4n 9 Edl n 1 , 2 , 3 , ...
KNna an CaGnuKmn_n n .

eroberogeday lwm pln TMBr 44


sIVtncMnYnBit nig esrI

dMeNaHRsay
KNna an CaGnuKmn_n n
tagsIVtCMnYy rn n CMnYskgTMnak;TMng an 1 3an 4n 8
eK)an rn 1 3rn 4n 8
(n 1) 3(n ) 4n 8
n ( ) ( 3 4)n ( 3 8)
3 4
eKTaj 3 8 naM[ 2 , 3 ehIy rn 2n 3

KNna an 1 rn 1 Edl rn 1 2(n 1) 3 2n 1
eK)an an 1 (2n 1) 3an 4n 8 2n 1
an 1 2n 1 3[an 2(n 1) 1]
tag bn an 2(n 1) 1
enaH bn1 an1 2n 1 3bn
naM[ (bn ) CasIVtFrNImaRtmanersug q 3 nig b1 a1 1 10 1 9
eK)an bn b1 qn 1 9 3n 1 3n 1
dUcenH an bn 2n 3 3n 1 2n 3 .

eroberogeday lwm pln TMBr 45


sIVtncMnYnBit nig esrI

K> TMnak;TMngkMeNInTRmg; an 2 pan 1 qan 0


cMeBaHsIVt (an ) Edlsal;TMnak;TMngkMeNInTRmg; an 2 pan1 qan 0
edIm,IKNna an eKRtUvkMNt;cMnYnBIr nig EdlbMeBjlkxN
p nig q EdlCabsrbs;smIkar x 2 px q 0 .
smIkar x2 px q 0 ehAfasmIkarsmal;nsIVt .
TMnak;TMng an 2 pan 1 qan 0 kayeTACa
an 2 ( )an 1 an 0 EdlGacbMElgtamBIrrebobKW
an 2 an 1 (an 1 an ) (1)
an 2 a n 1 (a n 1 an ) ( 2)
TMnak;TMngxagelIenH KWCasIVtFrNImaRtEdlGacedaHRsayrk an )an .
]TahrN_1
eK[sIVtncMnYnBit (u ) kMnt;eday
n n1

u 5 ; u 13 nig TMnak;TMngkMenIn u 5u 6u
1 2 n 2 n1 n

Edl n 1 ; 2 ; 3 ; ... .
cUrKNna u CaGnuKmn_n n
n

dMeNaHRsay
KNna u CaGnuKmn_n n
n

eKman u 5u 6u smmUl u 5u 6u 0
n 2 n1 n n 2 n1 n

eroberogeday lwm pln TMBr 46


sIVtncMnYnBit nig esrI

mansmIkarsMKal; r 5r 6 0 2

b (r 2)(r 3) 0 eKTaj r 2 r 3 1 2

TMnak;TMng u 5u 6u GacsresrCaBIrrebobKW
n 2 n1 n

un 2 2un 1 3( un 1 2un ) i

un 2 3un 1 2( un 1 3un ) ii
tagsIVtCMnYyBIr a nig b kMnt;eday
n n1 n n1

a u 2u a u 2u


n1
enaH
n n1 n 2 n1

b n un1 3 u b u 3u
n n1 n 2 n1

edayeyagtamsmIkar i nig ii eKTaj a 3a nig b n1 n n1 2bn


TMnak;TMngenHbBaak;fa a nig b suTEtCasIVtFrNImaRt
n n

EdlmanpleFobrYmerogKa q 3 nig q 2 ehIymantYTI1 1 2

erogKa a u 2u 3 nig b u 3u 2
1 2 1 1 2 1

a a .q 3.3 3 n 1 n1 n

eK)an n 1 1

bn b1 .q2 n 1 2.2n 1 2n
un 1 2un 3n 1
eKTaj
un 1 3un 2 2
n

eFIVplsg 1 nig 2 GgnwgGgeK)an u n 3n 2n


dUcenH u 2 3 .
n
n n

eroberogeday lwm pln TMBr 47


sIVtncMnYnBit nig esrI

]TahrN_2
eK[sIVtncMnYnBit (u ) kMnt;eday n n1

u 6 ; u 27 nig TMnak;TMngkMenIn u 6u
1 2 n 2 n1 9un
Edl n 1 ; 2 ; 3 ; ... .
cUrKNna u CaGnuKmn_n n
n

dMeNaHRsay
KNna u CaGnuKmn_n n
n

TMnak;TMng u 6u 9u GacsresrCa
n 2 n1 n

un 2 3un 1 3( un 1 3un ) i
tagsIVtCMnYy v u 3u eK)an v u 3u
n n1 n n1 n 2 n1

tamTMnak;TMng i eK)an v 3v naM[ v CasIVtFrNImaRt


n1 n n

EdlmanpleFobrYm q 3 nigtY v u 3u 27 18 9 1 2 1

tamrUbmn v v q 9 3 3
n 1
n1 n 1 n1

eKTaj u 3u 3 ii
n1 n
n1

tagsIVt w u3 enaH w u3
n
n
n n1
n1
n1

eK)an w w u3 u3
n1 n
n1
n1
n
n

eroberogeday lwm pln TMBr 48


sIVtncMnYnBit nig esrI
un 1 3un
wn 1 wn n1
iii
3
yk (ii ) CYskg iii eK)an w w 1 efr n1 n

naM[ w CasIVtnBVnmanplsgrYm d 1 nig w u3 2


n 1
1

tamrUbmn w w n 1d 2 n 1 n 1
n 1

eday w u3 enaHeKTaj u 3 w 3 (n 1)
n
n
n n
n
n
n

dUcenH u 3 (n 1) .
n
n

]TahrN_3
eK[sIVtncMnYnBit (u ) kMnt;eday n n1

u 1 ; u 2 nig TMnak;TMngkMenIn u 2u 4u
1 2 n 2 n1 n

Edl n 1 ; 2 ; 3 ; ... .
cUrKNna u CaGnuKmn_n n
n

dMeNaHRsay
KNna u CaGnuKmn_n n
n

eKman u 2u 4u smmUl u 2u 4u 0
n 2 n1 n n 2 n1 n

mansmIkarsMKal; r 2r 4 0 ' 1 4 3i
2 2

manbs r 1 i 3 ; r 1 i 3
1 2

eroberogeday lwm pln TMBr 49


sIVtncMnYnBit nig esrI

tagsIVt Z n un 1 (1 i 3 )un n 1
eK)an Z n1 un 2 (1 i 3 )un 1

Z n 1 2un 1 4un (1 i 3 )un 1


Z n 1 (1 i 3 )un 1 4un
Z n 1 (1 i 3 ) [ un 1 (1 i 3 )un ]

eKTaj Zn1
Zn
1 i 3 2e 3
i

n 1 Z n i 3
eK)an k 1
2 e
k 1
k 1 Z
k
( n 1 ) ( n 1 )
Z2 Z3
. ....
Z1 Z 2 Z n 1
Zn
2 .e 3
n 1
i Z
Z1
n
2 .e 3
n 1
i
b

eday Z u (1 i 3 )u 1 i 3 2e
1 2 1
i
3

eKTaj Z 2 e 2 cos n3 i . sin n3


n
i
n 3 n
n

eday Z u (1 i 3 )u n 1
n n1 n

n
b Z (u u ) i 3 u eKTaj 3u 2
n n1 n n n
n
sin
3
dUcenH u 23 sin n3 .
n

eroberogeday lwm pln TMBr 50


sIVtncMnYnBit nig esrI

n q
X> TMnak;TMngkMeNInTRmg; an 1 pa
ra s n
n q
cMeBaHsIVt (an ) Edlsal;TMnak;TMngkMeNInTRmg; an 1 pa
ra s
n
edIm,IKNnatY an eKRtUv
px q
edaHRsaysmIkarsmal; x b rx 2 (s p )x q 0 (*)
rx s
ebIsmIkar (*) manbsBIrepSgKa nig enaHeKRtUvtagsIVtCMnYy
a
bn n
a
rYcRtUvRsayfa (bn ) CasIVtFrNImaRt .
n
ebIsmIkar (*) manbsDb x1 x2 enaHeKRtUvtagsIVtCMnYy
1
bn
an
rYcRtUvRsayfa (bn ) CasIVtnBVn.

]TahrN_1
eK[sIVtncMnYnBit (u ) kMnt;eday
n n1

3u 8
u 4 nig TMnak;TMngkMeNIn u n1
n

u 5
1
n

Edl n 1 ; 2 ; 3 ; ... .
k> edaHRsaysmIkar r 3rr58
edaytag r nig r Cabs ehIy r r .
1 2 1 2

eroberogeday lwm pln TMBr 51


sIVtncMnYnBit nig esrI

x> eKBinitsIVt v uu rr Edl n 1 ; 2 ; 3 ; ..... .


n
n 1

n 2

bgajfa (v ) CasIVtFrNImaRtrYcKNna v CaGnuKmn_n n .


n n

K> cUrKNna u CaGnuKmn_n n .


n

dMeNaHRsay
k>edaHRsaysmIkar r 3rr58
ebI r 5 enaHsmIkarGacsresr r 2r 8 0 2

b (r 1) 9 (r 2)(r 4) 0
2

eKTaj r 2 ; r 4 .
1 2

x> bgajfa (v ) CasIVtFrNImaRt


n

eKman v uu rr uu 24 ehIy v uu 24
n
n 1 n
n1
n1

n 2 n n1
3un 8
eday u n1
un 5
eK)an
3un 8
2
un 5 u 2 1
vn 1 n vn
3un 8
4 7 un 28 7
un 5
dUcenH (v ) CasIVtFrNImaRtmanpleFobrYm q 71 .
n

eroberogeday lwm pln TMBr 52


sIVtncMnYnBit nig esrI

KNna v CaGnuKmn_n n
n

u1 2 4 2 1
tamrUbmn v v q eday v
n 1
n 1
1
u1 4 4 4 4
dUcenH v 14 71 .
n n 1

K> KNna u CaGnuKmn_n n


n

eKman v uu 24 naM[ u 41v v 2 eday v


n
n
n
n
n
1 1
n 1
4 7
n n

4 1 2 . 7 n 1
.
dUcenH un
4 . 7n 1 1
]TahrN_2
eK[sIVtncMnYnBit (u ) kMnt;eday
n n1

5u 8
u 3 nig TMnak;TMngkMeNIn u n1
n

2u 3
1
n

Edl n 1 ; 2 ; 3 ; ... .
k> edaHRsaysmIkar r 52rr 83 tag r Cabs 0

x> eKBinitsIVt v u 1 r Edl n 1 ; 2 ; 3 ; ..... .


n
n 0

bgajfa (v ) CasIVtnBVnrYcKNna v CaGnuKmn_n n .


n n

K> cUrKNna u CaGnuKmn_n n .


n

eroberogeday lwm pln TMBr 53


sIVtncMnYnBit nig esrI

dMeNaHRsay
k>edaHRsaysmIkar
ebI r 32 enaHsmIkar r 52rr 83 Gacsresr
2r 8r 8 0 b 2 r 2 0
2 2

dUcenH r 2 CabsDb .
0

x>bgajfa (v ) CasIVtnBVn
n

eKman v u 1 r u 1 2 ehIy v
n n1
1
un 1 2
n 0 n
5un 8
eday un 1
2un 3
2u 3
eK)an vn 1
1
5un 8
n 2
1
2 vn
2 un 2 un 2
2un 3
eday v v 2 b v v 2 efr
n1 n n1 n

dUcenH (v ) CasIVtnBVn manplsgrYm d 2 .


n

KNna v CaGnuKmn_n n
n

tamrUbmn v v (n 1)d eday v u 1 2 1


n 1 1
1

eroberogeday lwm pln TMBr 54


sIVtncMnYnBit nig esrI

dUcenH v 1 2(n 1) 2n 1 .
n

K>KNna u CaGnuKmn_n n
n

eday v u 1 2 eKTaj u 1 v2v


n n
n

1 2( 2n 1)
2n 1
n n
4n 1
dUcenH un
2n 1
.
2-TMnak;TMngrvag an nig Sn
cMeBaHRKb; n 2 eK)an Sn Sn 1 an nig S1 a1
Edl Sn a1 a2 a3 .... an .
]TahrN_
eKmanplbUk n tYdMbUgnsIVtcMnYnBit (an ) mYykMNt;eday Sn n3 2n
kMNt;tYTI n nsIVt (an ) .
dMeNaHRsay
kMNt;tYTI n nsIVt (an )
tamrUbmn an Sn Sn 1
eday Sn n3 2n naM[ Sn 1 (n 1)3 2(n 1)
eK)an an n3 2n (n 1)3 2(n 1) 3n2 3n 3
dUcenH an 3(n2 n 1) .

eroberogeday lwm pln TMBr 55


sIVtncMnYnBit nig esrI

lMhat;Gnuvtn_
!> cUrkMnt;tY u nsIVt u edaysal;TMnak;TMngkMeNIn
n n n1

kgkrNInImYyxageRkam
k> uu 2 2u 2
1

n1 n

13
u
1 2
x>
u 1 u 3
n 1 2 n
u1 0
K>
un 1 ( 3 2 )un 3 2 1
@> cUrkMnt;tY u nsIVt u edaysal;TMnak;TMngkMeNIn
n n n1

kgkrNInImYyxageRkam
k> uu 1; 2uu 2 3u
1 2

n 2 n1 n

u1 1 ; u2 4
x>
un 2 6un 1 9un
u1 0 ; u2 1
K>
un 2 3 un 1 4un
u1 u2 1 ; u3 4
X>
un 3 9un 2 26un 1 24un

eroberogeday lwm pln TMBr 56


sIVtncMnYnBit nig esrI

#> cUrkMnt;tY u nsIVt u edaysal;TMnak;TMngkMeNIn


n n n1

kgkrNInImYyxageRkam
u1 7 u1 2

k>
u
4un 5 x>
u
3( un 3)
n1 u 2 n1 4un 9
n
$> eK[sIVtncMnYnBit (U ) kMnt;eday n

3 5 5 1
U 0 , U1
2 2
n IN : U U U
n1 n n 1

3 5 1 5
k> n IN * eKyk
Vn
2
Un
2
U n 1 .
cUrRsayfa (V ) CasIVtFrNImaRtrYcsresr V GnuKmn_ n .
n n

x> cUrRsayfa U U U U U U , n 1
n
2
n1
2
n n 2 n 1 n1

rYcTajbBaak;TMnak;TMng U U U 0 . n
2
n 1 n1

K> rkRbePTnsIVt (U ) rYcsresr U CaGnuKmn_n n .


n n

X> eKyk W ln U ehIyeKman


n n

S W W ... W & P U .U .U ....U .


n 0 1 n n 0 1 2 n

KNna S rYcTajrk P CaGnuKmn_n n .


n n

eroberogeday lwm pln TMBr 57


sIVtncMnYnBit nig esrI

%> eK[sIVtncMnYnkMupic ( Z ) kMnt;edayTMnak;TMng


n

Z 0 0 , Z1 1

nig n IN : Z 1 22 i Z 1 2i Z
n 2 n 1 n

k> tag n IN :U Z Z . n n1 n

cUrbgajfa U 1 2i Un1 n

x> RsaybBaak;fa U cos n4 i . sin n4 .


n

K> tag S U .
n
n

k 0
k

cUrRsayfa Z S rYcTajrk Z CaGnuKmn_n n .


n1 n n

^> eKman (U ) CasIVtmYykMnt;eday n IN : U a a n n 1


n n

k> bgajfasIVt V 1 1 , n 2 CasIVtnBVn


U n U n 1 U n 1 U n 2
n

x> KNna S V kgkrNIEdl a 14 .


n
n

k 2
k

&> eK[ S 1 32 33 34 ......... n3 1


n 2 4 6 2n

cUrKNna S niglImItrbs;va .
n

eroberogeday lwm pln TMBr 58


sIVtncMnYnBit nig esrI

*> eK[ (U ) CasIVtnBVnmantYU , U


n 1 2 , U 3 , ........, U n
nigplsgrYm d 4k , k Z
cUrRsaybBaak;rUbmn
nd U1 U n
. Sin Sin
SinU k SinU 1 SinU 2 ..... SinU n
n
2 2
k 1 d
sin
2
nd U Un
Sin . Cos 1
CosU k CosU1 CosU 2 ..... CosU n
n
2 2
k 1 d
sin
2
(> eK[sIVtFrNImaRt (U ), n IN * EdlmantYsuTEtviCman
n

nigersug q . eK]bmafa (V ), n IN * CasIVtkMnt;eday


n

Vn log a (U n ) , a 0 , a 1
k> cUrRsayfa (V ), n IN * CasIVtnBVn .
n

x> eKtag S V . cUrRsayfa S n2 . log U .U


n
n

k 1
k n a 1 n

K> tag P U . cUrbgajfa P a
n
n

k 1
n n
Sn

X> Tajfa P (U U ) .
n 1 n
n

eroberogeday lwm pln TMBr 59


sIVtncMnYnBit nig esrI

!0> eK[sIVtncMnYnBitkMnt;eday
U 0 0 , U 1 1

U n 2 2U n1 4U n , n IN
k>eKtag n IN : Z U (1 i 3 )U .n1 n1 n

bgajfa Z (1 i 3 ) Z .
n1 n

x> cUrbgajfa Z 2 (cos n3 i . sin n3 ) .


n
n

K> TajrktYtUeTA U CaGnuKmn_n n .


n

!!> eKmansIVtncMnYnkMupic ( Z ) kMnt;eday


n

Z0 2

3i 2 3i
n1
Z Z , n IN
2
n
2
k> eKtag n IN :U Z 1 . n n

cUrbgajfa U 32 i .U rYcTajrk U CaGnuKmn_n n


n1 n n

x> RsaybBaak;fa Z 2 cos n12 (cos n12 i . sin n12 ) .


n

eroberogeday lwm pln TMBr 60


sIVtncMnYnBit nig esrI

emeronTI6
vicarGnumanrYmKNitviTa
1>eKalkarN_nvicarGnumanrYmKNitviTa
niymny
P (n) CasMeNIEdlTak;TgnwgcMnYnKt; n
edIm,IRsaybBaak;fa P (n) BitcMeBaHRKb; n IN * eKRtUv
!> epgpat;fa P (n) BitcMeBaH n 1
@> ]bmafa P (n) BitcMeBaHtm n
#> RsaybBaak;fa P (n) BitnaM[)an P (n 1) Bit
2>c,ab;eTVFa
RTwsIbT
n
(x y )
n
C(n , k ) xn k y k
k 0

Edl C(n , k ) k! (nn! k )! .

eroberogeday lwm pln TMBr 61


sIVtncMnYnBit nig esrI

]TahrN_1
u0 5 1
eK[sIVt un kMnt;eday un 1 un 2 2un
Edl n 0 , 1 , 2 , ..... .
edayeRbIGnumanrYmKNitviTacUrRsayfa
2n 2n
5 1 5 1
un 1 .
2 2
dMeNaHRsay
Rsayfa
2n 2n
5 1 5 1
un 1
2 2
- cMeBaH n 0
eK)an u 52 1 52 1 1
0 5 1 Bit .
]bmafavaBitdl; n k KW
2k 2k
5 1 5 1
uk 1 Bit
2 2

eroberogeday lwm pln TMBr 62


sIVtncMnYnBit nig esrI

eyIgnwgRsayfavaBitdl; n k 1 KW
2k 1 2k 1
5 1 5 1
uk 1 1 Bit
2 2
eKman u
k 1 uk 2uk uk 1
2
1 2

2k 2k
5 1 5 1
eday uk 1
2 2
eK)an
2
2k 2k
5 1 5 1
uk 1 1
2 2

2k 1 2k 1
5 1 5 1
uk 1 2 1
2 2
2k 1 2k 1
5 1 5 1
uk 1 1 Bit
2 2
2n 2n
5 1 5 1
dUcenH un 1 .
2 2

eroberogeday lwm pln TMBr 63


sIVtncMnYnBit nig esrI

]TahrN_2
eKmanGnuKmn_ f ( x ) 5 x 9 x 15 x 3
3 2

k> Rsayfa f ( x ) CaGnuKmn_ekInCanicelIEdnkMnt;rbs;va .


x> eKBinitsIVt a nig b kMnt;eday a f1(1) nigcMeBaH
n n 0 3

1 an
RKb; n IN : a n1
f ( an )
1
nig b
n 2
1 an
.
an f
3

an
cUrrkTMnak;TMngrvag b nig b . n1 n

3n
2
K>edayeRbIGnumanrYmKNitviTacUrRsayfa b n
3
.
X> Tajrk a CaGnuKmn_n n .
n

dMeNaHRsay
k> Rsayfa CaGnuKmn_ekInCanic
f ( x)
f ( x) 5 x3 9 x 15 x 3 manEdnkMnt; D IR
2

edrIev f ' ( x ) 15 x 18 x 15 0 x D
2

eRBaH a 15 0 nig ' 81 225 144 0


dUcenH f ( x ) CaGnuKmn_ekInCanicelIEdnkMnt;rbs;va .

eroberogeday lwm pln TMBr 64


sIVtncMnYnBit nig esrI

x> rkTMnak;TMngrvag b nig b n1 n

eKman b 2 11 aa enaH b 2 11 aa
n
n
n1
n1

n n1

eday a n1
f ( an )
1
an f
3

an
Et f (a ) 5a 9a 15a 3
n n
3
n
2
n

ehIy f a1 a5 a9 15 3 3 2
n n
a
n n

1
b an
3
f 5 9an 15an 3an
2 3

an
5a 9an 15an 3
3 2

naM[ an 1 n
5 9an 15an 3an
2 3

8(1 an )3
eKman 1 an 1
5 9an 15an 3an
2 3

2(1 an )3
ehIy 1 an 1
5 9an 15an 3an
2 3

8(1 an )3
eKTaj bn 1
1 an 3 n
b
3

dUcenH b n 1 bn
3
.

eroberogeday lwm pln TMBr 65


sIVtncMnYnBit nig esrI
3n
2
K>edayeRbIGnumanrYmKNitviTacUrRsayfa b n
3
cMeBaH n 0 eK)an b 2 11 aa
0
0

eday a 0 3
1

1

f (1) 3 5 9 15 3 2
1

1
1 30

eK)an b0 2 2
2 2
Bit
1 3 3
1
2
3k
2
]bmafavaBitcMeBaH n k KW b k
3
Bit
3k 1
2
eyIgnwgRsayfa vaBitcMeBaH n k 1 KW b k 1
3
eKman b b tamsRmaykgsMnY x
k 1 k
3

3k

Ettamkar]bma b 23
k

3
2 3 k 2
3k 1

eK)an bk 1 Bit
3 3
3n
2
dUcenH bn
3
.

eroberogeday lwm pln TMBr 66


sIVtncMnYnBit nig esrI

X> Tajrk a CaGnuKmn_n n


n

eKman b 2 11 aa eK)an b a b
n
n
n n n 2 2an
n
3n
2 bn 2
naM[ a n
2 bn
eday bn
3
3n
2
2
3 2 3 2
3n 3n

dUcenH an
2
3n
2 3 3n
2 3n

2
3
lMhat;Gnuvtn_

0u tan
11
!> eK[sIVt
3 u u
3
un 1 n n
; n IN
1 3un
2

3n
edayeRbIGnumanrYmKNitviTacUrRsayfa un tan
11
.
u0 5
@> eK[sIVt
un 1 un 2 2 ; n 0 , 1 , 2 , ...
edayeRbIGnumanrYmKNitviTacUrRsayfa
2n 2n
5 1 2
un



.
2 5 1

eroberogeday lwm pln TMBr 67


sIVtncMnYnBit nig esrI

emeronTI7
lImItnsIVt nig esrI
1>lImItnsIVt
k>sIVtGnn
sIVt a1 , a2 , ....an , ... EdlmantYeRcInGnnehAfasIVtGnn .
x>sIVtrYm
sIVt a1 , a2 , ....an , ... CasIVtrYmxiteTArkcMnYnBit ebI nlim

an .

K>sIVtrIk
sIVt a1 , a 2 , ....an , ... CasIVtrIkxiteTArk ebI lim an .
n
sIVt a1 , a 2 , ....an , ... CasIVtrIkxiteTArk ebI lim an .
n
X>RbmaNviFIelIlImIt
eKmansVIt (a ) nig (b ) Edlman lim a M
n n
n
n

nig lim b N eK)an


n
n

1> lim ka k .M
n
n

2> lim (a b ) M N , lim (a b ) M N


n
n n
n
n n

eroberogeday lwm pln TMBr 68


sIVtncMnYnBit nig esrI

3> lim (a b ) M.N


n
n n

an M
ebI N 0 enaH nlim
b

N n

g>lImItsVItFrNImaRtGnn
ebI r 1 enaH lim r ehIy r n CasVItrIkeTArk
n
n

ebI r 1 enaH (r n ) CasIVtefrehIy lim r 1 . n


n

ebI r 0 enaH (r n ) CasIVtefrehIy lim r 0 . n


n

ebI r 1 enaHsIV (r n ) CasIVtqas;ehIykalNa n


eKminGackMNt;lImItn (rn ) )aneT .
sVItFrNImaRtGnnEdlrYm sVIt (r n ) smmUl 1 r 1
2>lImItnsIVt
k>esrIGnn

esrIGnn an a1 a 2 ... an ...
n 1
KWCaplbUktYnsIVtGnn (an ) .
kenSam Sn a1 a 2 .... an ehAfaplbUkedayEpknesrIGnn

an a1 a2 a3 .... an .... .
n 1

eroberogeday lwm pln TMBr 69


sIVtncMnYnBit nig esrI

x>esrIrYm nig esrIrIk


ebIplbUkedayEpk Sn a1 a 2 .... an rYmxiteTArkcMnYnBit S

enaHeKfa (an ) CaesrIrYmxiteTArk S .
n 1

ebIesrI (an ) CaesrIrYmenaH lim an 0
n
n 1

ebIsIVt (an ) minrYmrk 0 eTenaH (an ) CaesrIrIk .
n 1

K>PaBrYmnigrIknesrIFrNImaRtGnn
RKb;esrIFrNImaRtGnn a ar ar ar .... ar 2 3 n 1
...
Edl a 0 CaesrIrYm b rIkeTAtamkrNIdUcxageRkam
a
ebI | r | 1 enaHesrIrYmeTArk .
1 r
ebI | r | 1 enaHesrIrIk .
X>lkNnesrIGnn

ebIesrI an nig bn CaesrIrYmenaHeK)an
n 1 n 1

1> kan k an k Edl k CacMnYnefr .
n 1 n 1

2> (an bn ) an bn .
n 1 n 1 n 1

eroberogeday lwm pln TMBr 70


sIVtncMnYnBit nig esrI

]TahrN_ 1
cUrsikSaPaBrYm b rIknsIVtcMnYnBit (u ) kMNt;eday
n

8n 3 3n 1
un
( 2n 1) n 4 n 2 1
dMeNaHRsay
sikSaPaBrYmbrIknsIVt (u )
n

8n 3 3n 1
eyIg)an lim u lim
( 2n 1) n 4 n 2 1
n
n n

3 1
n 3 (8 2 3 )
lim n n
n 1 1 1
n( 2 ) .n 2 1 2 4
n n n
3 1
8 2 3 8
lim n n 4
n 1 1 1 2
(2 ) 1 2 4
n n n
dUcenH (u ) CasIVtrYmxiteTArk 4 .
n

eroberogeday lwm pln TMBr 71


sIVtncMnYnBit nig esrI

]TahrN_ 2
cUrsikSaPaBrYm b rIknsIVtcMnYnBit (u ) kMNt;eday
n

n2
un
n2 n 1 n 1
dMeNaHRsay
sikSaPaBrYmbrIknsIVt (u ) n

n2
eK)an lim u lim
n2 n 1 n 1
n
n n

n2 ( n2 n 1 n 1)
lim
n (n 2 n 1) (n 1)
n2 ( n2 n 1 n 1)
lim
n n2
lim ( n 2 n 1 n 1 )
n

dUcenH (u ) CasIVtrIkxiteTArk .
n

kMNtcMNaM
sIVt (a ) : a , a , a , ..., a , ... CasIVtrYmxiteTArkcMnYnBit ebI
n 1 2 3 n

a manlImItesI kalNa n .
n

sIVt (a ) : a , a , a , ..., a , ... CasIVtrIkxiteTArk b


n 1 2 3 n

ebI a manlImIt b kalNa n .


n

eroberogeday lwm pln TMBr 72


sIVtncMnYnBit nig esrI

]TahrN_3
eKmansIVtcMnYnBit (a ) kMNt;RKb; n IN * eday a 52
n 1

nig TMnak;TMngkMeNIn a 12 a 1 .
n1 n

KNna a CaGnuKmn_n n rYcTajfa (a ) CasIVtrYm .


n n

dMeNaHRsay
KNna a CaGnuKmn_n n
n

smIkarsmal;rbs;sIVt a 12 a 1 KW c 12 c 1
n1 n

eKTaj 2c c 2 b c 2 .
tagsIVtCMnYy b a c a 2
n n n

eK)an b a 2
n 1 n 1

1
( an 1) 2
2
1
an 1
2
1 1
(a n 2 ) b n
2 2
eday b n1
1
bn
2
naM[
(b n ) CasIVtFrNImaRtmanpleFobrYm q 12
tamrUbmn b n b1 q n 1 eday b a 2 52 2 12
1 1

eroberogeday lwm pln TMBr 73


sIVtncMnYnBit nig esrI

ehtuenH b 12 ( 12 ) 21 .
n
n 1
n

eday b a 2 naM[ a b 2
n n n n

dUcenH a 21 2 .
n n

ma:geToteday lim a lim ( 21 2) 2 eRBaH lim 21


n
n
n n n n
0 .
dUcenH (a ) CasIVtrYmxiteTArk 2 .
n

kMNtcMNaM
edIm,IkMNt;tYTI n nsIVt (a ) : a , a , a , ..., a , ... EdlkMNt;eday
n 1 2 3 n

TMnak;TMngkMeNIn a p a q eKRtUvGnuvtn_dUcxageRkam
n 1 n

!> edaHRsaysmIkarsmal; c p.c q


@> tagsIVtCMnYy b a c n n

#> Rsayfa (b ) CasIVtFrNImaRt rYcKNna b .


n n

$> TajrktY a BITMnak;TMng b a c KW a b c .


n n n n n

eroberogeday lwm pln TMBr 74


sIVtncMnYnBit nig esrI

]TahrN_4
eKmansIVtcMnYnBit (a ) kMNt;eday a 2 , a 4
n 1 2

nig TMnak;TMngkMeNIn a 32 a 12 a .
n 2 n1 n

KNna a CaGnuKmn_n n rYcTajfa (a ) CasIVtrYm .


n n

dMeNaHRsay
KNna a CaGnuKmn_n n
n

smIkarsmal;nsIVt a 32 a 12 a KW x 32 x 12
n 2 n1 n
2

b 2x 3x 1 0 manbs x 1 ; x ac 12
2
1 2

b n an 1 an
b n an 1 x1an
tagsIVtCMnYy b 1
c n a n 1 x 2a n n
c a n1 an
2
eKman b n1 an 2 an 1
3 1
( an 1 an ) an 1
2 2
1
(a n 1 a n )
2
1
bn
2
naM[ (b ) CasIVtFrNImaRtmanersug q 12 nigtY b
n 1 a 2 a1 2

eroberogeday lwm pln TMBr 75


sIVtncMnYnBit nig esrI

eK)an b b q 2 21
n 1
n 1
n 1

1
2n 2
.
eKTaj a a 2 1 (1)
n 1 n n2

ma:geTot c a 12 a
n1 n 2 n1

3 1 1
( an 1 an ) an
2 2 2
1
an 1 an
2
cn
naM[ (c ) CasIVtefrRKb; n IN * .
n

eK)an c c Et c a 12 a 4 1 3 ehtuenH c
n 1 1 2 1 n 3 .
eKTaj a 12 a 3 (2)
n 1 n

eFVIpldkrvagsmIkar (1) nig (2) eK)an


1 1
(a n 1 a n ) (a n 1 a n ) n 2 3
2 2
1 1
an n 2 3
2 2
dUcenH .
1
an n 3 6
2
ma:geToteday n n 2
1
lim an lim ( n 3 6) 6

dUcenH CasIVtrYmxiteTArk .
(a n ) 6

eroberogeday lwm pln TMBr 76


sIVtncMnYnBit nig esrI

]TahrN_5
eKmansIVtcMnYnBit (a ) kMNt;eday a 0
n 1

nig TMnak;TMngkMeNIn a 3 2a Edl n 1 , 2 , 3 , ... .


n1
n

KNna a CaGnuKmn_n n rYcTajfa (a ) CasIVtrYm .


n n

Solution :
KNna a CaGnuKmn_n n
n

smIkarsmal;nsIVt a 3 2a KW x 3 2 x
n1
n

b x 3x 2 0 manbs x 1 ; x 2
2
1 2

tagsIVtCMnYy b aa xx aa 12
n
n 1 n

n 2 n
an 1 1
eK)an bn1
an 1 2
a eday
n1
2
3 an
2
1
3 an a 1
bn 1 n
2 2an 4
2
3 an
1 a 1 1
bn 1 . n bn
2 an 2 2
a1 1 1
naM[ (b ) CasIVtFrNImaRtmanersug q 12 nigtY b
n 1
a1 2 2

eroberogeday lwm pln TMBr 77


sIVtncMnYnBit nig esrI

tamrUbmn b b q 12 21
n 1
n 1
n 1

1
2n
tamTMnak;TMng b aa 12
n
n

n
2
2b n 1 2n 1
eKTaj a b n n 2b n an 1 naM[ an
an 1

1
1
2n
2 2n 2 2n
dUcenH an
1 2n
. ma:geToteday lim an lim (
n n 1 2n
)1

dUcenH (a n )CasIVtrYmxiteTArk 1 .
]TahrN_6
eKmansIVtcMnYnBit (a ) kMNt;eday a 3
n 1

nig TMnak;TMngkMeNIn Edl n 1 , 2 , 3 , ... .


KNna a CaGnuKmn_n n rYcTajfa (a ) CasIVtrYm .
n n

dMeNaHRsay
KNna a CaGnuKmn_n n
n

smIkarsmal;nsIVt a 3aa 14 KW x 3xx14


n 1
n

b x 4x 4 0 manbsDb x x 2
2
1 2

tagsIVtCMnYy b a 1 2
n
n

eroberogeday lwm pln TMBr 78


sIVtncMnYnBit nig esrI
an 1
eK)an b n1
1

an 1 2 3an 4
1

an 2
2
an 1
a 1 a 2
eKTaj bn 1 bn n
1
n
an 2 an 2 an 2
1 efr
dUcenH (b ) CasIVtnBVnmanplsgrYm d 1 nigtY b
n 1
1
a1 2
1

tamrUbmn b b (n 1)d 1 (n 1) n
n 1

tam b a 1 2 eKTaj a 2 b1 2 n1
n n
n n

dUcenH a 2 n1 .
n

ma:geToteday lim a lim (2 n1 ) 2 eRBaH lim n1 0


n
n
n n

dUcenH (a ) CasIVtrYmxiteTArk 2 .
n

eroberogeday lwm pln TMBr 79


sIVtncMnYnBit nig esrI

]TahrN_7
eKmansIVtcMnYnBit (a ) kMNt;eday
n

1 1 1
an (1 )(1 ) ....(1 n )
2 4 22
cUrKNnalImIt .
lim an
n

dMeNaHRsay
KNnalImIt lim a n
n

yk q 12 eK)an
n
an (1 q )(1 q 2 )......(1 q 2 )
x2 y 2
edayeRbIsmPaB xy
xy
eK)an
n 1
1 q2 1 q4 1 q2
an . ....
1 q 1 q2 1 q2
n

2n 1
1
2n 1 1 1 2n 1
1q 2
an 2 1
1q 1 2
1
2
2n 1

kalNa n enaH 12 xitCitsUn



dUcenH lim a 2 .
n
n

eroberogeday lwm pln TMBr 80


sIVtncMnYnBit nig esrI

]TahrN_8
KNnalImIt
n n n
lim 4 ....
n
n 1 n 4
2 n 4
n
Solution :
KNnalImIt
tag S n n .... n
n 1 n 2 n n
n 4 4 4

cMeBaHRKb; n 1 nig k IN * eKyk 1 k n


eK)an n 1 n k n n
4 4 4

b 1

1

1
n4 n n4 k n4 1
b n

n

n
n4 n n4 k n4 1
n
n n n n
eK)an 4 4 4
n
k 1 n n k 1 n k k 1 n 1
n2 n2
b Sn
n4 n n4 1
n2 n2
eday lim 1 nig lim 1
n
n n
4 n
n 1
4

n
dUcenH lim Sn lim 4
n
....
n
1 .
n n
n 1 n4 2 n4 n

eroberogeday lwm pln TMBr 81


sIVtncMnYnBit nig esrI

]TahrN_9
KNnalImIt
23 1 33 1 43 1 n3 1
lim [ 3 .... 3 ]
n 2 1 33 1 43 1 n 1
dMeNaHRsay
KNnalImIt
tag P 22 11 33 1 43 1 n3 1
3 3
3 .... 3
1 4 1 n 1
n 3 3

k 3 1 (k 1)(k 2 k 1)
eKBinit
k 1 (k 1)(k 2 k 1)
3

n
k3 1 n
k 1 n k2 k 1
eK)an Pn [ 3
k 1
] [
k 1
] [ 2
k k 1
]
k2 k2 k2

2 n n1
2

n(n 1) 3
2 n(n 1) 1
.
3 n(n 1)
2 1
1
n(n 1)
3
1 2
dUcenH n
2
lim Pn lim 1
3 n

n(n 1) 3
.

eroberogeday lwm pln TMBr 82


sIVtncMnYnBit nig esrI

]TahrN_10
eK[plbUk
1 1 1 1 1 1 1
Sn
n n 2 (n 1) 2
1 1 .... 1
22 32 32 42
cUrKNna lim S .
n
n

dMeNaHRsay
KNnalImIt lim S
n
n

k 2 (k 1)2 (k 1)2 k 2
eKBinit 1 k1 (k 1 1)
2 2

k 2 (k 1)2
k 2 (k 1)2 k 2 2k 1 k 2

k 2 (k 1)2
k 2 (k 1)2 2k (k 1) 1

k 2 (k 1)2


k (k 1) 1
2

1
1
2

k 2 (k 1)2 k (k 1)

2
1 1
1
k k 1
1 n 1 1
eK)an Sn 1
n k 1
1
(n 1
k k 1 n
1
n1
) 1
1
n1
1
dUcenH lim S n lim 1 1
n 1
.
n n

eroberogeday lwm pln TMBr 83


sIVtncMnYnBit nig esrI

]TahrN_11
eKmanesrIGnnmYy

1 1 1 1 1
n(n 1) 1.2 2.3 3.4
...

....
n 1 n ( n 1)
cUrsikSaPaBrYm b rIknesrIxagelIenH .
dMeNaHRsay
sikSaPaBrYm b rIknesrI

1 1 1 1 1
n(n 1) 1.2 2.3 3.4 ... n(n 1) ....
n 1
esrIenHmanplbUkedayEpk S n
1

1

1
1.2 2.3 3.4
...
1
n(n 1)
eKsmal;eXIjfa 1 1

k (k 1) k k 1
1

eK)an 1 1 1 1 1 1
S n (1 ) ( ) ( ) ... (
2 2 3 3 4 n n1
1
)
1
1
n1
eday lim S n lim (1
n n
1
n1
)1

dUcenHesrIGnnenHCaesrIrYmeTArk . 1

eroberogeday lwm pln TMBr 84


sIVtncMnYnBit nig esrI

]TahrN_12
eKmanesrIGnnmYy

1 1 1
2n 1 2n 1
3 1

5 3
....
n 1

cUrsikSaPaBrYm b rIknesrIxagelIenH .
dMeNaHRsay
sikSaPaBrYm b rIknesrI

1 1 1
2n 1 2n 1
3 1

5 3
....
n 1

esrIenHmanplbUkedayEpk
1 1 1
Sn ...
31 5 3 2n 1 2n 1
2k 1 2k 1
eKsmal;eXIjfa 1
2k 1 2k 1

2
eK)anplbUkedayEpk
3 1 5 3 2n 1 2n 1
Sn ....
2 2 2
2n 1 1

2
2n 1 1
eday lim S n lim
n n 2

dUcenHesrIGnnenHCaesrI rIkeTArk .

eroberogeday lwm pln TMBr 85


sIVtncMnYnBit nig esrI

]TahrN_ 13
eKmanesrIGnnmYy

n 1 2 3 n
n 1 2 3 4
....
n1
....
n1
cUrsikSaPaBrYm b rIknesrIxagelIenH .
dMeNaHRsay
sikSaPaBrYm b rIknesrI

n 1 2 3 n
n1 2 3 4
....

....
n1 n 1
eKmantYTI nsIVt
n an
n
n1
eday lim an lim
n
n
n n 1
10

n 1 2 3
dUcenH n1 2 3 4
....
n

.... CaesrIrIk .
n1 n 1

kMNtcMNaM

cMeBaHesrIGnn (a ) manplbUkedayEpk S
n n a1 a 2 ... an
n 1

ebI lim S S enaHvaCaesrIrYm .


n
n

ebI lim S enaHvaCaesrI rIk .


n
n

ebI lim a 0 enaHvaCaesrI rIk .


n
n

eroberogeday lwm pln TMBr 86


sIVtncMnYnBit nig esrI

]TahrN_ 14
eKmanesrIGnnmYy

n 1 2 3 n
[ ] .....
(n 1)! 2! 3! 4! (n 1)!
...
n1

cUrRsayfaesrIxagelIenHCaesrIbRgYm .
dMeNaHRsay
sikSaPaBrYmnesrI

n 1 2 3 n
[ (n 1)! ] 2! 3! 4! ..... (n 1)! ...
n1

plbUkedayEpkrbs;esrIenHKW S n
1 2 3

2! 3! 4!
.....
n
(n 1)!
(k 1) 1 1
eKBinit k
(k 1)!

(k 1)!

1
k! (k 1)!
eK)an 1 1 1
S n (1 ) ( ) .... (
2! 2! 3!
1 1
n! (n 1)!
)

1
1
(n 1)!
1
eday lim S n lim 1 1
n n
(n 1)!

dUcenH n 1 2 3 n
[ (n 1)! ] 2! 3! 4! ..... (n 1)! ...CaesrIrYm .
n1

eroberogeday lwm pln TMBr 87


sIVtncMnYnBit nig esrI

]TahrN_ 15
eKmanesrIGnnmYy

2n 1 1 3 5 2n 1
( 2n
)
2 22 23
.....
2n
....
n1

cUrRsayfaesrIxagelIenHCaesrIbRgYm .
dMeNaHRsay
RsayfaCaesrIbRgYm

2n 1 1 3 5 2n 1
( 2 n
)
2 2 2 2
3
.....
2n
....
n1
2n 1
plbUkedayEpkrbs;esrIenHKW 1 3 5
S n 2 3 ...
2 2 2 2n
BinitGnuKmn_ EdlRKb;
f (k ) ak b eKman
k IN *
2k 1 f (k ) f (k 1) 2k 1 ak b a(k 1) b
2 k

2 k

2
b
k 1
2 k

2 k

2k 1
b 4k 2 2ak 2b ak a b ak (b a )
eKTaj a 4 nig b a 2 b 2
ehtuenH 2k2 1 4k2 2 4k2 6
k k k 1

eK)an S ( 4k2 2 4k2 6 ) 3 2n2 3


n

n k k 1 n
k 1

eday lim S lim 3 2n2 3 3



n n
n n

dUcenHesrIGnnxagelICaesrIbRgYm .
eroberogeday lwm pln TMBr 88
sIVtncMnYnBit nig esrI

]TahrN_ 16
2 2 2 2 2
eKmanesrIGnnmYy ( n2 ) 12 22 32 ..... n2 ....
n 2 3 n
n1

cUrRsayfaesrIxagelIenHCaesrIbRgYm .
dMeNaHRsay
2 2 2 2
plbUkedayEpkrbs;esrIenHKW S 12 22 32 ... n2
n 2 3 n

BinitGnuKmn_ f (k ) ak bk c EdlRKb; k IN * eKman


2

k 2 f (k ) f (k 1)
k
2 k
2 2k 1
k 2 ak 2 bk c a(k 1)2 b(k 1) c
b 2k

2k

2k 1
b 2k 2 2ak 2 2bk 2c ak 2 2ak a bk b c

2k 2 ak 2 (b 2a )k c a b
eKTaj a 2 ehIy b 2a 0 b 4 nig c a b 6
ehtuenH k2 2k 24k 6 2k 28k 12
2 2 2

k k k 1

2k 2 4k 6 2k 2 8k 12 n 2 4n 6
eK)an Sn (
n

2 k

2 k 1
) 6
2n
k 1

n 2 4n 6
eday lim S n lim 6 n
6
n n
2
dUcenHesrIGnnxagelICaesrIbRgYm .

eroberogeday lwm pln TMBr 89


sIVtncMnYnBit nig esrI

]TahrN_ 17
cUrsikSaPaBrYm b rIknesrIGnnxageRkam

n3 13 23 33 n3
( n ) 2 22 23 .... 2n ....
n 1 2

dMeNaHRsay
sikSaPaBrYmbrIknesrIGnn

n3 13 23 33 n3
( 2n ) 2 22 23 .... 2n ....
n 1

13 23 33 n3
plbUkedayEpkrbs;esrIenHKW S n 2 3 ... n
2 2 2 2
BinitGnuKmn_f (k ) ak 3 bk 2 ck d EdlRKb;
k IN *
eKman
k 3 f (k ) f (k 1)
k
2 k
2 2k 1
k 3 ak 3 bk 2 ck d a(k 1)3 b(k 1)2 c(k 1) d

2k 2k 2k 1
2k 3 ak 3 (b 3a)k 2 (c 3a 2b )k d a b c
eKTaj a 2 ehIy b 3a 0 b 6 nig c 3a 2b 18
nig d a b c 26 ehIy f (n) 2n 6n 18n 26
3 2

eK)an S ( f 2(k ) f (2k 1) ) f (21) f (2n 1)


n

n k k 1 n1
k 1

eday f (1) 2 6 18 26 52

eroberogeday lwm pln TMBr 90


sIVtncMnYnBit nig esrI

nig f (n 1) 2(n 1) 3
6(n 1) 2 18(n 1) 26
2n 3 6n 2 6n 2 6n 2 12n 6 18n 18 26
2n 3 12n 2 36n 52
2n 3 12n 2 36n 52
eK)an S n 26
2n 1
n 3 6n 2 18n 26
b S n 26
2n
n 3 6n 2 18n 26
eday lim S n lim ( 26
n n 2 n
) 26
3 3 3 3
n3
dUcenH n 1 2 3
( 2n ) 2 22 23 .... 2n .... CaesrIbRgYm .
n 1

kMNtcMNaM
rebobKNnaplbUkrag S q .P (k )
n
k
n m
k 1

Edl P (k ) c c k c k .... c k CaBhuFadWeRkTI m


m 0 1 2
2
m
m

eKRtUvGnuvtn_dUcxageRkam
tagGnuKmn_BhuFa f (k ) a a k a k ... a k
m 0 1 2
2
m
m

Edlepgpat;smPaB q P (k ) q f (k ) q f (k 1)
k
m
k
m
k 1
m

rkGnuKmn_ f (k ) [eXIj
m

KNnaplbUk S q f (k ) q f (k 1) .
n
k k 1
n m m
k 1

eroberogeday lwm pln TMBr 91


sIVtncMnYnBit nig esrI

]TahrN_ 18
cMeBaHRKb; n IN * eKmanplbUk
5 7 9 2n 3
Sn ....
1.3.22 3.5.23 5.7.24 ( 2n 1)( 2n 1).2n 1
k> kMNt;cMnYnBit a nig b edIm,I[eK)ansmPaB
2n 3 a b

( 2n 1)( 2n 1).2n 1 ( 2n 1).2n ( 2n 1).2n 1
x> KNna S nig lim S .
n
n
n

dMeNaHRsay
k> kMNt;cMnYnBit a nig b edIm,I[eK)ansmPaB
2n 3 a b

( 2n 1)( 2n 1).2n 1 ( 2n 1).2n ( 2n 1).2n 1
eK)an 2n 3 2a(2n 1) b(2n 1)
2n 3 (4a 2b )n 2a b
4a 2b 2
eKTaj

naM[ a 1 , b 1
2a b 3
dUcenH a 1 nig b 1 .
x> KNna S nig lim S
n
n
n

tamsRmayxagelIenHcMeBaH a 1 nig b 1 eK)an


2n 3 1 1

( 2n 1)( 2n 1).2n 1 ( 2n 1).2n ( 2n 1).2n 1

eroberogeday lwm pln TMBr 92


sIVtncMnYnBit nig esrI

b (2p 12)(p2p 3 1)2


p1
1

1
( 2p 1)2p ( 2p 1)2p 1
n
2p 3
eK)an Sn p1
p 1 ( 2p 1)( 2p 1).2
n
1 1

p 1 ( 2p 1)2
p
( 2p 1)2p 1
1 1

2 ( 2n 1).2n 1
1
dUcenH Sn 1
2
1
( 2n 1).2n
.
1
ehIy 1
lim S n lim 1
1
n
.
n 2 n
( 2n 1).2 2

eroberogeday lwm pln TMBr 93


sIVtncMnYnBit nig esrI

]TahrN_ 19
cMeBaHRKb; n IN * eKmanplbUk
2 4 2n
Sn ..... n
3.5 5.9 ( 2 1)( 2n 1 1)
k> kMNt;cMnYnBit a nig b edIm,I[eK)ansmPaB
2k a b

( 2k 1)( 2k 1 1) 2k 1 2k 1 1
x> KNna S nig lim S .
n
n
n

dMeNaHRsay
k> kMNt;cMnYnBit a nig b
k
eK)an (2 1)(22 1) 2 a 1 2
k k 1 k
b
k 1
1
b 2 a(2 1) b(2 1)
k k 1 k

2k ( 2a b ).2k a b
2a b 1
eKTaj naM[ a 1 , b 1
a b 0
dUcenH a 1 , b 1 .
x> KNna S nig lim S
n
n
n

cMeBaH a 1 , b 1 eKmansmPaB

eroberogeday lwm pln TMBr 94


sIVtncMnYnBit nig esrI
2k 1 1

( 2k 1)( 2k 1 1) 2k 1 2k 1 1
n
2k
eK)an Sn k k 1
1)
k 1 ( 2 1)( 2
n
1 1
k k 1
k 1 2 1 2 1
1 1 1 1 1 1
( ) ( ) ... ( n n1 )
3 5 5 9 2 1 2 1
1 1
n1
3 2 1
dUcenH 1
Sn n 1
3 2 1
1
nig .
lim S n
n
1
3

eroberogeday lwm pln TMBr 95


sIVtncMnYnBit nig esrI

]TahrN_ 20
cMeBaHRKb; n IN * eKmansIVtcMnYnBit (a ) kMNt;eday n

1
a 3 nig a 1
1
0 n1 a n
2n 2n
3 3
k> KNna a CaGnuKmn_n n .
n

x> KNnalImIt lim a . n


n

dMeNaHRsay
k> KNna a CaGnuKmn_n n
n

eKman a ( 1 1 ) a 1
n1 n n n
32 32
1 1
an 1 1 (1 ) an (1 )
2n 2n
3 3
1
an 1 1 (1 ) (an 1)
2n
3
an 1 1 1
1 n
an 1 32
n 1
ak 1 1 n 1 1
eK)an a 1 1 k
k 0 k k 0 32
tamsmPaB ( )( )
2 2

2 2
b

eroberogeday lwm pln TMBr 96


sIVtncMnYnBit nig esrI

edayyk 1 nig 3 2k

1
1
2k 1
eK)an 1 1
2k
3
1
3 1 k
32
1
1
n 1
1 n 1 2k 1
ehtuenH 1 2k ( 3
1
)
k 0 3 k0 1
k
32
1
1 n
3 2 3 1
1 n
1 2 32
1
3
n 1
ak 1 1 an 1 an 1
ehIyeday a 1 a 1 2
k 0 k 0

an 1 3
eKTaj 2
(1 n )
2
1
an 4 n b
3
32 32
dUcenH an 4 n
3
32
x> KNnalImIt lim an
n

eK)an lim a lim (4 3 ) 4 eRBaH lim 3


n
n
n 2n n 2n
0
3 3
dUcenH lim a 4 .
n
n

eroberogeday lwm pln TMBr 97


sIVtncMnYnBit nig esrI

kRmglMhat;eRCIserIsmandMeNaHRsay
lMhat;TI1
eK[ a , b , c begIt)anCakMeNInsIVtnBVnmYy .
2 2 2

cUrbgajfabrimaN b 1 c , c 1 a , a 1 b kbegIt)anCa kMeNIn


nsIVtnBVnmYyEdr .
dMeNaHRsay
karbgaj
ebI a , b , c begIt)anCakMeNInsIVtnBVnmYyenaHeK)an
2 2 2

a c
2 2
b
2
(1) mFmnBVn
2
eKman b 1 c a 1 b (aab2)(bbcc) b a(a 2bc)bc ac (2)
2

yksmIkar (1) CMnYskg (2) eK)an

eroberogeday lwm pln TMBr 98


sIVtncMnYnBit nig esrI
1 1 a 2b c
2
b c a b a c2
(a c)b ac
2
1 1 2(a 2b c) 2(a 2b c)

b c a b (a c) 2 2(a c)b (a c)(a c 2b )
1 1 2

bc ab ac
tamTMnak;TMngenHbBaak;fabrimaN b 1 c , c 1 a , a 1 b
kbegIt)anCa kMeNInnsIVtnBVnmYyEdr .

lMhat;TI2
bgajfaebI a , b , c tagerogKaCatYTI p , q , r nsIVtnBVn
mYyenaHeK)ansmPaB (q r)a (r p)b (p q)c 0 .
dMeNaHRsay
karbgaj
ebI a , b , c tagerogKaCatYTI p , q , r nsIVtnBVn (u ) enaHeK)an
n

a u p u 1 (p 1)d , b u q u 1 (q 1)d , c u r u 1 (r 1)d

eKman (q r)a (r p)b (p q)c 0


b aq ar br bp cp cq 0
eroberogeday lwm pln TMBr 99
sIVtncMnYnBit nig esrI
c b (r q )d

b p( c b ) q( a c ) r ( b a ) 0 eday a c (p r )d
b a (q p )d

enaH pd(r q) qd(p r) rd(q p) 0
b d(pr pq pq qr qr pr ) 0
00 Bit

lMhat;TI3
eKmansmIkar x 3mx 2(6m 7)x 10m 16 0
3 2

kMNt; m edIm,I[smIkarenHmanbsbI x , x , x begIt)anCasIVtnBVn


1 2 3

dMeNaHRsay
kMNt; m
ebI x1 , x2 , x3 Cabsrbs;smIkarenaHtamRTwsIbTEvt
eKmanTMnak;TMng
x1 x 2 x 3 3m (1)

x1 x 2 x 2 x 3 x1 x 3 2( 6m 7 ) ( 2)
x x x 10m 16
1 2 3 ( 3)

ebI x 1 , x2 , x3 begIt)anCasIVtnBVnenaH x 1 x 3 2x 2 ( 4)

eroberogeday lwm pln TMBr 100


sIVtncMnYnBit nig esrI

yksmIkar (4) CYskg (1) eK)an 3x 3m b x m2 2

tamsmIkar (3) eK)an x x 10mx 16 10mm 16


1 3
2

tamsmIkar (2) eK)an (x x )x x x 12m 14


1 3 2 1 3

b 2m(m) 10mm 16 12m 14


b 2m 12m 24m 16 0
3 2

b 2(m 2) 0 naM[ m 2 .
3

dUcenH m 2 .
lMhat;TI4
eK[sIVtncMnYnBit (u ) kMnt;eday
n n 0

u 5 nig TMnak;TMngkMenIn u
2
0 u 6
n 1 n
5
Edl n 0 ; 1 ; 2 ; ... .
cUrKNna u CaGnuKmn_n n
n

dMeNaHRsay
KNna u CaGnuKmn_n n
n

eKman u 52 u 6 enaH 5u 2u 12 0
n1 n n1 n

smIkarsMKal; 5r 2r 12 0 naM[ r 4
eroberogeday lwm pln TMBr 101
sIVtncMnYnBit nig esrI

tagsIVtCMnYy v u r u 4
n n n

eK)an v u 4 52 u 6 4 52 u 4 52 v
n1 n1 n n n

naM[ v CasIVtFrNImaRtmanpleFobrYm q 52 nig v 1


n 0

n
2
eK)an v n v0 q
5
n
eday v n un 4
n

dUcenH un 4
2
5
.
lMhat;TI5
eK[sIVtncMnYnBit (a ) kMnt;eday n

a0 2

an 1 4an 1 4an 1 ; n IN
cUrKNna a CaGnuKmn_n n .
n

dMeNaHRsay
KNna a CaGnuKmn_n n
n

eKman a 4a 1 4a 1
n1 n n

KuNGgTaMgBIrnwg 4 eK)an

eroberogeday lwm pln TMBr 102


sIVtncMnYnBit nig esrI
4an 1 4(4an 1) 4 4an 1
4an 1 1 4(4an 1) 4 4an 1 1
4an 1 1 2 4an 1 1
2

4an 1 1 2 4an 1 1
4an 1 1 1 2 4an 1 1 i
tag v 4a 1 1
n n

tam (i ) eK)an v 2v naM[ (v ) CasIVtFrNImaRtman


n1 n n

pleFobrYm q 2 nigtY v 4a 1 1 3 1 4
0 0

tamrUbmn v v q 2
n 0
n n 2

eKTaj 4a 1 1 2
n
n 2

4an 1 2n 2 1
4a n 1 4 n 2 2 n 3 1
4n 2 2n 3
an 4n 1 2n 1
4

dUcenH an 4 n 1 2 n 1 .

eroberogeday lwm pln TMBr 103


sIVtncMnYnBit nig esrI

lMhat;TI6
eK[sIVtncMnYnBit u kMnt;eday
n


0
u sin
11
u 3u 4u 3 ; n IN
n1 n n

3n
edayeRbIGnumanrYmKNitviTacUrRsayfa . un sin
11
dMeNaHRsay
Rsayfa u sin 311
n

cMeBaH n 0 eK)an u sin 311 sin 11


0

0 Bit
]bmafasmPaBBitcMeBaH n k KW u sin 311 Bit
k

eyIgnwgRsayfavaBitdl; n k 1 KW u sin 3 11 Bit


k 1

k 1

eKman u 3u 4u eday u sin 311


k
3
k 1 k k k

eK)an u 3 sin 311 4 sin 311 sin 3 11 Bit


k k k 1
3
k 1

dUcenH u sin 311 .


n

eroberogeday lwm pln TMBr 104


sIVtncMnYnBit nig esrI

lMhat;TI7
eK[sIVtncMnYnBit u kMnt;eday n

u0 1 ; u1 2

3 1 n2

n 2 2 n 1 2 n 1
u u u ; n 0 ; 1 ; 2 ; ...
2
k>tag v u u n . bgajfa v CasIVtFrNImaRt .
n n 1 n n

x> KNna v nig u CaGnuKmn_n n .


n n

dMeNaHRsay
k> bgajfa v CasIVtFrNImaRt
n

eKman v u u n
n n 1 n

eK)an v u u n 1
n1 n 2 n1

eday u 32 u 12 u n 2 2
n 2 n1 n

enaH v 32 u 12 u n 2 2 u n 1
n1 n1 n n1

1 1 n
vn 1 un 1 un
2 2 2
1 1
vn 1 un 1 un n vn
2 2
dUcenH v CasIVtFrNImaRtmanpleFobrYm q 12 .
n

eroberogeday lwm pln TMBr 105


sIVtncMnYnBit nig esrI

x> KNna v nig u CaGnuKmn_n n


n n

tamrUbmn v v q eday v u u
n 0
n
0 1 0 1

dUcenH v 21 .
n n

eday v u u n
n n 1 n

eKTaj u u 21 n
n1 n n

n 1
1 n 1
uk 1 uk k
k 0 2
k
k 0

1
1 n ( n 1)n
un u0 2
1 2
1
2
1 ( n 1)n
un 1 2 1 n
2 2
1 ( n 1)n
un n 1 3
2 2
1 ( n 2)( n 3)
un n 1
2 2
( n 2)( n 3)
dUcenH 1
un n 1
2 2
.

eroberogeday lwm pln TMBr 106


sIVtncMnYnBit nig esrI

lMhat;TI8
eK[sIVtncMnYnBit u kMnt;cMeBaHRKb; n N eday
n

( 2n 1)( u 1)
u 1 nig u n1 . n

4u 6n 1
0
n

k>tag v 2uu n1 cMeBaH n 0 ; 1 ; 2 ; ... .


n
n

bgajfa v CasIVtFrNImaRt .
n

x> KNna v nig u CaGnuKmn_n n .


n n

dMeNaHRsay
k> bgajfa v CasIVtFrNImaRt
n

eKman v 2uu n1 enaH v u2u n11


n
n
n1
n1

n n1
( 2n 1)( un 1)
eday u n1
4un 6n 1
eK)an
2( 2n 1)( un 1)
1
4un 6n 1
vn 1
( 2n 1)( un 1)
n1
4un 6n 1

eroberogeday lwm pln TMBr 107


sIVtncMnYnBit nig esrI
2( 2n 1)un 4n 2 4un 6n 1
vn 1
( 2n 1)un 2n 1 4( n 1)un ( n 1)(6n 1)
(4n 2)un 2n 1 1 ( 2n 1)( 2un 1)
vn 1 .
(6n 3)un 6n 3n 3 ( 2n 1)( un n)
2

1 2u 1 1
vn 1 . n vn
3 un n 3
dUcenH vn CasIVtFrNImaRtmanpleFobrYmesI q
1
3
.
x> KNna v nig u CaGnuKmn_n n
n n

tamrUbmn v v q eday v 2uu 1 1


n 0
n
0
0

dUcenH v 31 .
n n

ehIyeday v 2uu n1 enaH u


n
n
n
nv n 1
2 vn
n

n 3n
dUcenH un
2 3n 1
.

eroberogeday lwm pln TMBr 108


sIVtncMnYnBit nig esrI

lMhat;TI9
eK[sIVtncMnYnBit (u ) kMnt;eday n n 0
2

u 2 nig u u 2n Edl n 0 ; 1 ; 2 ; ...


1 n 7
0 n1 n
2 2 2
eKtag v u (n 1) .
n n
2

k> bgajfa v CasIVtFrNImaRt .


n

x>KNna v nig u CaGnuKmn_n n .


n n

dMeNaHRsay
k> bgajfa v CasIVtFrNImaRt
n

eKman v u (n 1)
n n
2

eK)an v u (n 1) 1 u n 2n 2
n1 n1
2
n1
2

eday u 12 u n2 2n 32
n1 n

eK)an v 12 u n2 2n 32 n 2n 2
n1 n
2

vn 1 un n2 un ( n 2 1) vn
1 1 1 1 1
2 2 2 2 2
dUcenH CasIVtFrNImaRtmanpleFobrYm
vn q
1
2
.
x>KNna v nig u CaGnuKmn_n n
n n

eroberogeday lwm pln TMBr 109


sIVtncMnYnBit nig esrI

eday v CasIVtFrNImaRtmanpleFobrYm q 12 nigman


n

tYdMbUg v u 1 2 1 1
0 0

tamrUbmneK)an v v q n 0
n

dUcenH v 21 ehIy u v n 1 21 n 1
n n n n
2
n
2

lMhat;TI10
eK[sIVtncMnYnBit (u ) kMnt;eday n

2n 1
u
n
( n 1)( n 2n 2)
2 2
Edl n 0 ; 1 ; 2 ; ....
k> cUrsresr u Carag n A 1 n B2n 2 Edl A nig B
n 2 2

CaBIrcMnYnBitRtUvkMnt; .
x> KNnaplbUk S u u u u ... u
n
n

k 0
k 0 1 2 n

CaGnuKmn_n n rYcTajrklImItvakalNa n .
dMeNaHRsay
k> sresr u Carag n A 1 n B2n 2
n 2 2

eKman u (n 1)(2nn 12n 2)


n 2 2

eroberogeday lwm pln TMBr 110


sIVtncMnYnBit nig esrI
( 2n 1)
eK)an n A 1 n
2 2
B

2n 2 ( n2 1)( n 2 2n 2)
A( n 2 2n 2) B( n2 1) 2n 1
( A B )n2 2 An ( 2 A B ) 2n 1
smIkareRkayenHBitcMeBaHRKb;cMnYnKt; n 0 luHRtaEt

A B 0

2A 2 naM[ A 1 ; B 1
2A B 1

dUcenH u n 1 1 n 21n 2 .
n 2 2

x> KNnaplbUk S u u u u ... u


n
n

k 0
k 0 1 2 n

tamsRmayxagelIeKman u n 1 1 n 21n 2 n 2 2

eK)an S k 1 1 k 21k 2
n
n

k 0
2 2

1 1 1 1 1
1 ... 2 2
2 2 5 n 1 n 2n 2
1 n 2 2n 2 1
1 2
n 2n 2 n 2 2n 2
n 2 2n 1 ( n 1)2
2
n 2n 2 n 2 2n 2

eroberogeday lwm pln TMBr 111


sIVtncMnYnBit nig esrI
( n 1)2
dUcenH Sn 2
n 2n 2
.
rklImItn S kalNa n
n

eKman S 1 n 21n 2
n 2

dUcenH lim S 1 eRBaH lim n 21n 2 0 .


n
n
n 2

lMhat;TI11
eK[sIVtncMnYnBit (u ) kMnt;eday n n 0

u 2 nig TMnak;TMngkMeNIn u u 4u 2
2
0 n1 n n

Edl n 0 ; 1 ; 2 ; 3 ; ... .
k> eKtag v lnu 2 cMeBaHRKb; n 0 .
n n

bgajfa v CasIVtFrNImaRt rYcKNna v CaGnuKmn_n n


n n

x> cUrKNna u CaGnuKmn_n n .


n

K>KNna P u 3 CaGnuKmn_n n .
n
n

k 0
k

dMeNaHRsay
k> bgajfa v CasIVtFrNImaRt nig KNna
n n

eKman v lnu 2 enaH v lnu 2


n n n1 n1

eroberogeday lwm pln TMBr 112


sIVtncMnYnBit nig esrI

eday u n 1 un 4 un 2
2

eK)an v
n 1 ln un 4 un 4 ln un 2
2
2

vn 1 2 lnun 2 2vn
dUcenH v CasIVtFrNImaRtmanpleFobrYm q 2
n

nigtYdMbUg v ln 4 . tamrUbmn v v q 2 ln 4 .
0 n 0
n n

x> KNna u CaGnuKmn_n n


n

eKman v 2 ln 4 ln4 nig v lnu 2


n
n 2n
n n

eKTaj u 2 4 naM[ u 4 2
n
2n
n
2n

dUcenH u 4 2 .
n
2n

K>KNna P u 3 CaGnuKmn_n n
n
n

k 0
k

eKman u 3 4 2 3 4 1
k
2k 2k

tamsmPaB a 1 aa 11
2

k 1
42 1
edayCMnYs a 4 eK)an 2k
uk 3 k
42 1
42 k 1 1
eKTaj Pn 2 k
k 0 4
n

1

eroberogeday lwm pln TMBr 113


sIVtncMnYnBit nig esrI
2n1
4 1 4 1 4 1
2
4 4
1 8

2 4 ... 2 n
41 4 1 4 1 4 1


2n1
4 1 1 2n1
4 1
41 3
dUcenH P 13 4
n
2n1
1 .

lMhat;TI12
eK[sIVtncMnYnBit (u ) kMnt;eday n n 0

u 2 nig TMnak;TMngkMeNIn u 4u 6u 3u
3 2
0 n1 n n n

Edl n 0 ; 1 ; 2 ; 3 ; ... .
k> eKtag v ln2u 1 cMeBaHRKb; n 0 .
n n

bgajfa v CasIVtFrNImaRt rYcKNna v CaGnuKmn_n n


n n

x> cUrKNna u CaGnuKmn_n n .


n

dMeNaHRsay
k> bgajfa v CasIVtFrNImaRt
n

eKman v ln2u 1 enaH v ln2u 1


n n n1 n1

eday u 4u 6u 3u
n1 n
3
n
2
n

eroberogeday lwm pln TMBr 114


sIVtncMnYnBit nig esrI

eK)an v
n 1 ln 8 un 12un 6un 1
3 2

vn 1 ln2un 1
3

vn 1 3 ln2un 1
v n 1 3v n
TMnak;TMngenHbBaak;fa v CasIVtFrNImaRtmanpleFobrYm
n

esI q 3 nig tY v ln2u 1 ln 3 .


0 0

tamrUbmn v v q dUcenH v 3 ln 3 .
n 0
n
n
n

x> KNna u CaGnuKmn_n n


n

eKman v ln2u 1
n n

eday v 3 ln 3 ln3
n
n 3n

3n

eKTaj 2u 1 3 naM[ u 1 23
n
3n
n

3n
1 3
dUcenH un
2
.

eroberogeday lwm pln TMBr 115


sIVtncMnYnBit nig esrI

lMhat;TI13
eK[sIVtncMnYnBit (u ) kMnt;eday n n 0

u 3
2

u 2 nig TMnak;TMngkMeNIn u n1
n

2u 2
0
n

Edl n 0 ; 1 ; 2 ; 3 ; ... .
k> cUrRsayfa u 1 CanicRKb; n 0 .
n

x> eKtag v ln uu 13 cMeBaHRKb; n 0 .


n
n

bgajfa v CasIVtFrNImaRt rYcKNna v CaGnuKmn_n n


n n

x> cUrKNna u CaGnuKmn_n n rYcTajrk lim u .


n
n
n

dMeNaHRsay
k>Rsayfa u 1 CanicRKb; n 0
n

eKman u 2 1 Bit
0

u 3 7
2

u 0
1 Bit
2u 2 6
1
0

]bmafa vaBitcMeBaH n k KW u 1 Bit k

eyIgnwgRsayfa vaBitcMeBaH n k 1 KW u 1 k 1

eroberogeday lwm pln TMBr 116


sIVtncMnYnBit nig esrI

uk 3 ( uk 1)2
2

eKman uk 1 1
2uk 2
1
2uk 2
eday u 1 enaH u 1 0 naM[ u 1
k k 1 K 1

dUcenH u 1 CanicRKb; n 0
n

x>bgajfa v CasIVtFrNImaRt rYcKNna v CaGnuKmn_n n


n n

man v ln uu 13 cMeBaHRKb; n 0 .
n
n

u 1 u 3
2

eK)an vn 1 ln n 1

eday un 1 n
2un 2
n1
u 3
un 2 3
1 2
2u 2
ln un 1
enaH vn 1 ln 2n u 3
un 3 n
3
2un 2
u 1
vn 1 2 ln n 2vn
un 3
dUcenH (v ) CasIVtFrNImaRtmanpleFobrYm q 2 .
n

tamrUbmn v v q eday v ln 15
n 0
n
0

dUcenH v 2 ln0,2 .
n
n

K> KNna u CaGnuKmn_n n


n

eroberogeday lwm pln TMBr 117


sIVtncMnYnBit nig esrI
un 1
eKman v ln
un 3
n

1 1
eday vn 2 n
ln

ln 2n
5 5
un 1
eKTaj un 3 5
2n
1

b 5 u 5 u 3
2n
n
2n
n

2n

dUcenH u 5 3 .
n 2n
5 1
KNnalImIt
2n

eyIg)an lim u lim 5 3


n n
n
52 1
n

3
1 2n
lim 5 1
n 1
1 2n
5
dUcenH lim un 1
n
eRBaH 1
lim 2 n 0
n
.
5

eroberogeday lwm pln TMBr 118


sIVtncMnYnBit nig esrI

lMhat;TI14
eK[sIVtncMnYnBit (u ) kMnt;eday n n 0

u 6u 2
3

u 2 nig TMnak;TMngkMeNIn u n1
n n

3( u u 1)
0 2
n n

Edl n 0 ; 1 ; 2 ; 3 ; ... .
k> cUrRsayfa u 1 CanicRKb; n 0 .
n

x> eKtag v ln uu 12 cMeBaHRKb; n 0 .


n
n

bgajfa v CasIVtFrNImaRt rYcKNna v CaGnuKmn_n n


n n

x> cUrKNna u CaGnuKmn_n n rYcTajrk lim u .


n
n
n

dMeNaHRsay
k>Rsayfa u 1 n

eKman u 2 1 Bit
0

u 6u 2 22
3

u 0
1 Bit
0

3( u u 1) 21
1 2
0 0

]bmafa vaBitcMeBaH n k KW u 1 Bit k

eyIgnwgRsayfa vaBitcMeBaH n k 1 KW u 1 k 1

eroberogeday lwm pln TMBr 119


sIVtncMnYnBit nig esrI

u 6uk 2
3
uk 1 3

eKman uk 1 1 k 2
3( uk uk 1)
1
3( uk uk 1)
2 0

naM[ u 1 Bit eRBaH u 1


k 1 k

dUcenH u 1 CanicRKb; n 0 .
n

x>bgajfa v CasIVtFrNImaRt rYcKNna v CaGnuKmn_n n


n n

man v ln uu 12 cMeBaHRKb; n 0
n
n

n
un 1 1 un 6un 2
3

eK)an vn 1 ln

eday
un 1
3( un un 1)
2
n1
u 2
un 3 6un 2
1 3
3( u un 1)
2
un 1
vn 1 ln 3n ln

u 6un 2
n 2 2 un 2
3( un un 1)
u 1
vn 1 3 ln n 3 vn
un 2
dUcenH (v ) CasIVtFrNImaRtmanpleFobrYm q 3 .
n

tamrUbmn v v q eday v ln 14
n 0
n
0

dUcenH v 3 ln 14 .
n
n

x> KNna u CaGnuKmn_n n rYcTajrk lim u


n
n
n

eroberogeday lwm pln TMBr 120


sIVtncMnYnBit nig esrI
un 1
eKman v ln
un 2
n

1 1
eday vn 3 n
ln

ln 3n
4 4
un 1
eK)an un 2 4
3n
1

naM[ 4 u 4
3n
n
3n
un 2
3n
2
dUcenH u 4n 3n
ehIy lim u n 1 .
4 1 n

eroberogeday lwm pln TMBr 121


sIVtncMnYnBit nig esrI

lMhat;TI15
eK[sIVtncMnYnBit (u ) nig v kMnt;eday
n n 0 n n 0


u0 4 ; v0 1

13un 3vn
n1
u
5
12vn 2un
v
n 1
5
Edl n 0 ; 1 ; 2 ; 3 ; ... .
k> eKtag x u v nig y 2u 3v cMeBaHRKb; n 0
n n n n n n

bgajfa x nig y CasIVtFrNImaRt.


n n

KNna x nig y CaGnuKmn_n n .


n n

x> cUrKNna u nig v CaGnuKmn_n n .


n n

dMeNaHRsay
k>bgajfa x nig y CasIVtFrNImaRt
n n

eKman x u v naM[ x u v
n n n n1 n1 n1

eday u 13u 5 3v nig v 12v 5 2u


n1
n n
n1
n n

eK)an x 13u 5 3v 12v 5 2u


n1
n n n n

eroberogeday lwm pln TMBr 122


sIVtncMnYnBit nig esrI

xn 1 3( un vn ) 3 xn
dUcenH x CasIVtFrNImaRtmanpleFobrYm q 3
n

nigtY x u v 4 1 5 dUcenH x 5 3 .
0 0 0 n
n

ma:geTot y 2u 3v naM[ y 2u 3v
n n n n1 n1 n1

13un 3vn 12vn 2un


yn 1 2 3
5 5
26un 6vn 36vn 6un
yn 1
5
yn 1 4un 6vn 2 yn
dUcenH y CasIVtFrNImaRtmanpleFobrYm q 2
n

nigtY y 2u 3v 4 3 1 dUcenH y 2 .
0 0 0 n
n

x> KNna u nig v CaGnuKmn_n n


n n

u v 5 3 n

tamsRmayxagelIeKTaj 2u 3v 2 n n
n
n n

bnab;BIedaHRsayRbBnenHeKTTYl)an
5 3n 1 2n 2n 10 3n
un
5
; vn
5
.

eroberogeday lwm pln TMBr 123


sIVtncMnYnBit nig esrI

lMhat;TI16
eK[sIVtncMnYnBit (u ) nig v kMnt;eday
n n 0 n n 0


u0 3 ; v0 1

8 2
n1
u un vn
3 3
1 7

n 1 3 n 3 vn
v u

Edl n 0 ; 1 ; 2 ; 3 ; ... .
k> cUrkMnt;RKb;KU r ; edIm,I[ u v r u
n1 n1 n vn
cMeBaHRKb; n 0 .
x> KNna u nig v CaGnuKmn_n n .
n n

dMeNaHRsay
k> kMnt;RKb;KU r ;
eKman u v r u v *
n1 n1 n n

eday u 83 u 23 v nig v 13 u 73 v
n1 n n n1 n n

eK)ansmIkar

eroberogeday lwm pln TMBr 124


sIVtncMnYnBit nig esrI
8 2 1 7
un vn un vn r un vn
3 3 3 3
8 1 2 7
n vn r un r vn
u
3 3 3 3
smIkarenHepgpat;RKb; n 0 luHRtaEt
8 1
r 1
3 3

2 7

r 2
3 3
yksmIkar 1 CYskg 2 eK)an
8 1
b 2 7 8
2 7 2

3 3 3 3
b 2 0 eKTaj 1 2
2
1 2

cMeBaH 1 enaH r 3
cMeBaH 2 enaH r 2
dUcenH r ; { 3 ; 1 ; 2 ; 2 }
x> KNna u nig v CaGnuKmn_n n
n n

edayyk r ; { 3 ; 1 ; 2 ; 2 } CYskg *
u v 3( u v ) i
eK)an u 2v 2(u 2v ) ii
n1 n1 n n

n1 n1 n n

eroberogeday lwm pln TMBr 125


sIVtncMnYnBit nig esrI
xn un vn
tagsIVt
yn un 2vn
xn 1 3 xn
tam nig eK)an
i ii
yn 1 2 yn
TMnak;TMngenHbBaak;fa x nig y CasIVtFrNImaRt
n n

manpleFobrYmerogKa q 3 nig q 2 nigtYdMbUg


1 2

x u v 4 nig y u 2v 1
0 0 0 0 0 0

tamrUbmneK)an x 4 3 nig y 2
n
n
n
n

u v 4 3 n

eKTaj)anRbBn u 2v 2 n n
n
n n

bnab;BIedaHRsayRbBnenHeKTTYl)an
8 3 2
n n
4 3 2 n n

u
n
3
nig v
3
. n

eroberogeday lwm pln TMBr 126


sIVtncMnYnBit nig esrI

lMhat;TI17
eK[sIVtncMnYnBit (u ) nig v kMnt;eday
n n 0 n n 0

u0 4 ; v0 2

un 1 un 2vn
2 2



2
n1
v 2 u v
n n v n

Edl n 0 ; 1 ; 2 ; 3 ; ... .
k> cUrRsayfa u v CaniccMeBaHRKb; n 0 .
n n

x> bgajfaeKGackMnt;cMnYnBit r edIm,I[)an


un 1 r vn 1 un r vn
2

K> cUrKNna u nig v CaGuKmn_n n .


n n

dMeNaHRsay
k> Rsayfa u v CaniccMeBaHRKb; n 0
n n

eyIgman u 4 v 2 Bit
0 0

]bmafavaBitcMeBaH n k KW u v Bit k k

eyIgnwgRsayfavaBitcMeBaH n k 1
KW u v Bit
k 1 k 1

eKman u v u 2v 2u v v
k 1 k 1 k
2
k
2
k k k
2

eroberogeday lwm pln TMBr 127


sIVtncMnYnBit nig esrI

uk 1 vk 1 uk vk 0
2
eRBaH u k vk
eKTaj u v Bit k 1 k 1

dUcenH u v CaniccMeBaHRKb; n 0 .
n n

x> kMnt;cMnYnBit r
eKman u r v u r v *
n1 n1 n n
2

eday u u 2v nig v 2u v v
n1 n
2
n
2
n1 n n n
2

eK)an u 2v r 2u v v u r v
n
2
n
2
n n n
2
n n
2

un 2r unvn ( 2 r )vn un 2runvn r 2vn


2 2 2 2

eKTaj 2 r r b r r 2 0 2 2

dUcenH r 1 b r 2 .
1 2

K> KNna u nig v CaGuKmn_n n


n n

yktm r 1 ; r 2 CMnYskg * eK)an


un 1 vn 1 un vn
2


un 1 2vn 1 un 2vn
2

ln( un 1 vn 1 ) 2 ln un vn
i

ln( un 1 2vn 1 ) 2 ln un 2vn ii

tag xn ln( un vn ) nig vn lnun 2vn

eroberogeday lwm pln TMBr 128


sIVtncMnYnBit nig esrI

tam (i ) & ii eK)an x 2 x nig y 2 y n1 n n1 n

naM[ x nig y CasIVtFrNImaRtmanersugerogKa q


n n 1 2
nig q 2 nigtY x ln 2 nig y ln 8
2 0 0

eK)an x 2 ln 2 nig y 2 ln 8
n
n
n
n

eday x ln( u v ) nig v lnu 2v


n n n n n n

eKTaj lnlnuu v2v 22lnln28


n
n n
n
n n

un vn 22 n
naM[
un 2vn 82
n

bnab;BIedaHRsayRbBnenHeKTTYl)an
2n 1 2n 2n 2n
8 8 2
u
n
2
3
ni g v
3
n

eroberogeday lwm pln TMBr 129


sIVtncMnYnBit nig esrI

lMhat;TI18
eK[sIVtncMnYnBit (u ) nig v kMnt;eday
n n 0 n n 0

u 4 u u 6u v 6v 3 2 3

v 2
0
nig n1 n n n n

0 vn 1 3un 2vn 9unvn 2 7vn 3


Edl n 0 ; 1 ; 2 ; 3 ; ... .
k> cMeBaHRKb; n 0 cUrRsayfa u v 0 nig u n n n 2vn 0
x> bgajfaeKGackMnt;cMnYnBit r edIm,I[)an
un 1 r vn 1 un r vn
3

K> cUrKNna u nig v CaGuKmn_n n .


n n

dMeNaHRsay
k> cMeBaHRKb; n 0 Rsayfa u v 0 n n

eKman u v 4 2 6 0 Bit
0 0

]bmafavaBitdl; n k KW u v 0 Bit k k

eyIgnwgRsayfavaBitdl; n k 1 KW u v k 1 k 1 0 Bit
eKman u v u 3u v 3u v v
k 1 k 1 n
3
n
2
n n n
2
n
3

u v u v 0 Bit
3
k 1 k 1 k k

dUcenH u v 0 cMeBaHRKb; n 0 .
n n

eroberogeday lwm pln TMBr 130


sIVtncMnYnBit nig esrI

cMeBaHRKb; n 0 Rsayfa u 2v 0 n n

eKman u 2v 4 4 8 0 Bit
0 0

]bmafavaBitdl; n k KW u 2v 0 Bit k k

eyIgnwgRsayfavaBitdl; n k 1 KW u 2v 0 Bit k 1 k 1

eKman u 2v u 6u v 12u v 8v
k 1 k 1 n
3
n
2
n n n
2
n
3

u 2v u 2v 0 Bit
3
k 1 k 1 k k

dUcenH u 2v 0 cMeBaHRKb; n 0 .
n n

x> kMnt;cMnYnBit r edIm,I[)an


un 1 r vn 1 un r vn *
3

un 1 un 3 6unvn 2 6vn 3
eday eK)an
vn 1 3un 2vn 9unvn 2 7vn 3

un 1 r vn 1 un 3run vn ( 9r 6)un vn ( 7 r 6)vn i


3 2 2 3

ehIy u r v u 3ru v 3r u v r v
n n
3
n
3
n
2
n
2
n n
2 3
n
3
ii
edayeRbobeFobTMnak;TMng i nig ii eKTaj)an
3r 2 9r 6 r 2 3r 2 0
3 3
r 7r 6 r 7r 6 0
dUcenH r1 1 ; r2 2 .

eroberogeday lwm pln TMBr 131


sIVtncMnYnBit nig esrI

K> KNna u nig v CaGuKmn_n n


n n

yktm r 1 ; r 2 CYskg * eK)an


1 2

un 1 vn 1 ( un vn )3

n1
u 2 v n1 un 2v n
3

lnun 1 vn 1 3 lnun vn

lnun 1 2vn 1 3 lnun 2vn
tamTMnak;TMngenHbBaak;fa { ln( u v )} nig {ln(u 2v )} n n n n

suTEtCasIVtFrNImaRtEdlmanpleFobrYm q 3 dUcKa .
eK)an lnlnuu v2v 33lnlnuuv 2v)3 3ln 6ln 8
n n
n n 0 0
n n
n n 0 0

un vn 6 3n

eKTaj
un 2vn 83
n

bnab;BIedaHRsayRbBnenHeKTTYl)an
u 2 6 8 nig v 8 6
n
3n 3n
n
3n 3n
.

eroberogeday lwm pln TMBr 132


sIVtncMnYnBit nig esrI

lMhat;TI19
eK[sIVt S 1 n1 (n 11) Edl n IN * .
n 2 2

k-cMeBaHRKb; n IN * cUrbgajfa S 1 n1 n 1 1 .
n

x-KNnaplbUk
1 1 1 1 1 1
n 1 1 ..... 1
12 22 22 32 n 2 (n 1)2
dMeNaHRsay
k-bgajfa S 1 n1 n 1 1
n

eyIgman S 1 n1 (n 11)
n 2 2
cMeBaHRKb; n IN *
n 2 (n 1) 2 (n 1) 2 n 2

n 2 (n 1) 2
n 2 (n 1) 2 n 2 2n 1 n 2

n 2 (n 1) 2
n 2 (n 1) 2 2n(n 1) 1 [n(n 1) 1]2

n (n 1)
2 2
n 2 (n 1) 2
n(n 1) 1 1 1 1
1 1
n(n 1) n(n 1) n n1

dUcenH 1
Sn 1
n n1
1
.

eroberogeday lwm pln TMBr 133


sIVtncMnYnBit nig esrI

x-KNnaplbUk
eK)an
1 1 1 1 1 1 n
n 1 2 2 1 2 2 ..... 1 2 (S k )
1 2 2 3 n (n 1) 2
k 1
n
1 1 1 (n 1)2 1 n(n 2)
1 n 1
k 1 k k 1 n1 n(n 1) n1
nn 2
dUcenH n
n1
.

eroberogeday lwm pln TMBr 134


sIVtncMnYnBit nig esrI

lMhat;TI20
eK[sIVtncMnYnBit (u ) kMnt;eday
n
n

u
n
2
( 2 1)( 2 1)
n n1
Edl n 0 ; 1 ; 2 ; ....
k> cUrsresr u Carag 2 A 1 2 B 1 Edl A nig B
n n n1

CaBIrcMnYnBitRtUvkMnt; .
x> KNnaplbUk S u u u u ... u
n
n

k 0
k 0 1 2 n

CaGnuKmn_n n rYcTajrklImItvakalNa n .
dMeNaHRsay
k> sresr u Carag 2 A 1 2 B 1
n n n1

eKman u (2 1)(22 1)
n n n1

( 2 1)2n 2n 1 2n
n
( 2 1)( 2n 1 1) ( 2n 1)( 2n 1 1)
( 2n 1 1) ( 2n 1) 1 1

( 2n 1)( 2n 1 1) 2n 1 2n 1 1
dUcenH un n
1
2 1 2 1
n1
1
ehIy
A 1 ; B 1 .
x> KNnaplbUk S n uk u0 u1 u2 ... un
n

k 0

eroberogeday lwm pln TMBr 135


sIVtncMnYnBit nig esrI

eKman u n
1

1
2n 1 2n 1 1
.
eK)an
n
1 1
Sn k k 1
k 0 2 1 2 1
1 1 1 1 1 1
... n n1
2 3 3 5 2 1 2 1
1 1
n1
2 2 1
dUcenH 1
Sn n 1
2 2 1
1
.
KNnalImIt lim S n
n

eyIg)an lim S lim 12 2


n
n
n n1
1

1
eday lim 2
n n1
1

1
0

dUcenH lim S
n
n
1
2
.

eroberogeday lwm pln TMBr 136


sIVtncMnYnBit nig esrI

lMhat;TI21
eK[sIVtncMnYnBit (u ) kMnt;eday n

2( n 1) 2

u
n
n n 1
4
Edl n 1 ; 2 ; 3 ; ....
2

k> cUrsresr u Carag n An 1 n Bn 1 Edl A nig B


n 2 2

CaBIrcMnYnBitRtUvkMnt; .
x> KNnaplbUk
Sn 3k 1 uk u1 3u2 32 u3 ... 3n 1 un
n

k 1

CaGnuKmn_n n .
dMeNaHRsay
k>sresr u Carag n An 1 n Bn 1
n 2 2

eKman u n2(n n 1) 1
2

n 4 2

eK)an n An 1 n Bn 1 n2(n n 1) 1
2

2 2 4 2

A( n 2 n 1) B( n2 n 1) 2( n 1)2
( A B )n 2 ( A B )n ( A B ) 2 n 2 4 n 2

eroberogeday lwm pln TMBr 137


sIVtncMnYnBit nig esrI
A B 2
eKTaj)an

eKTaj
A 3 ; B 1
A B 4
dUcenH un 2
3

1
n n 1 n2 n 1
ehIy
A 3 ; B 1 .
x> KNnaplbUk
Sn 3k 1 uk u1 3u2 32 u3 ... 3n 1 un
n

k 1

eKman u n
3

1
n2 n 1 n2 n 1
eK)an
n 3k 3k 1
Sn 2 2
k 1 k k 1 k k 1
3 9 3 27 9 81 27
1 ....
3 7 3 13 7 21 13
3n 3n 1
2 2
n n 1 n n 1
3n 3n n2 n 1
1 2
n n1 n2 n 1
3n n 2 n 1
dUcenH Sn
n2 n 1
.

eroberogeday lwm pln TMBr 138


sIVtncMnYnBit nig esrI

lMhat;TI22
eK[sIVtncMnYnBit (u ) kMnt;eday
n

u
n 2n
1
Edl n 0 ; 1 ; 2 ; ....
2 1
k> cUrsresr u Carag A B Edl A nig B
n 2n 2n1
2 1 2 1
CaBIrcMnYnBitRtUvkMnt; .
x> KNnaplbUk
Sn 2k uk u0 2u1 4u2 ... 2n un
n

k 0

CaGnuKmn_n n rYcTajrklImItvakalNa n .
dMeNaHRsay
k>sresr u Carag A B
n 2n 2n1
2 1 2 1
eKman u 1 Edl n 0 ; 1 ; 2 ; ....
n 2n
2 1
eK)an A B 1
2n 2n1 2n
2 1 2 1 2 1
ebI n 0 enaH A B3 13 (1)
ebI n 1 enaH A3 15B 15 (2)

eroberogeday lwm pln TMBr 139


sIVtncMnYnBit nig esrI

tam (1) nig (2) eK)anRbBnsmIkar


B 1
A
3 A B 1

3 3
smmUl
A B 1 5 A B 3
3 15 5
bnab;BIedaHRsayeK)an A 1 ; B 2 .
dUcenH u 1 2 .
n 2n 2n1
2 1 2 1
x> KNnaplbUk
Sn 2k uk u0 2u1 4u2 ... 2n un
n

k 0

eKman un
1
n
2
n1
22 1 22 1
n 2k 2k 1 2n 1
eK)an Sn 2 k 2k 1 1 2n1
k 0 2 1 2 1 2 1
n1
22 2n 1 1
dUcenH Sn . 2n1
2 1
KNnalImIt
n1 n1

eday S 1 2 Edl lim 2


n 2n1 2n1
0
2 1 2 n
1
dUcenH lim S 1 .
n
n

eroberogeday lwm pln TMBr 140


sIVtncMnYnBit nig esrI

lMhat;TI23
eK[sIVtncMnYnBit (u ) kMnt;eday
n

2 2
n 2n 1
un 3 2
n ( n 1)2
Edl n 1 ; 2 ; 3 ; ....
k> cUrsresr u Carag nA (n B1) Edl A nig B
n 2 2

CaBIrcMnYnBitRtUvkMnt; .
x> KNnaplbUk
n
u 1 1 1 1
Sn kk u1 2 u2 3 u3 ... n un
k 1 3 3 3 3 3
CaGnuKmn_n n rYcTajrklImItvakalNa n .
dMeNaHRsay
k> sresr u Carag nA (n B1)
n 2 2

2 2
n 2n 1
eK)an A
2

B
n ( n 1) 2
3 2
n ( n 1)2
2
A( n 1)2 Bn 2 n2 2n 1
3
( 3 A 3 B )n2 6 An 3 A 2n2 6n 3

eroberogeday lwm pln TMBr 141


sIVtncMnYnBit nig esrI
3 A 3B 2

eKTaj 6 A 6 naM[ A 1 ; B
1
3
3 A 3

dUcenH 1
un 2
1
n 3( n 1)2
.
x> KNnaplbUk
n
u 1 1 1 1
Sn kk u1 2 u2 3 u3 ... n un
k 1 3 3 3 3 3
eday un 2
1
n 3( n 1)2
1

n 1 1
eK)an Sn 2 . k
k 1 k
1 1
.
3 ( k 1)2 3k 1
1 1

3 ( n 1)2 3n 1
dUcenH Sn
1 1
.
3 ( n 1)2 3n 1
eday lim
1
n ( n 1) 2 .3 n 1
0

dUcenH lim Sn
n
. 1
3

eroberogeday lwm pln TMBr 142


sIVtncMnYnBit nig esrI

lMhat;TI24
eK[sIVtncMnYnBit (u ) kMnt;eday
n

( n 1)( 3n 1)
u
n
4 n
Edl n 1 ; 2 ; 3 ; ....
B( n 1) 2 2

k> cUrsresr u Carag An n


4

4
Edl n
A nig B n1

CaBIrcMnYnBitRtUvkMnt; .
x> KNnaplbUk S u u u u ... u
n
k 0
n

k 0 1 2 n

CaGnuKmn_n n rYcTajrklImItvakalNa n .
dMeNaHRsay
B( n 1) 2 2

k>sresr u Carag An
n
4

4 n n1

2
B( n 1) ( n 1)( 3n 1) 2

eK)an An 4 n

4
n1
4 n

4 An 2 B( n 1)2 4( 3n 2 2n 1)
(4 A B )n 2 2 Bn B 12n 2 8n 4
4 A B 12

eKTaj 2 B 8 naM[
A 4 ; B 4 .
B 4

4n 2 4( n 1)2
dUcenH un n
4 4n 1
.

eroberogeday lwm pln TMBr 143


sIVtncMnYnBit nig esrI

x> KNnaplbUk S u u u u ... u


n
n

k 0
k 0 1 2 n

eKman u 44n 4(n4 1) 4n (n 4 1)


2 2 2 2

n n n1 n 1 n

eK)an S 4k (k 4 1)
n 2 2

n k 1

k
k 0

4 4 9 9 16
0 1 1 ...
4 4 16 16 64
n 2 ( n 1)2
.... n 1

n
4 4
( n 1)2

4n
( n 1)2
dUcenH Sn
4n
ehIy lim S
n
n 0 .

eroberogeday lwm pln TMBr 144


sIVtncMnYnBit nig esrI

lMhat;TI25
eK[sIVtncMnYnBit (u ) kMnt;eday n

u log
n
n 2 Edl n 1 ; 2 ; 3 ; .... .
( n 1)

k>cUrbgajfa u CasIVtcuHCaniccMeBaHRKb; n 1
n

x>KNnaplbUk S ln u ln u ln u .... ln u
n 1 2 3 n

CaGnuKmn_n n
dMeNaHRsay
k>bgajfa u CasIVtcuHCaniccMeBaHRKb; n 1
n

eKman u log n 2 nig u log n 3


n ( n 1) n1 ( n 2)

cMeBaHRKb; n 1 eKman (n 2) (n 2) 1 2 2

b (n 2) (n 2 1)(n 2 1) (n 1)(n 3)
2

eK)an log n 2 log (n 1)(n 3)


( n 2)
2
( n 2)

b 2 log n 1 log n 3 1
( n 2) (n 2)

tamvismPaB AM GM eKman
log ( n 2 ) n 1 log ( n 1) n 2 2 log ( n 2 ) n 1 log ( n 1) n 2

b log ( n 2 ) n 1 log ( n 1 ) n 2 2 2

eroberogeday lwm pln TMBr 145


sIVtncMnYnBit nig esrI

tam 1 nig 2 eKTaj)an


log ( n 2 ) n 1 log ( n 1 ) n 2 log ( n 2 ) n 1 log ( n 2 ) n 3
log ( n 1 ) n 2 log ( n 2 ) n 3
un un 1
dUcenH u CasIVtcuHCaniccMeBaHRKb; n 1 .
n

x>KNnaplbUk S ln u ln u ln u .... ln u
n 1 2 3 n

n 2)
eKman u log n 2 ln(
n ( n 1)
ln( n 1)
eK)an S ln u ln u
n
n

k 1
k
n

k 1
k

eday u lnlnnn 12
n

k
n

k 1
k 1
ln 3 ln 4 ln 5 ln( n 2)
. . .....
ln 2 ln 3 ln 4 ln( n 1)
ln( n 2)
log 2 ( n 2)
ln 2
dUcenH S n lnlog 2 n 2 .

eroberogeday lwm pln TMBr 146


sIVtncMnYnBit nig esrI

lMhat;TI26
f (1) 1

cMeBaHRKb; n 1 eKman f ( n) an bn cn d
3 2

f ( n 1) f ( n) n 2

k>KNnatmelxn a ; b ; c ; d EdlbMeBjlkxNxagelI.
x>Tajrktm S 1 2 3 .... n .
n
2 2 2 2

dMeNaHRsay
k>KNnatmelxn a ; b ; c ; d
f (1) 1

eKman f ( n) an bn cn d
3 2

f ( n 1) f ( n) n 2

eday f (n) an bn cn d
3 2

enaH f (n 1) a(n 1) b(n 1) c(n 1) d


3 2

eK)an f (n 1) f (n) 3an (3a 2b)n a b c


2

eKTaj 3an (3a 2b)n a b c n


2 2

naM[ a 13 ; b 12 ; c 16
ehIy f (1) a b c d 1 enaH d 1

eroberogeday lwm pln TMBr 147


sIVtncMnYnBit nig esrI

dUcenH a 13 ; b 12 ; c 16 ; d 1 .
x>Tajrktm S 1 2 3 .... n
n
2 2 2 2

tamsRmayxagelIeK)an
1 1 1
f ( n) n3 n2 n 1
3 2 6
eday f (n 1) f (n) n 2

eK)an S k f (k 1) f (k )
n
n

k 1
2
n

k 1

f ( n 1) f (1)
eday 1 1
f ( n) n3 n2 n 1
3 2
1
6
enaH 1 1 1
f (1) 1 1
3 2 6
ehIy 1 1 1
f ( n 1) ( n 1)3 ( n 1)2 ( n 1) 1
3 2 6
1 1 1
n3 n2 n 1
3 2 6
2n 3 3n 2 n n( n 1)( 2n 1)
1 1
6 6
n( n 1)( 2n 1)
dUcenH S n k 2 .
n

k 1 6

eroberogeday lwm pln TMBr 148


sIVtncMnYnBit nig esrI

lMhat;TI27
1
1
eK[ 2 3 2 1 1
1 3 2 3
1 3
2
1 1
1 3 1 3
2 3 2 1 1

1 1 3 3 4
1 3 1 3
2 3
BI]TahrN_xagelIcUrrkrUbmnTUeTAnigRsayrUbmnenaHpg .
dMeNaHRsay
rkrUbmnTUeTA
1
1
eKman 1 23 1 2 1 3
3
2
1 3
2
1 1
1 3 1
2 33 2 1 1

1 1 3 3 4
1 3 1 3
2 3
Tam]TahrN_xagelIenHeyIg)anrUbmnTUeTAdUcxageRkam

eroberogeday lwm pln TMBr 149


sIVtncMnYnBit nig esrI
1 1 1
1 1 1
23 3 3 ...... n 3 2 1 1
.
1 1 1 3 n (n 1)
1 3
1 1 3
2 33 n
karRsaybBaak;rUbmn
1 1 1
1
1
3 3
1 3 n 1
1 3

tag Pn 2 3 ...... n k
1 1 1 k 2 1
1 3 1 3 1 3 1
2 3 n k3
eyIgman
1
1
k3 k 1 k 1 k k 1
3 2

1 k3 1 k 1 k2 k 1
1 3
k
k 1 k k (k 1) 1

k k 1 (k 1)k 1

eK)an Pn k k k 1 k(k(k 1)1k) 11


n k 1

k 2
n k 1
n k n k (k 1) 1

k2 k k 2 k 1 k 2 ( k 1)k 1
1 2 n(n 1) 1 2 n(n 1) 1
. .
n n 1 1.2 1 3 n(n 1)

eroberogeday lwm pln TMBr 150


sIVtncMnYnBit nig esrI
n
k 1 n k n k (k 1) 1

k2 k k2 k 1 k 2 (k 1)k 1
1 2 n(n 1) 1 2 n(n 1) 1
. .
n n 1 1.2 1 3 n(n 1)
2 1 2 1
1 1
3 n(n 1) 3 n (n 1)
1 1 1
1 3 1 3 1 3
dUcenH 2 3 ...... n 2 1 1
.
1 1 1 3 n (n 1)
1 3 1 3 1 3
2 3 n

lMhat;TI28
eK[sIVtncMnYnBit (un ) kMnt;eday
1 1
u 0 1 nig u n1 ( 2u n 1 8u n )
8 4
k> tag v n 1 8u n . bgajfa v n1 12 ( v n 2)
x>bgajfasIVt w n v n 2 CasIVtFrNImaRt rYcKNna w n nig v n
CaGnuKmn_n n .
K> TajrktY u n nsIVt .

eroberogeday lwm pln TMBr 151


sIVtncMnYnBit nig esrI

dMeNaHRsay
k> bgajfa v n1 12 ( v n 2)
eKman v n 1 8u n
naM[ v n1 1 8u n1 Et u n1 18 ( 14 2u n 1 8u n )
1
v n1 1 2u n 1 8u n
4
1 8u n 1 8u n
v n1 1 2.
2 4
1 8u n 2 1 8u n 1
v n1 (1 ) 1 1 vn
2 2 2
dUcenH v n1 12 ( v n 2) .
x>bgajfasIVt w n v n 2 CasIVtFrNImaRt
eyIgman w n v n 2 naM[ w n1 v n1 2
Et v n1 12 ( v n 2)
1 1 1
eK)an w n 1 ( v n 2 ) 2 ( v n 2 ) w n
2 2 2
.
dUcenH (w n ) CasIVtFrNImaRtmanersug q 12 .

eroberogeday lwm pln TMBr 152


sIVtncMnYnBit nig esrI

tamrUbmn w n w 0 q n Et w 0 v 0 2 1 8u 0 2 1

dUcenH w n 1n nig v n 2 1n .
2 2
K> TajrktY u n nsIVt
vn 2 1
eyIgman v n 1 8u n naM[ u n 8
1
(2 n )2 1
un 2
8
1 1
4 4. n n 1
2 4
8
3 1 1
n 1 2n 3
8 2 2
dUcenH u n 83 n11 21n 3 .
2 2

eroberogeday lwm pln TMBr 153


sIVtncMnYnBit nig esrI

lMhat;TI29
eK[sIVtncMnYnBit (u ) kMnt;eday n n 0

u 4 ; u 7 ; u 5 nig TMnak;TMngkMenIn
0 1 2

un 3 10un 2 31un 1 30un


Edl n 0 ; 1 ; 2 ; 3 ; ... .
k>cUrkMnt;RKb;KUcMnYnBit r ; ; edIm,I[cMeBaHRKb; n 0
eK)an u u u r u u u
n 3 n 2 n1 n 2 n1 n

x> cUrKNna u CaGnuKmn_n n .


n

dMeNaHRsay
k>kMnt;RKb;KUcMnYnBit r ; ;
eKman u u u r u u u *
n 3 n 2 n1 n 2 n1 n

tamTMnak;TMng * eKGacsresr
un 3 ( r )un 2 ( r ) un 1 r i
tambMrab; u 10u 31u 30u
n 3 n 2 n1 n
ii
edayeRbobeFob i nig ii eK)an

eroberogeday lwm pln TMBr 154


sIVtncMnYnBit nig esrI

r 10 r 10 1

r 31 b r 31 2
r 30

30
3
r
yksmIkar 1 & 3 CMnYskg 2 eK)an
30
r ( r 10) 31
r
r 3 10r 2 31r 30 0
( r 2)( r 3)( r 5) 0
eKTajbs r { 2 ; 3 ; 5 } .
yk r { 2 ; 3 ; 5 } CMnYskgsmIkar 1 & 3 eK)an
{ 8 ; 7 ; 5 } nig { 15 ; 10 ; 6 } .
dUcenH (r ; ; ) { 2 ; 8 ;15; 3 ; 7 ;10; 5 ; 5 ; 6}
x>KNna u CaGnuKmn_n n
n

yk (r ; ; ) { 2 ; 8 ;15; 3 ; 7 ;10; 5 ; 5 ; 6}
CYskTMnak;TMng * eK)an
un 3 8un 2 15un 1 2( un 2 8un 1 15un ) (a )

un 3 7 un 2 10un 1 3( un 2 7 un 1 10un ) (b )
u 5u 6u 5( u 5u 6u )
n 3 n 2 n1 n 2 n1 n (c )

eroberogeday lwm pln TMBr 155


sIVtncMnYnBit nig esrI
xn un 2 8un 1 15un

tagsIVtCMnYy yn un 2 7 un 1 10un
z u 5u 6u
n n 2 n1 n

eyagtamsmIkar a ; b & c eK)an


xn 1 2 xn

yn 1 3 yn
z 5z
n1 n

naM[ x ; y ; z suTEtCasIVtFrNImaRtEdlman
n n n

pleFobrYmerogKa q 2 ; q 3 ; q 5
x y z

x0 u2 8u1 15u0 5 56 60 9

nigtY y0 u2 7 u1 10u0 5 49 40 4
z u 5u 6u 5 35 24 6
0 2 1 0

x n x0 q x n 9 2 n

tamrUbmneK)an n y y 0 q y 4 3
n n


zn z0 qz 6 5n
x

un 2 8un 1 15un 9 2n

eKTaj un 2 7 un 1 10un 4 3
n

z u 5u 6u 6 5 n
n n 2 n1 n

bnab;BIedaHRsayeK)an u n 3 . 2 n 2 .3 n 5 n

eroberogeday lwm pln TMBr 156


sIVtncMnYnBit nig esrI

lMhat;TI30
eK[sIVtncMnYnBit (u ) kMnt;eday n n 0

u u 1 ; u 2 nig TMnak;TMngkMenIn
0 1 2

un 3 6un 2 12un 1 8un


Edl n 0 ; 1 ; 2 ; 3 ; ... .
cUrKNna u CaGnuKmn_n n .
n

dMeNaHRsay
KNna u CaGnuKmn_n n
n

TMnak;TMng u 6u 12u
n 3 n 2 n1 8un Gacsresr
un 3 4un 2 4un 1 2( un 2 4un 1 4un ) i
tag v u 4u 4u
n n 2 n1 n

eK)an v u 4u 4u
n1 n 3 n 2 n1

tam (i ) eKTaj v 2v naM[ (v ) CasIVtFrNImaRtman


n1 n n

pleFobrYm q 2 nig tY v u 4u 4u 2 0 2 1 0

tamrUbmn v v q 2
n 0
n n1

eKTaj u 4u 4u 2
n 2 n1 n
n1

eroberogeday lwm pln TMBr 157


sIVtncMnYnBit nig esrI

b (u 2u ) 2(u 2u ) 2
n 2 n1 n1 n
n1

b u 2 2u u 2 2u 1 ii
n 2
n1
n1 n1
n
n

tag w u 2 2u enaH w u 2 2u
n
n1
n
n
n1
n 2
n1
n1

tam ii eKTaj w w 1 efr enaH w CasIVtnBVn


n1 n n

manplsgrYm d 1 nigtY w u 2 2u 1 2 1 0
1
0
0

eK)an w w nd n 1
n o

eKTaj u 2 2u n 1
n1
n
n

b u2 2u n 1
n1
n
n
n 1

eK)an u2 2u k 1
n 1

k 0
k 1
k
k
k 1
n 1

k 0

un ( n 2)( n 1)
2 u 1
2n 1
0
2
un n 2 3n
n 1
2
2 2
dUcenH u n ( n 2 3 n 4 ) .2 n 2 .

eroberogeday lwm pln TMBr 158


sIVtncMnYnBit nig esrI

lMhat;TI31
eK[sIVtncMnYnBit (u ) nig v kMnt;eday
n n 0 n n 0

u 4 u u 6u v 6v 3 2 3

v 2
0
nig n1 n n n n

0 vn 1 3un 2vn 9unvn 2 7vn 3


Edl n 0 ; 1 ; 2 ; 3 ; ... .
k> cMeBaHRKb; n 0 cUrRsayfa u v 0 nig u n n n 2vn 0
x> bgajfaeKGackMnt;cMnYnBit r edIm,I[)an
un 1 r vn 1 un r vn
3

K> cUrKNna u nig v CaGuKmn_n n .


n n

dMeNaHRsay
k> cMeBaHRKb; n 0 Rsayfa u v 0 n n

eKman u v 4 2 6 0 Bit
0 0

]bmafavaBitdl; n k KW u v 0 Bit k k

eyIgnwgRsayfavaBitdl; n k 1 KW u v k 1 k 1 0 Bit
eKman u v u 3u v 3u v v
k 1 k 1 n
3
n
2
n n n
2
n
3

u v u v 0 Bit
3
k 1 k 1 k k

dUcenH u v 0 cMeBaHRKb; n 0 .
n n

eroberogeday lwm pln TMBr 159


sIVtncMnYnBit nig esrI

cMeBaHRKb; n 0 Rsayfa u 2v 0 n n

eKman u 2v 4 4 8 0 Bit
0 0

]bmafavaBitdl; n k KW u 2v 0 Bit k k

eyIgnwgRsayfavaBitdl; n k 1 KW u 2v 0 Bit k 1 k 1

eKman u 2v u 6u v 12u v 8v
k 1 k 1 n
3
n
2
n n n
2
n
3

u 2v u 2v 0 Bit
3
k 1 k 1 k k

dUcenH u 2v 0 cMeBaHRKb; n 0 .
n n

x> kMnt;cMnYnBit r edIm,I[)an


un 1 r vn 1 un r vn *
3

un 1 un 3 6unvn 2 6vn 3
eday eK)an
vn 1 3un 2vn 9unvn 2 7vn 3

un 1 r vn 1 un 3run vn ( 9r 6)un vn ( 7 r 6)vn i


3 2 2 3

ehIy u r v u 3ru v 3r u v r v
n n
3
n
3
n
2
n
2
n n
2 3
n
3
ii
edayeRbobeFobTMnak;TMng i nig ii eKTaj)an
3r 2 9r 6 r 2 3r 2 0
3 3
r 7r 6 r 7r 6 0
dUcenH r1 1 ; r2 2 .

eroberogeday lwm pln TMBr 160


sIVtncMnYnBit nig esrI

K> KNna u nig v CaGuKmn_n n


n n

yktm r 1 ; r 2 CYskg * eK)an


1 2

un 1 vn 1 ( un vn )3

un 1 2vn 1 un 2vn
3

lnun 1 vn 1 3 lnun vn

lnun 1 2vn 1 3 lnun 2vn
tamTMnak;TMngenHbBaak;fa { ln(u v )} nig {ln(u 2v )} n n n n

suTEtCasIVtFrNImaRtEdlmanpleFobrYm q 3 dUcKa .
eK)an lnlnuu v2v 33lnlnuuv 2v)3 3ln 6ln 8
n n
n n 0 0
n n
n n 0 0

un vn 6 3n

eKTaj
un 2vn 83
n

bnab;BIedaHRsayRbBnenHeKTTYl)an
u 2 6 8 nig v 8 6
n
3n 3n
n
3n 3n
.

eroberogeday lwm pln TMBr 161


sIVtncMnYnBit nig esrI

lMhat;TI32
21
sIVt {an } kMNt;eday a1 16 nigcMeBaH n 2 : 2an 3an1 n31
2
eKyk m CacMnYnKt;mYyEdl m 2 . cUrbgajfacMeBaH n m
1 n ( m 1)
2 m m2 1
eyIg)an an 3
n 3
m
m 3 mn1 .
2

dMeNaHRsay
1 n ( m 1)
3 2 m m2 1
bgajfa an n 3
m
m 3 mn1
2

3
eKman 2an 3an 1
2n 1
naM[ 2n an 3.2n 1 an 1 43
n n 1
b 23 an 23 an 1 43 1n
3
n 2 k k 1 3 n 1
2
eK)an 3 ak 3 ak 1 4 k
k 2 k 2 3

eroberogeday lwm pln TMBr 162


sIVtncMnYnBit nig esrI
n 1
1
n 1
2 2 3 1 3
an a1 2 .
3 3 4 3 1
1
3
n
2 2 21 1 1
an . 1 n 1
3 3 16 8 3
n n
3 3 3 3
eKTaj)an an 2 n 3 b an n 3 2
2 2
1 n ( m 1)
3 m 2 m
tag P an n 3 m 3
2

n n ( m 1)

eK)an P 32 m m 23 m


m2 1 (m 1)(m 1) m 1
eKman m n 1 (m 1) n
n
1
m1
m2 1
edIm,IRsay[eXIjfa P m n 1
eyIgRtUvRsay[eXIjfa (1 mn 1) P m 1
tamvismPaB Bernoully eKman

eroberogeday lwm pln TMBr 163


sIVtncMnYnBit nig esrI
n

n

n 1 n m 1
1 (1 )
m1 m1 m 1 1
1
m
mn n

n
m 1 1
naM[ 1 m 1
1 m
1
1
(1 )
m m
cMeBaHRKb; m 2 tameTVFajtun eKman
m
1 1 1 1 2 1
1 1 Cm 2 Cm .... m Cm
m
m m m m
m 2
enaH 1 m1 1 1 2 5 1
1 C1m 2 Cm
9 3

m m 2 2m 4 2
2n
m 2n
2 m
eKTaj)an 1 mn 1 2
b 1
n

m 1 3
3
2n

eKTaj 1 mn 1 P 3 2m
P

n n ( m 1)
3 m 2 m
Et P m 3
2

eroberogeday lwm pln TMBr 164


sIVtncMnYnBit nig esrI
n n
( m 1)

eK)an (1 m 1) P 3 m 3
n 2 m2 m



n
( m 1)
n
2 m 2 m
]bmafa 3 m 3 m 1 Bit


n
2 m
edaytag u 3 enaH u(m um 1 ) m 1

smmUl mu um m 1 0
m(u 1) (um 1) 0
(u 1)[m (um 1 .... u 1)] 0
n
2 m
eday m n & m 2 enaH 0 u 3 1

naM[ (u 1)[m (um 1 .... u 1)] 0 Bit
n ( m 1)
dUcenH a 3 m 2 m 1
1
2
m m
2n 3
n
3 mn1

eroberogeday lwm pln TMBr 165


sIVtncMnYnBit nig esrI

lMhat;TI33
eKkMNt;cMnYn a0 , a1 , a2 , ...., an dUcxageRkam
1 ak 2
a 0 ; ak 1 ak ( n 1 , k 0 , 1 , 2 , ...., n 1 )
2 n
cUrbgajfa 1 n1 an 1 .
dMeNaHRsay
1
bgajfa 1
n
an 1

ak 2 a k (n a k )
eKman ak 1 ak n n
eK)an a 1 a (an n) a1 a 1 n
k 1 k k k k

b a 1 n a1 a 1
k k k 1
n 1 n 1 1
ehtuenH a n ( a a 1 ) a1 a1 2 a1 (*)
1
k 0 k k 0 k k 1 0 n n

eKman a0 12 0 Bit . ]bmafa ak 0 Bit


ak 2
tam ak 1 ak n eKTaj)an ak 1 0 Bit
dUcenH ak 0 enaH ak n n b a 1 n n1 , n 1
k

eroberogeday lwm pln TMBr 166


sIVtncMnYnBit nig esrI
n 1 1 n 1 1 1 1 1 n
eK)an ... 1
ak n k 0 n n
n n n
k 0
(n )

tam (*) eKTaj)an 2 a1 1 naM[ an 1 (i )


n
ma:geToteday an 1 enaH ak 1 b ak n n 1
b a 1 n n 1 1 RKb; k 0 , 1 , 2 , ...., n 1 .
k
n 1 1 n 11 n
eK)an ak n
n1 n1
k 0 k 0

edayBiniteXIjfaRKb; n 1 eKman n n 1 nn 12 22 0
n 1
n 1
enaHeKTaj)an a 1 n n n 1 nn 12
k 0 k

tam (*) eKTaj)an 2 a1 nn 12


n

b a1 2 nn 12 n n 1 enaH an n n 1 1 n1 (ii )
n

tam (i ) & (ii ) eKTaj)an 1 n1 an 1 .


dUcenH 1 n1 an 1

eroberogeday lwm pln TMBr 167


sIVtncMnYnBit nig esrI

lMhat;TI34
3 1 1 1 0 0
eK[m:aRTIs A 1 3 1 nig I 0 1 0
1 1 3 0 0 1
bgajfamansIVtcMnYnBitBIr (un ) nig ( vn ) Edlepgpat;
n IN : An un . I v n .A EdleKnwgbBaak;tYTUeTA un nig vn
CaGnuKmn_n n .
dMeNaHRsay
bgajfa n IN : An un . I vn .A
3 1 1 1 0 0 3 1 1
eKman A 1 3 1 0.0 1 0 1.1 3 1
1 1 3 0 0 1 1 1 3
eK)an A1 u1 .I v1 .A BitcMeBaH n 1 Edl u1 0 , v1 1 .
3 1 1 3 1 1 11 7 7
A 2 A.A 1 3 1 .1 3 1 7 11 7

1 1 3 1 1 3 7 7 11
10 0 0 21 7 7
A2 0 10 0 7 21 7 10.I 7.A

0 0 10 7 7 21
eK)an A 2 u2 .I v 2 .A BitcMeBaH n 2 .

eroberogeday lwm pln TMBr 168


sIVtncMnYnBit nig esrI

]bmafavaBitcMeBaHRKb; n IN KW An un .I vn .A
eyIgnwgRsayfa An 1 un 1 .I vn 1 .A Bit .
eKman An 1 A.An A.(un .I vn .A) un .A.I vn .A 2
eday A.I A nig A 2 u2 .I v 2 .A
eK)an An 1 un .A vn (u 2 .I v 2 .A)
An 1 u 2 v n .I (un v 2 v n ).A
A n 1 un 1 .I v n 1 .A Bit
Edl un 1 u 2 vn 10.vn nig vn 1 un v 2 vn un 7 vn
dUcenH n IN : An un . I vn .A
Edl (u n ) nig ( vn ) CasIVtcMnYnBitBIrkMNt;eday u1 0 , v1 1
nig u n 1 10.vn , vn 1 u n 7vn RKb; n IN .
bBaak;tYTUeTA un nig vn CaGnuKmn_n n
eKman un 1 10.vn , vn 1 un 7 vn RKb; n IN
tam vn 1 un 7 vn eK)an vn 2 un 1 7 vn 1
Et un 1 10vn enaH vn 2 7 vn 1 10vn
smIkarsmal;n vn 2 7 vn 1 10vn KW r 2 7r 10
b r 2 7r 10 0 manbs r1 2 , r2 5 .

eroberogeday lwm pln TMBr 169


sIVtncMnYnBit nig esrI
a n v n 1 2 v n
tagsIVtCMnYy b v 5v
n n 1 n
an 1 v n 2 2 v n 1 7 v n 1 10 v n 2 v n 1 5an
eK)an b v 5v 7 v 10v 5v 2b
n 1 n 2 n 1 n 1 n n 1 n
naM[ (an ) nig (bn ) CasIVtFrNImaRtmanersug 5 nig 2 erogKa .
eK)an an a1 .5n 1 nig bn b1 .2n 1 .
eday a1 v 2 2v1 7 2 5 nig b1 v 2 5v1 7 2 2
eKTaj an 5.5n 1 5n nig bn 2.2n 1 2n .
vn 1 2 vn 5n
eK)anRbBn
vn 1 5 vn 2n
5n 2n
dksmIkarBIrenHeK)an 3vn 5 n
2n
naM[ vn
3
ehIy un 1 10vn 103 (5n 2n )
naM[ un 103 (5n 1 2n 1 ) .
5n 2n
dUcenH un 103 (5n 1 2n 1 ) nig vn
3
.

eroberogeday lwm pln TMBr 170


sIVtncMnYnBit nig esrI

lMhat;TI35
cUrkMNt;tYTUeTAnsIVtEdlkMNt;eday
x 3 , x 4 nig x x nx cMeBaHRKb; n .
0 1 n 1
2
n 1 n

dMeNaHRsay
KNnatYtUeTA x n

eKman
x0 3 0 3
x1 4 1 3
x 2 x 0 2 x1 9 4 5 2 3
]bmafa x n 2 , x n 3 Bit
n 1 n

eyIgnwgRsayfa x n 4 n 1

eKman x x nx
n 1
2
n 1 n

(n 2)2 n(n 3)
n 2 4n 4 n 2 3n
n4
dUcenH xn n 3 .

eroberogeday lwm pln TMBr 171


sIVtncMnYnBit nig esrI

lMhat;TI36
cUrKNnaplbUk
3 4 n2
Sn ...
1! 2! 3! 2! 3! 4! n! (n 1)! (n 2)!
dMeNaHRsay
KNnaplbUk S n

2
eKman S k ! (k k 1)!
n
n

k 1 (k )!
k2
tag ak
k ! (k 1)! (k 2)!
k2

k ![1 (k 1) (k 1)(k 2)]
k2

k !(1 k 1 k 2 3k 2)
k2 1

k !(k 2)2 k !(k 2)
k 1 (k 2) 1

(k 2)! (k 2)!
1 1

(k 1)! (k 2)!
dUcenH Sn kn1 ak 12 (n 1 2)!

eroberogeday lwm pln TMBr 172


sIVtncMnYnBit nig esrI

lMhat;TI37
cMeBaHRKb;cMnYnKt;viCman n eK[
1 1 1
Sn 1 ....
2 3 n
Tn S1 S 2 S 3 .... Sn
T1 T2 T3 T
Un .... n
2 3 4 n1
cUrkMNt;edayeFIVdMeNaHRsay nUvcMnYnKt; 0 a, b, c, d 1000 000
edaydwgfa T1988 aS1989 b nig U1988 cS1989 d .
dMeNaHRsay
kMNt;cMnYnKt; a , b , c , d
eyIgnwgRsaytamkMeNInfaRKb; n 2 : Tn 1 nSn n
-cMeBaH n 2 : T1 2S2 2 2(1 12 ) 2 1 S1 Bit
-]bmafa Tn 1 nSn n Bit
eyIgnwgRsayfa Tn (n 1)Sn 1 (n 1) Bit
eKman Tn S1 S2 ... Sn 1 Sn Tn 1 Sn
Tn nSn n Sn (n 1)Sn n
eday Sn 1 Sn n 1 1 enaH Sn Sn 1 n 1 1

eroberogeday lwm pln TMBr 173


sIVtncMnYnBit nig esrI

eK)an Tn (n 1)(Sn 1 n 1 1) n (n 1)Sn 1 (n 1) Bit


dUcenH n 2 : Tn 1 nSn n
yk n 1989 eK)an T1988 1989 S1989 1989
eday T1988 aS1989 b enaHeKTaj a 1989 , b 1989 .
n
ehIy Un 2 3 4 ... n 1 ( kTk 1)
T1 T2 T3 Tn
k 1
n (k 1)Sk 1 (k 1)
Un k 1
k 1
n
(Sk 1 1)
k 1
S 2 S 3 .... Sn 1 n
Tn 1 S1 n
(n 1)Sn 1 (n 1) 1 n
(n 1)Sn 1 2(n 1)
yk n 1988 eK)an U1988 1989S1989 3978
eday U1988 cS1989 d enaH c 1989 , d 3978
dUcenH a 1989 , b 1989 , c 1989 , d 3978 .

eroberogeday lwm pln TMBr 174


sIVtncMnYnBit nig esrI

lMhat;TI38
ana n 1
eK[sIVt a1 1, a 2 5 nig an 1 n 2
an a
2 2
n 1
1
cUrkMNt;tYTUeTAnsIVt {a } CaGnuKmn_n n .
n

dMeNaHRsay
kMNt;tYTUeTAnsIVt {a } CaGnuKmn_n n
n

eKman a a a
n 1
n
n 2
n 1

an 2 an2 1 1
eK)an
1 an 2 an2 1 1

an2 1 an 2an2 1
1 1 1 1

an2 1 an 2 an2 1 an 2an2 1
1 1 1 1
1 (1 ) (1 )
an2 1 an 2 an2 1 an 2
1 1 1
1 (1
)(1 )
an2 1 an2 an2 1
b 1 a12 (1 a12 )(1 a12 ) (*)
n 2 n n 1

tag bn ln(1 a12 ) enaH bn1 ln(1 a12 )


n n 1

eroberogeday lwm pln TMBr 175


sIVtncMnYnBit nig esrI
1
ehIy bn 2 ln(1
a 2
) (**)
n 2

yk (*) CMnYskg (**) eKTTYl)an b b b n 2 n 1 n

smIkarsmal; q q 1 0 manbs q 1 2 5
2
1
, q2
1 5
2
1 5 n 1 5 n
eK)an b (
n
2
) (
2
) Edl , kMNt;dUcxageRkam

cMeBaH n 1 b1 . 1 2 5 . 1 2 5
cMeBaH n 2 : b2 3 2 5 3 2 5
eday b1 ln(1 a12 ) ln 2 nig b2 ln(1 a12 ) ln 26 25
1 2

1 5 1 5
ln 2
eK)an 2 2
3 5 3 5 ln 26
2 2 25

eKTaj 12 ln 2 513 1 ln 100 3




5

nig 12 ln 2 513 1 ln 100 3




5
ehIy bn ln(1 a12 ) eKTaj an b1n .
n e 1

eroberogeday lwm pln TMBr 176


sIVtncMnYnBit nig esrI

lMhat;TI39
eKGnuKmn_ f (x) 112x
1 2
cUrKNna S f ( n1 ) f ( n2 ) ... f ( n n 1)
dMeNaHRsay
CadMbUgeyIgRtUvRsayfa ebI p q 1 enaH f (p) f (q) 1
eKman f (p) 11 2p ; f (q) 11 2q
1 2 1 2
1 1
f (p ) f (q )
1 21 2p 1 21 2q
2 2 1 2 p 2 1 2 q

(1 21 2p )(1 21 2q )
2 21 2 p 21 2 q

1 21 2 p 21 2 q 2 2 2 p 2 q
ebI p q 1 enaH 2 2p 2q 2 2(p q) 0
2 21 2p 21 2q
f (p ) f (q ) 1 2p 1 2q
1
2 2 2
n1
eKman 1
n
2
S f ( ) f ( ) ... f (
n n
) (1)
n1 n2
b S f(
n
) f(
n
1
) ... f ( ) ( 2)
n

eroberogeday lwm pln TMBr 177


sIVtncMnYnBit nig esrI

bUksmIkar (1) nig (2) eK)an


1 n1 n1 1
2S [f ( ) f ( )] ..... [f ( ) f ( )]
n n n n
2S 11 1 1 n 1
.....
( n 1 )

dUcenH S n 2 1 .

eroberogeday lwm pln TMBr 178


sIVtncMnYnBit nig esrI

lMhat;TI40
eKmansIVtcMnYnBit (a ) kMNt;eday
n

1 1 1
an (1 )(1 ) ....(1 n )
2 4 22
cUrKNnalImIt lim an
n
.
dMeNaHRsay
KNnalImIt lim a n
n

yk q 12 eK)an
n
an (1 q )(1 q 2 )......(1 q 2 )
x2 y 2
edayeRbIsmPaB xy
xy
eK)an
n 1
1 q2 1 q4 1 q2
an . ....
1 q 1 q2 1 q2
n

2n 1
1
2n 1 1 1 2n 1
1q 2
an 2 1
1q 1 2
1
2
2n 1

kalNa n enaH 12 xitCitsUn



dUcenH lim a 2 .
n
n

eroberogeday lwm pln TMBr 179


sIVtncMnYnBit nig esrI

lMhat;TI41

n 2 12 22 32 n2
eKmanesrIGnnmYy ( 2n ) 2 22 23 ..... 2n ....
n1

cUrRsayfaesrIxagelIenHCaesrIbRgYm .
dMeNaHRsay
12 22 32 n2
plbUkedayEpkrbs;esrIenHKW S n 2 3 ... n
2 2 2 2
BinitGnuKmn_ f (k ) ak 2 bk c EdlRKb;
k IN * eKman
k 2 f (k ) f (k 1)
k
2k 2 2k 1
k 2 ak 2 bk c a(k 1)2 b(k 1) c
b 2 k

2 k

2k 1
b 2k 2 2ak 2 2bk 2c ak 2 2ak a bk b c

2k 2 ak 2 (b 2a )k c a b
eKTaj a 2 ehIy b 2a 0 b 4 nig c a b 6
ehtuenH k2 2k 24k 6 2k 28k 12
2 2 2

k k k 1

2k 2 4k 6 2k 2 8k 12 n 2 4n 6
eK)an n
Sn ( k 1
) 6
k 1 2 k
2 2n
n 2 4n 6
eday lim S n lim 6 n
6
n n
2
dUcenHesrIGnnxagelICaesrIbRgYm .

eroberogeday lwm pln TMBr 180


sIVtncMnYnBit nig esrI

lMhat;TI42
cMeBaHRKb; n IN * eKmanplbUk
5 7 9 2n 3
Sn ....
1.3.22 3.5.23 5.7.24 ( 2n 1)( 2n 1).2n 1
k> kMNt;cMnYnBit a nig b edIm,I[eK)ansmPaB
2n 3 a b

( 2n 1)( 2n 1).2n 1 ( 2n 1).2n ( 2n 1).2n 1
x> KNna S nig lim S .
n
n
n

dMeNaHRsay
k> kMNt;cMnYnBit a nig b edIm,I[eK)ansmPaB
2n 3 a b

( 2n 1)( 2n 1).2n 1 ( 2n 1).2n ( 2n 1).2n 1
eK)an 2n 3 2a(2n 1) b(2n 1)
2n 3 (4a 2b )n 2a b
4a 2b 2
eKTaj

naM[ a 1 , b 1
2a b 3
dUcenH a 1 nig b 1 .
x> KNna S nig lim S
n
n
n

tamsRmayxagelIenHcMeBaH a 1 nig b 1 eK)an

eroberogeday lwm pln TMBr 181


sIVtncMnYnBit nig esrI
2n 3 1 1

( 2n 1)( 2n 1).2n 1 ( 2n 1).2n ( 2n 1).2n 1
2p 3
b
1

( 2p 1)( 2p 1)2p 1 ( 2p 1)2p ( 2p 1)2p 1
1

n
2p 3
eK)an Sn p1
p 1 ( 2p 1)( 2p 1).2
n
1 1

p 1 ( 2p 1)2
p
( 2p 1)2p 1
1 1

2 ( 2n 1).2n 1
1
dUcenH Sn 1
2
1
.
( 2n 1).2n
1
ehIy 1
lim S n lim 1
1
n
.
n 2 n
( 2n 1).2 2

eroberogeday lwm pln TMBr 182


sIVtncMnYnBit nig esrI

lMhat;TI43
cMeBaHRKb; n IN * eKmansIVtcMnYnBit (a ) kMNt;eday n

1
a 3 nig a 1
1
0 n1 a n
2n 2n
3 3
k> KNna a CaGnuKmn_n n .
n

x> KNnalImIt lim a . n


n

dMeNaHRsay
k> KNna a CaGnuKmn_n n
n

eKman a ( 1 1 ) a
n1 n n
1
n
32 32
1 1
an 1 1 (1 ) an (1 )
2n 2n
3 3
1
an 1 1 (1 ) (an 1)
2n
3
an 1 1 1
1 n
an 1 32
n 1
ak 1 1 n 1 1
eK)an a 1 1 k
k 0 k k 0 32
tamsmPaB ( )( )
2 2

2 2
b

eroberogeday lwm pln TMBr 183


sIVtncMnYnBit nig esrI

edayyk 1 nig 3 2k

1
1
2k 1
eK)an 1 1
2k
3
1
3 1 k
32
1
1
n 1
1 n 1 2k 1
ehtuenH 1 2k ( 3
1
)
k 0 3 k0 1
k
32
1
1 n
3 2 3 1
1 n
1 2 32
1
3
n 1
ak 1 1 an 1 an 1
ehIyeday a 1 a 1 2
k 0 k 0

an 1 3
eKTaj 2
(1 n )
2
1
an 4 n b
3
32 32
dUcenH an 4 n
3
32
x> KNnalImIt lim an
n

eK)an lim a lim (4 3 ) 4 eRBaH lim 3


n
n
n 2n n 2n
0
3 3
dUcenH lim a 4 .
n
n

eroberogeday lwm pln TMBr 184


sIVtncMnYnBit nig esrI

lMhat;TI44
KNnaplbUk
n!
Sn C1n 4Cn2 9Cn3 ..... n 2Cnn Edl C p
n
p! (n p )!
dMeNaHRsay
KNnaplbUk
Sn C1n 4Cn2 9Cn3 ..... n 2Cnn
tameTVFajtun (1 x)n Cn0 C1nx Cn2 x2 Cn3 x3 ... Cnn xn
eFVIedrIevelIGgTaMgBIrnsmIkarenHeK)an
n(1 x )n 1 C1n 2Cn2 x 3Cn3 x 2 .... nCnn xn 1
KuNGgTaMgBIrnwg x eK)an
nx (1 x )n 1 C1n x 2Cn2 x 2 3Cn3 x 3 .... nCnn xn
eFVIedrIevelIGgTaMgBIrnsmIkarenHeK)an
n(1 x)n 1 n(n 1)x(1 x)n 2 C1n 4Cn2 x ... n 2Cnn xn 1
yk x 1 CYskgsmPaBenHeK)an
n.2n 1 n(n 1).2n 2 C1n 4Cn2 9Cn3 ... n 2Cnn
[2n n(n 1)]2n 2 C1n 4Cn2 9Cn3 ... n 2Cnn
dUcenH Sn n(n 1).2n 2

eroberogeday lwm pln TMBr 185


sIVtncMnYnBit nig esrI

lMhat;TI46
x4
eKmanGnuKmn_ f ( x ) Edl x 1 .
x1
KNna f n [ f [... f [ f ( x )]...]]
dMeNaHRsay
KNna f n [ f [... f [ f ( x )]...]]
tag a1 f ( x )
a 2 f [ f ( x )] f (a1 )
a 3 f [ f [ f ( x )] ] f (a 2 )


a n f n [ f [.... f [ f ( x )] .... ] ] f (a n 1 )
eK)an an1 f (an ) an 4
an 1
dUcenHkarKNna f n [ f [... f [ f ( x )]...] ] KWRtUvkMNt;tY a n
x4
1a f ( x )
x1
nsVItEdlkMNt;eday
a n1 a n 4 , n 1
an 1

eroberogeday lwm pln TMBr 186


sIVtncMnYnBit nig esrI

smIkarsmal;nsVItKW r r 4
r 1
eK)an r 2 r r 4 naM[ r1 2 , r2 2
a n r1 a n 2
tagsIVtCMnYy bn
a n r2an 2
an 4
2
eK)an bn1 an1 2 aan 14
a n1 2 n
2
an 1
3a 6 a 2
bn 1 n 3. n 3bn
an 2 an 2
naM[ (bn ) CasIVtFrNImaRtmanersug q 3
nigtY b1 a1 2 x 4 2 x 2 3. x 2
a1 2 x 4 2x 2 x2
x2
tamrUbmn bn b1 q
n 1
( 3 ) n
x2
eday bn an 2 eKTaj an 2(b 1)
an 2 b1
2[ ( x 2)( 3) n x 2]
dUcenH an .
( x 2)( 3) x 2
n

eroberogeday lwm pln TMBr 187


sIVtncMnYnBit nig esrI

lMhat;TI45
x0 1
eKmansIVt ( xn ) nig ( yn ) kMNt;eday nig
y0 0
1 1
x n 1 2 (sin a cos a ) x n 2 sin a (1 tan a ) y n

y 1 cos a (cot a 1) x 1 (sin a cos a ) y
n 1 2 n
2
n

Edl 0 a nig n 0 ,1 , 2 , .... .


2
k> cMeBaHRKb; n 0 tag un xn cos a yn sin a nig
v n x n cos a yn sin a .
cUrRsayfa (un ) nig (v n ) suTEtCasIVtFrNImaRt .
x>KNna un nig v n CaGnuKmn_n n nig a .
K> Tajrk xn nig yn CaGnuKmn_n n nig a .
dMeNaHRsay
k> Rsayfa (un ) nig (v n ) suTEtCasIVtFrNImaRt
eKman
1 1
x n1 (sin a cos a ) x n sin a (1 tan a ) y n
2 2

eroberogeday lwm pln TMBr 188


sIVtncMnYnBit nig esrI

KuNGgTaMgBIrnwg cos a eK)an


cos a(sin a cos a ) sin a(cos a sin a )
xn 1 cos a xn yn (1)
2 2
eKman
1 1
y n 1 cos a (cot a 1) x n (sin a cos a ) y n
2 2
KuNGgTaMgBIrnwg sin a eK)an
cos a(cos a sin a ) sin a(sin a cos a )
y n1 sin a xn y n ( 2)
2 2
bUksmIkar (1) nig (2) Gg nig GgeK)an
x n 1 cos a y n 1 sin a cos a ( x n cos a y n sin a )
eday un xn cos a yn sin a
eKTaj)an un1 cos a .un naM[ (un ) CasIVtFrNImaRtman
ersug qu cos a .
dksmIkar (1) nig (2) Gg nig GgeK)an
x n 1 cos a y n 1 sin a sin a ( x n cos a y n sin a )
eday v n x n cos a yn sin a
eKTaj)an v n1 sin a .v n naM[ (vn ) CasIVtFrNImaRtman

eroberogeday lwm pln TMBr 189


sIVtncMnYnBit nig esrI

ersug qv sin a .
x>KNna un nig v n CaGnuKmn_n n nig a
eKman u0 x0 cos a y0 sin a cos a
eK)an un u0 qu n cos a . cos n a cos n1 a
ehIy v 0 x0 cos a y0 sin a cos a
eK)an v n v 0 qv n cos a . sin n a
dUcenH un cos n1 a , v n cos a sin n a .
K> Tajrk xn nig yn CaGnuKmn_n n nig a
eday un xn cos a yn sin a
nig v n x n cos a yn sin a
eK)an un v n 2 x n cos a
cos n 1 a cos a sin n a
eKTaj x n
2 cos a
cos n a sin n a
xn .
2
ehIy un v n 2 y n sin a
cos n 1 a cos a sin n a
eKTaj y n .
2 sin a

eroberogeday lwm pln TMBr 190


sIVtncMnYnBit nig esrI

lMhat;TI46
eK[sIVtncMnYnBit (un ) kMNt;eday
7 1
u1 nig u n 1 u n 2 u n RKb; n 1
2 4
bgajfaeKGackMNt;cMnYnBit a Edl un1 a (un a)2
cMeBaHRKb; n 1 rYcKNna un CaGnuKmn_n n .
dMeNaHRsay
kMNt;cMnYnBit a
eKman un1 un 2 un 1 (1)
4
ehIy un1 a (un a)2 (2)
yk (1) CMnYskg (2) eK)an
1
u n 2 u n a (u n a ) 2
4
1
u n u n a u n 2 2au n a 2
2
4
1
(1 2a )u n a 2 a
4
smIkarenHBitCaniccMeBaHRKb;tm n luHRtaEt

eroberogeday lwm pln TMBr 191


sIVtncMnYnBit nig esrI

1 2a 0
1
2 1 naM ; [ a
a a 4 0 2

dUcenH a 1 .
2
KNna un CaGnuKmn_n n
1 1 1 2
cMeBaH a eK)an un1 (un )
2 2 2
eKTaj ln(un1 1 ) 2 ln(un 1 ) (3)
2 2
tag v n ln(un 1 ) v n1 ln(un1 1 )
2 2
tam (3) eK)an v n1 2 v n naM[ ( v n ) CasIVtFrNImaRt
1
manpleFobrYm q 2 nig v1 ln(u1 ) ln 4
2
2n
eK)an v n v1 q 2 ln 4 2 ln 2 ln 2
n 1 n 1 n

1 1 2n
eday v n ln(un ) eKTaj un 2
2 2
2n 1
dUcenH vn 2 .
2

eroberogeday lwm pln TMBr 192


sIVtncMnYnBit nig esrI

lMhat;TI47
eK[sIVtcMnYnBit (an ) kMNt;eday
a 1 1 , a 2 1

a n 2 a n 1 a n , n 1 , 2 , 3 , ...
1 i 3
eKtagsIVtcMnYnkMupic z n an1 an .
2
1 i 3
k> cUrRsayfa z n1 z n cMeBaHRKb; n 1 .
2
x> cUrdak; 1 i 3 CaTRmg;RtIekaNmaRtrYcTajrk z n CaGnuKmn_
2
n n .
K> TajrktYTUeTAnsIVt an . etI (an ) CasIVtxYbbeT ?
dMeNaHRsay
k> Rsayfa z n1 1 i 3 z n cMeBaHRKb; n 1
2
eKman z n an1 1 i 3 an
2
1 i 3
eK)an z n1 an 2 a n 1
2
eday an 2 an1 an
eroberogeday lwm pln TMBr 193
sIVtncMnYnBit nig esrI

1 i 3
eK)an z n1 an1 an a n 1
2
1 i 3
a n 1 a n
2
1 i 3 2
(a n 1 an )
2 1 i 3
1 i 3 1 i 3
(a n 1 an )
2 2
dUcenH z n1 1 i 3 z n .
2
1 i 3
x> dak; CaTRmg;RtIekaNmaRt
2
eK)an 1 i 3 1 i 3 cos i sin
2 2 2 3 3
Tajrk z n CaGnuKmn_n n
1 i 3
eday z n1 z n enaH ( z n ) CasIVtFrNImaRtncMnYn
2
kMupicEdlmanersug q 1 i 3 cos i sin
2 3 3
nigtY z 1 a 2 1 i 3 a 1 1 i 3 cos i sin
2 2 3 3

eroberogeday lwm pln TMBr 194


sIVtncMnYnBit nig esrI

n
tamrUbmn z n z1 q (cos i sin )
n 1
3 3
n n
tamrUbmndWmreK)an z n cos i sin .
3 3
K> TajrktYTUeTAnsIVt an
1 i 3
eKman z n an1 an
2
an 3
eK)an z n (an1 ) i an (1)
2 2
n n
eday z n cos i. sin (2)
3 3
3 n
tamTMnak;TMng (1) & (2) eK)an an sin
2 3
2 n
dUcenH an 3 sin 3 .
ehIy (an ) CasIVtxYbEdlmanxYb p 2 6 .
3

eroberogeday lwm pln TMBr 195


sIVtncMnYnBit nig esrI

lMhat;TI48
eK[sIVtncMnYnkMupic (z n ) kMNt;eday
2 3i 3i 2 3i
z1 nig z n1 zn
2 2 2
Edl n 1 , 2 , 3 , ... .
k> tag w n z n 1 . bgajfa (w n ) CasIVtFrNImaRtncMnYn
kMupic rYcKNna w n CaGnuKmn_n n edaysresrlTplCaTRmg;
RtIekaNmaRt .
x> Tajbgajfa z n 2 cos n (cos n i sin n ) .
12 12 12
dMeNaHRsay
k> bgajfa (w n ) CasIVtFrNImaRtncMnYnkMupic
eK)an w n1 z n1 1
3i 2 3i
zn 1
2 2
3i 3i
( z n 1) wn
2 2
dUcenH (w n ) CasIVtFrNImaRtncMnYnkMupic .

eroberogeday lwm pln TMBr 196


sIVtncMnYnBit nig esrI

KNna w n CaGnuKmn_n n
eK)an w n w1 qn1
eday w1 3 i cos i sin
2 6 6
nig q 3 i cos i sin
2 6 6
eK)an w n (cos i sin )n
6 6
n n
dUcenH w n cos i sin rUbmndWmr
6 6
n n n
x> Tajbgajfa z n 2 cos (cos i sin )
12 12 12
eKman w n z n 1 enaH z n 1 w n
n n
z n 1 cos i sin
6 6
n n n
2 cos 2 2i . sin cos
12 12 12
n n n
2 cos (cos i sin )
12 12 12
dUcenH z n 2 cos n (cos n i sin n ) .
12 12 12

eroberogeday lwm pln TMBr 197


sIVtncMnYnBit nig esrI

lMhat;TI49
eK[sIVtncMnYnBit (un ) nig ( v n ) kMNt;eday
2 un vn
u1 2 u n 1 2
nig Edl n 1
v 2 v un vn
1 2 n 1 2
.
k> eKBinitsIVtncMnYnkMupic z n un i.v n
cUrRsayfa (z n ) CasIVtFrNImaRtncMnYnkMupic rYcKNna z n
CaGnuKmn_n n edaysresrlTplCaTRmg;RtIekaNmaRt .
x> sMEdg un nig v n CaGnuKmn_n n .
dMeNaHRsay
k> Rsayfa (z n ) CasIVtFrNImaRtncMnYnkMupic
eKman z n un i.v n
eK)an z n1 un1 i.v n1
un vn u vn
i. n
2 2
2i 2 2i 2
(u n iv n ) zn
2 1

eroberogeday lwm pln TMBr 198


sIVtncMnYnBit nig esrI

dUcenH(z n ) CasIVtFrNImaRtncMnYnkMupic .
KNna z n CaGnuKmn_n n
eK)an z n z1 qn1
2 2
Et z1 u1 iv1 i cos i. sin
2 2 4 4
2 2
nig q i cos i. sin
2 2 4 4
n
eK)an z n (cos i sin )
4 4
dUcenH z n cos n i. sin n rUbmndWmr
4 4
x> sMEdg un nig v n CaGnuKmn_n n
eKman z n un i.v n
eday z n cos n i. sin n
4 4
dUcenH un cos n4 nig v n sin n4 .

eroberogeday lwm pln TMBr 199


sIVtncMnYnBit nig esrI

lMhat;TI50
eK[sIVtncMnYnBit (un ) nig ( v n ) kMNt;eday
u 0 1 u n 1 u n 2 v n 2
nig Edl n 0
v0 3 v n 1 2u n v n
k> eKBinitsIVtncMnYnkMupic z n un i.v n .
2n
cUrRsayfa z n1 z n rYcTajfa z n z 0 .
2

x> sMEdg un nig v n CaGnuKmn_n n .


dMeNaHRsay
2n
k>Rsayfa z n1 z n rYcTajfa z n z 0
2

eKman z n un i.v n
eK)an z n1 un1 iv n1
u n 2 v n 2 2iu n v n
u n 2 2iu n v n (iv n ) 2
(u n iv n ) 2
dUcenH z n1 z n 2 .

eroberogeday lwm pln TMBr 200


sIVtncMnYnBit nig esrI

ma:geTotebI n 0 enaH z1 z 0 2
ebI n 1 enaH z 2 z12 z 0 4
ebI n 2 enaH z 3 z 2 2 z 08
2k
]bmafavaBitdl;tYTI k KW z k z 0
2k 1
eyIgnwgRsayfavaBitdl;tYTI k 1 KW z k 1 z 0
2k
eKman z k 1 z k Ettamkar]bma z k z 0
2

2k 2 2k 1
eK)an z k 1 (z 0 ) z 0 Bit .
2n
dUcenH z n z 0 .
x> sMEdg un nig v n CaGnuKmn_n n
2n
eKman z n z 0 eday z 0 u 0 iv 0 1 i 3

2(cos i sin )
3 3
2n 2n
eK)an 2n
Zn 2 (cos
3
i sin
3
)

2n 2n 2n 2n
dUcenH un 2 cos ; v n 2 sin .
3 3

eroberogeday lwm pln TMBr 201


sIVtncMnYnBit nig esrI

lMhat;TI51
eK[sIVtncMnYnBit (u ) kMnteday n
4

u 1 nig u RKb
u
n 1
n
n IN
4u 6u 4u 1
0 3 2
n n n
4
1
cUrbgHajfa 1
1
1 RKb n IN
un 1 un
rYcKNna un CaGnuKmn_n n .
dMeNaHRsay
4
1
bgHajfa 1
1
1
un 1 un
4u n 6u n 4u n 1
3 2

eyIgman 1
1
un 1
1
un
4

u 4u n 6u n 4u n 1
4 3 2

n 4
un
4
(u n 1)4 1
4
1
un u n
4
1
dUcen 1
1
1 .
un 1 un

KNna un CaGnuKmn_n n

eroberogeday lwm pln TMBr 202


sIVtncMnYnBit nig esrI
4
1
eyIgman 1
1
1
un 1 un
1 1
eKn ln 1 4 ln 1
un 1 un
1 1
tag v n ln 1 naM[
v n 1 ln 1
u n u n1

eKn v 4v .n1 n

TMnakTMngenbBaakfa ( v ) CasIVtFrNImaRtmanersug
n q4

nigtY v ln 1 u1 ln 2 ( eRBa u 1 ) .
0 0
0
tamrUbmn v v .q 4
n 0
n n
ln 2 .
eday v ln 1 u1
n
n
1
eKTaj ln 1 4n ln 2 naM[ 1
1
24
n

un un

dUcen un 4n
1
.
2 1

eroberogeday lwm pln TMBr 203


sIVtncMnYnBit nig esrI

lMhat;TI52
a 0 a 1 1
eK[sIVtcMnYnBit
an 1 2an an 1 2 , n 1
cUrRsayfa an 2 1 an an 1 .
dMeNaHRsay
Rsayfa a a a n 2 1 n n1

eKman a 2a a
n1 n n 1

tag c a a RKb
n n n 1 n1

eKn c a a
n1 n1 n

cn 1 ( 2an an 1 2) an cn 2
TMnakTMngenbBaakfa (c ) CasIVtnBVnmanplsgrYm
n d2
nigtY c a a 1 1 0 .
1 1 0

eKn c c (n 1)d 2(n 1) 2n 2


n 1

eKTaj a a 2n 2
n n 1

ak ak 1 2 k 1
n n

k 1 k 1

n(n 1)
an a0 2
2
an 1 n(n 1)

eroberogeday lwm pln TMBr 204


sIVtncMnYnBit nig esrI

eKTaj a n n2 n 1 nig an 1 (n 1)2 (n 1) 1

a n
n1
2
n1 nig an 2 1 n 4 n 2 1

eday a a n n1 (n 2 n 1)(n 2 n 1) (n 2 1)2 n 2

aa n n1 n4 n2 1 .
dUcen a n 2 1
an an 1 .

eroberogeday lwm pln TMBr 205


sIVtncMnYnBit nig esrI

lMhat;TI53
eK[sIVt { a } kMnteday
n

a 1 nig a a .a ....a 4 cMeBaRKb n 1 .


0 n 1 0 1 n

cUrbgHajfa a a 2 cMeBaRKb n 1 .
n n1

dMeNaHRsay
bgHajfa a a 2
n n 1

eyIgseg;teXIjfaRKb k IN eKman ak 0 .
eKman a a .a ....a 4
n 1 0 1 n

eKn a a .a .....a 4
n 2 0 1 n1

an 2 (a 0 .a1 .....an )(a 0 .a1 .....an 4) 4


an 2 (a 0a1 .....an )2 4(a 0 .a1 ....an ) 4
an 2 (a 0a1 .....an 2)2
eKTaj an 2 a 0a1 .....an 2
a n 2 (a n 1 4) 2 an 1 an 2 2
dUcen an an 1 2 .

eroberogeday lwm pln TMBr 206


sIVtncMnYnBit nig esrI

lMhat;TI54
kKNna 11.2 21.3 31.4 .... (n 11) n Edl n 2
xedayeRbIvismPaB AM GM n (n 1) cMnYnxageRkam
1
;
1
;
1
;
1
1.2 2.3 3.4 (n 1) n
cU r bgH a jfa n ( n! ) . n 2

dMeNaHRsay
n
kKNna 1

1
1.2 2.3 3.4

1
....
1

1
(n 1) n k 2 (k 1) k
k (k 1)
eKman 1
(k 1) k

(k 1) k

1
k 1 k

1

n n 1 1
eKn
1
k 2 (k 1) k

k2 k 1 k 1
1
n
1 1 1 1 1
dUcen
1.2 2.3 3.4
....
(n 1) n
1
n
.
x bgHajfa n ( n ! )
n 2

tamvismPaB AM GM
a 1 a 2 a 3 ..... a n n n a 1 .a 2 . a 3 ......a n ; a k 0
eKn
1 1 1 1 1 1
.... (n 1) ( n 1) . ......
1.2 2.3 (n 1) n 1 .2 2 .3 (n 1)n

eroberogeday lwm pln TMBr 207


sIVtncMnYnBit nig esrI
1 1
1 (n 1) ( n 1)
n n ! (n 1) !
n1 1
(n 1) ( n 1)
n n! (n 1) !
1 n
( n 1 )
n (n ! ) 2
n 1
1 n
nn ( n! )2
n
2
(n! )
dUcen nn ( n! )2 .

eroberogeday lwm pln TMBr 208


sIVtncMnYnBit nig esrI

lMhat;TI55
eKBinitsIVtncMnYnBit (u n ) kMnteday
u 1 4

u n 1 u n , n IN *
cUrKNna un CaGnuKmn_n n .
dMeNaHRsay
KNna u CaGnuKmn_n n
n

eKman u un 1 n

eKn ln u 12 ln u
n 1 n

TMnakTMngenbBakfa (ln u ) CasIVtFrNImaRtmanersug


n q
1
2
tamrUbmn ln u ln u q 21 ln 4 ln4
1
n 1
n 1 n 1 2n 1

dUcen u 4 .
1

n 2n 1

eroberogeday lwm pln TMBr 209


sIVtncMnYnBit nig esrI

lMhat;TI56
eK[sIVtncMnYnBit (Un ) kMntelI IN eday
U 0 2

U n 1 2 U n , n IN
kcUrKNna U CaGnuKmn_n n .
n

xKNnaplKuN P U U U n 0 1 2 .... U n .
dMeNaHRsay
KNna U CaGnuKmn_n n
n

eyIgman U 2 2 cos 4
0


U 1 2 U 0 2 2 cos 2 cos
4 8

bmafavaBitdltYTI p KW U p 2 cos p 2
2
eyIgnwgRsayfavaBitdltYTI (p 1) KW U 2 cos 2 Bit p 1 p 3

eyIgman U 2 U Ettamkarbma U 2 cos 2


p 1 p p p 2

eyIgn U 2 2 cos 2 4 cos 2 2 cos 2 Bit


p 1 p 2
2
p 3 p 3


dUcen U n 2 cos
2n 2
.

eroberogeday lwm pln TMBr 210


sIVtncMnYnBit nig esrI

x KNnaplKuN P U U U .... U
n 0 1 2 n

tamrUbmn sin 2a 2 sin a cos a naM[ 2 cos a sin 2a


sin a

sin
n n n
2k1
Pn (Uk ) (2cos k2 ) ( )
k0 k0 2 k0
sin k2
2

sin
2 1

sin n2 sin n2
2 2
dUcen Pn
1

.
sin n 2
2

eroberogeday lwm pln TMBr 211


sIVtncMnYnBit nig esrI

lMhat;TI57
eK[sIVtncMnYnBit (Un ) kMntelI IN eday
1 1 Un
2

U0 nig U
2
2
n 1
2
, n IN

KNna U CaGnuKmn_n
n n .
dMeNaHRsay
KNna U CaGnuKmn_n n
n

eyIgman U 22 sin 4
0


1 1 sin 2
1 1 U0 4 sin
2

U1
2 2 8

bmafavaBitdltYTI p KW U p sin p 2
2

eyIgnwgRsayfavaBitdltYTI (p 1) KW U p 1 sin
2 p 3
Bit
1 1 Up
2

eyIgman U p 1
2
karbma U p sin
2p 2

1 1 sin 2
eyIgn U p 1 2p 2
2

eroberogeday lwm pln TMBr 212


sIVtncMnYnBit nig esrI

1 cos 2 sin 2
2p 2 2p 3 sin Bit
2 2 2p 3

dUcen U n sin n 2
.
2

lMhat;TI58
eK[sIVtncMnYnBit (u n ) kMnteday
u 0 a

n 2 , n IN , a 2
2
n 1
u u
2n 2n
a a 4 2
a a 42
cUrRsaybBaakfa u n
2


2



dMeNaHRsay
2n 2n
a a 4 2
a a 4 2
RsaybBaakfa
un
2


2



a a2 4 a a2 4
ebI n0 eKn u0
2

2
a Bit
2k 2k
a a 4 a a 4
bmavaBitdltYTI k KW
2 2
u k


2 2
eyIgnwgRsayfavaBitdltYTI k 1 KW

eroberogeday lwm pln TMBr 213


sIVtncMnYnBit nig esrI
2k 1 2k 1
a a 4 2
a a 4 2
u k 1


2 2
eyIgman u k 1 u k 2
2

2k 2k
a a 4
a a 4
2 2
Ettamkarbma u k
2 2

2

2k
a a2 4
2k
2

eyIgn u k 1


a a
2
4
2
2


2k 1 2k 1
a a2 4 a a2 4 a2 a2 4
u k 1

2 2
2 2 4
2k 1 2k 1
a a 4 2
a a 4 2
u k 1
2


2


Bit

2n 2n
a a 4 2
a a 4 2
dUcen u n
2


2

.

eroberogeday lwm pln TMBr 214


sIVtncMnYnBit nig esrI

lMhat;TI59
eKman
6 2 5 2 11 , 56 2 45 2 1111 , 556 2 445 2 111111
5556 2 4445 2 11111111.
BITahrN_xagelIcUrrkrUbmnTUeTA nigRsaybBaakrUbmnenpg
dMeNaHRsay
rkrUbmnTUeTA nigRsaybBaak
eKman
62 52 11
562 452 1111
5562 4452 111111

5556 2 4445 2 11111111


tamlMnaMEdleK[eyIgGacsresrrUbmnTUeTAdUcxageRkam
444 111
2 2
555
....
556 .....445 .......
111
(n ) (n) ( 2n )

sRmayepgpatrUbmnen
tag T 555
....
n 556 444
2
.....445

2

(n) (n )

eroberogeday lwm pln TMBr 215


sIVtncMnYnBit nig esrI
555.....555 1 (444......444 1) 2
2

2 2
5 4
(10 n 1) 1 (10 n 1) 1
9 9
5 4 5 4
(10 n 1) 1 (10 n 1) 1 (10 n 1) 1 (10 n 1) 1
9 9 9 9
999
.........
999

10 1 10 1
10 1 (
n 2n
)
n ( 2n )

9 9 9
Tn 111
..........
111
Bit .
( 2n )

dUcen 555
.... 444
556
2
111
.....445 2
.......
111 .
(n ) (n ) ( 2n )

eroberogeday lwm pln TMBr 216


sIVtncMnYnBit nig esrI

lMhat;TI60
eK[ (a n ) CasIVtncMnYnBitEdl a1
1
2
2
a
nigcMeBaRKbcMnYnKtviCman n eyIgman a n 1 2 n
an an 1
cUrRsaybBaakfacMeBaRKbcMnYnKtviCman n eyIgman
a1 a 2 a 3 ......... an 1 .

dMeNaHRsay
RsaybBaakfa a 1 a 2 a 3 ......... a n 1
2
a
eyIgman a n 1 2 n
an an 1
tag bn
1
an
enaTMnakTMngEdl[kayeTACa
b n 1 b n b n 1
2

eKTaj b n 1 1 b n b n
2

b n 1 1 b n (b n 1)
1 1 1 1

b n 1 1 b n (b n 1) b n 1 b n
eKTaj an
1

1

1
b n b n 1 b n 1 1

eroberogeday lwm pln TMBr 217


sIVtncMnYnBit nig esrI
1 1 1
a 1 a 2 .... a n ......
b1 b 2 bn
1 1 1 1 1 1
( )( ) ... ( )
b1 1 b2 1 b2 1 b3 1 bn 1 bn1 1
1 1 1
1 1
b1 1 bn1 1 bn1 1

BIeRBa b1
1
a1
2 .
dUcen a 1 a 2 a 3 ......... a n 1 .

eroberogeday lwm pln TMBr 218


sIVtncMnYnBit nig esrI

lMhat;TI61
eK[sIVtncMnYnBit (U ) kMnteday U 2 . sin n4
n n
n

Edl n IN * .
k-cUrbgajfa 2 . cos (n 41) cos n4 sin n4
x-Taj[nfa U ( 2 ) cos n4 ( 2 ) cos (n 41)
n
n n 1

K-KNnaplbUk
S U U U ......... U CaGnuKmn_n n .
n 1 2 3 n

dMeNaHRsay
k-cUrbgHajfa 2 . cos (n 41) cos n4 sin n4
tamrUbmn cos(a b) cos a cos b sin a sin b
eyIgn
(n 1) n
2 cos 2 cos( )
4 4 4
n n
2 (cos cos i sin sin )
4 4 4 4
(n 1) 2 n 2 n n n
2 cos 2 ( cos sin ) cos sin
4 2 4 2 4 4 4
(n 1) n n
dUcen 2 . cos
4
cos
4
sin
4
.

eroberogeday lwm pln TMBr 219


sIVtncMnYnBit nig esrI

x-Taj[nfa U ( 2 ) cos n4 ( 2 )
n
n n 1
cos
(n 1)
4
eyIgman 2 . cos (n 41) cos n4 sin n4
naM[ sin n4 cos n4 2 cos (n 41)
KuNGg:TaMgBIrnwg ( 2 ) n

eKn ( 2 ) sin n4 ( 2 ) cos n4 ( 2 )


n n n 1
cos
(n 1)
4
n (n 1)
dUcen U (
n 2 ) n cos
4
( 2 ) n 1 cos
4
.
K-KNnaplbUk S n U 1 U 2 U 3 ......... U n
eyIgn
n
k (k 1)
S n ( 2 )k cos ( 2 )k 1 cos
k 1 4 4
(n 1)
2 cos ( 2 )n 1 cos
4 4
(n 1)
dUcen S n 1 ( 2 ) n 1 cos
4
.

eroberogeday lwm pln TMBr 220


sIVtncMnYnBit nig esrI

lMhat;TI62
eK[ (x ) nig (y ) CasIVtcMnYnBitkMntelI IN
n n

eday x 5 , y 1 nigTMnakTMngkMenIn
0 0

x x 3x y
n1 n
3
nig y 3x y y RKb
n n
2
n1 n
2
n n
3
n IN
cUrKNna x nig y CaGnuKmn_n n .
n n

dMeNaHRsay
KNna x nig y CaGnuKmn_n n
n n

eKman x x 3x y (1)
n1 n
3
n n
2

nig y 3x y y (2)
n1 n
2
n n
3

bUksmIkar (1) nig (2) eKn


xn 1 y n 1 ( xn y n )3
ln( xn 1 y n 1 ) 3. ln( xn y n )
TMnakTMngenbBaakfa { ln( x y ) } CasIVtFrNImaRtman n n

ersug q 3 nigtYdMbUg ln( x y ) ln 6 . 0 0

eKn ln( x y ) 3 ln 6
n n
n

eKTaj x y 6 (3)
n n
3n

dkksmIkar (1) nig (2) eKn

eroberogeday lwm pln TMBr 221


sIVtncMnYnBit nig esrI
xn 1 y n 1 ( xn y n )3
ln( xn 1 y n 1 ) 3. ln( xn y n )
TMnakTMngenbBaakfa { ln( x y ) } CasIVtFrNImaRtman
n n

ersug q 3 nigtYdMbUg ln( x y ) ln 4 . 0 0

eKn ln( x y ) 3 ln 4
n n
n

eKTaj x y 4 (4)
n n
3n

bUksmIkar (3) nig (4) eKn 2x 6 4 n


3n 3n

n n
63 43
eKTaj xn
2
.
dksmIkar ( 3) nig
( 4) eKn 2y n 6 3n
4 3n

n n
63 43
eKTaj yn
2
.
n n n n
63 43 63 43
dUcen xn
2
nig yn
2
.

eroberogeday lwm pln TMBr 222


sIVtncMnYnBit nig esrI

lMhat;TI63
3
KNnaplbUk S 1 3xx 3x Edl x 101i ; i 1 , 2 , 3 , ...
101
i
2 i
i0 i i

dMeNaHRsay
3
KNnaplbUk S 1 3xx 3x
101
i
2
i0 i i

eyIgman 1 3x 3x x (1 x) x (x 1)
2 3 3 3 3

3 3
tag f (x) 1 3xx 3x x (x1 x) 2 3 3

3
eK)an f (x ) x
i 3
xi
(1 x i ) 3
i
(1 x i ) 3
ehIy f (1 x ) (1 x )
i 3
xi
3
i
(1 x i ) 3
3
eK)an f (x ) f (1 x ) x (1 x ) (1 x ) x 1
i i 3
xi
3 3 3
i i i i

eKTaj f (x ) 1 f (1 x )
i i

eday x 101i enaH 1 x 1 101i 101101 i


i i

eK)an S f (x ) f 101i 1 f (101101 i )


101 101 101
i
i0 i0 i0
101 i
102 S eKTaj S 51 .
101 102
S 102 f
101
i0 2

eroberogeday lwm pln TMBr 223


sIVtncMnYnBit nig esrI

lMhat;TI64
cUrbgajfacMeBaHRKb;cMnYnKt;viCman n eKman
1 1 1 1 1 1
... ....
1.2 2.4 ( 2n 1)2n n 1 n 2 2n
dMeNaHRsay
bgajfa
1 1 1 1 1 1
... ....
1.2 2.4 ( 2n 1)2n n 1 n 2 2n
tag T
1

1.2 2.4
1
.....
1
( 2n 1)2n
1 1 1 1 1
(1 ) ( ) .... ( )
2 3 4 2n 1 2n
1 1 1 1 1 1
1 ..... 2( .... )
2 3 2n 2 4 2n
1 1 1 1 1 1
T 1 .... (1 .... )
2 3 2n 2 3 n
1 1 1
.....
n1 n 2 2n
dUcenH 1

1.2 2.4
1
...
1

1

( 2n 1)2n n 1 n 2
1
....
1
2n

eroberogeday lwm pln TMBr 224


sIVtncMnYnBit nig esrI

lMhat;TI65
eK[sIVtncMnYnBit
2 2
1 1
an 1 1 1 1 ; n 1
n n
cUrbgajfa 1 1 1
....
a1 a 2 a 3
1
a 20
CacMnYnKt; .
dMeNaHRsay
bgajfa a1 a1 a1 .... a1 CacMnYnKt;
1 2 3 20
2 2
1 1
eyIgman an 1 1 1 1 ; n 1
n n
eK)an 1
an
2
1
2
1 1
1 1 1 1
n n
n 1
2 2
1
1 1 1 1
4
n n


1
4
2n 2 2n 1 2n 2 2n 1

eroberogeday lwm pln TMBr 225


sIVtncMnYnBit nig esrI
20 1 1 20

2n 2 2n 1 2n 2 2n 1
n 1 a n 4 n 1


1
4

( 5 1) ( 13 5 ) .... ( 29 761)
1
( 1 29) 7
4
dUcenH 1 1 1
....
a1 a 2 a 3
1
a 20
7 CacMnYnKt; .

eroberogeday lwm pln TMBr 226


sIVtncMnYnBit nig esrI

lMhat;TI66
eK[sIVt a ; a 1 epgpat;lkxN
2 ; ....; an

a 0 ; | a || a 1 |; ... nig | a || a 1 | .
1 2 1 n n 1

bgajfa a a n ... a 12 .
1 2 n

dMeNaHRsay
Rsayfa a a n ... a 12
1 2 n

eyIgman | a || a 1 |
n n 1

naM[ a a 2a 1
n
2 2
n 1 n 1
n1 n 1
eK)an (a ) (a 2a 1) (a 2a
n
1)
2 2 2
k k 1 k 1 k k
k 1 k 1 k 1
n n n
an2 1 (a k ) ( a k ) 2 ( a k ) n
2 2

k 1 k 1 k 1

eKTaj a 2 n n2 an2 1
n
k
k 1

dUcenH a a n ... a 12 .
1 2 n

eroberogeday lwm pln TMBr 227


sIVtncMnYnBit nig esrI

lMhat;TI67
eK[sIVtncMnYnBit ( v ) kMnt;eday n nIN

v 5 nig TMnak;TMngkMenIn v 2 v 1 ; n 0
2
0 n 1 n

bgajfa v v 1 ( v v 1 )
n 1
2
n 1 n n
2 2

rYcKNna v CaGnuKmn_n n .
n

dMeNaHRsay
bgajfa v v 1 ( v v 1 )
n 1
2
n 1 n n
2 2

tag w v v 1 eday v 2v 1 ; n 0
n n 1
2
n 1 n 1 n
2

eK)an w 2v 1 (2v 1) 1
n n
2
n
2 2

2vn 1 4vn 4vn


2 4 2

v n 2 v n v n 1 ( v n 1)
2 2 2

( v n v n 1 )2
2

dUcenH v v 1 ( v v 1 )
n 1
2
n 1 n n
2 2

KNna v CaGnuKmn_n n
n

tag t ln( v v 1)
n n n
2

eK)an t ln( v v 1)
n 1 n 1
2
n 1

eday v v 1 ( v v 1 )
n 1
2
n 1 n n
2 2

eroberogeday lwm pln TMBr 228


sIVtncMnYnBit nig esrI

eKTaj t 2 ln( v v 1) 2t
n 1 n n
2
n

naM[ (t ) CasIVtFrNImaRtmanersug q 2
n

nigtY t ln( 5 2) .
0

tamrUbmn t t q 2 ln( 5 2) ln(


n 0
n n
5 2) 2n

eday t ln( v v 1)
n n n
2

eKTaj)an v v 1 ( 5 2) n (1)
n n
2 2

eday ( v v 1)( v v 1) 1
n n
2
n n
2

eKTaj v v 1
n n
1
2

vn vn 1
2

1 2n
vn vn 1
2
( 5 2 ) ( 2)
2n
( 5 2)
bUksmIkar (1) nig (2) eKTaj)an
n n
2 v n ( 5 2)2 ( 5 2) 2
n n
( 5 2)2 ( 5 2)2
dUcenH vn
2
.

eroberogeday lwm pln TMBr 229


sIVtncMnYnBit nig esrI

lMhat;TI68
eK[sIVtncMnYnBit (u ) kMnt;eday
k kIN

u 9 nig TMnak;TMngkMenIn u C
n
p p
0 k 1 n uk
p 1

Edl C p!(nn! p)! .


p
n

cUrKNna u CaGnuKmn_n k nig n


k

dMeNaHRsay
KNna u CaGnuKmn_n k nig n
k

eyIgman u C u
n
p p
k 1 n k
p 1

eday 1 C u 1 (1 u )
n n
p p
Cpn uk p
n k k
p

p 1 p0

eK)an u 1 (1 u )
k 1 k
p

eKTaj ln(1 u ) p ln(1 u )


k 1 k

TMnak;TMngenHbBaak;fa { ln(1 u ) } CasIVtFrNImaRtman k

pleFobrYm p nigtYdMbUg ln(1 u ) ln 10 o

eK)an ln(1 u ) p ln 10 naM[ u 10 1 .


k
k k
k
p

eroberogeday lwm pln TMBr 230


sIVtncMnYnBit nig esrI

lMhat;TI69
eK[sIVtncMnYnBit (u ) kMnt;eday
n nIN

u 1 nig TMnak;TMngkMenIn u 2u 4u 1
2
0 n 1 n n

cUrKNna u CaGnuKmn_n n
n

dMeNaHRsay
KNna u CaGnuKmn_n n
n

eKman u 2u 4u 1
n 1 n
2
n

KuNGgTaMgBIr nwg 2 eK)an


2un 1 4un 8un 2
2

EfmGgTaMgBIrnwg 2 eK)an
2(un 1 1) 4(un 1)2
tag v ln2(u 1)
n n

eK)an v ln 2(u
n 1 n 1 1)

v n 1 ln 4 ( u n 1 ) 2
v n 1 2 ln 2 ( u n 1 )
naM[ ( v ) CasIVtFrNImaRtmanpleFobrYm q 2
n

nigtY v ln [2(u 1)] ln(4)


0 0

eroberogeday lwm pln TMBr 231


sIVtncMnYnBit nig esrI
2n 1
tamrUbmn v v q 2 ln 4 ln 2
n 0
n n

eday v ln2(u 1)
n n

eKTaj 2(u 1) 2 n1
n
2

n 1
dUcenH u 2
n 1 .2 1

eroberogeday lwm pln TMBr 232


sIVtncMnYnBit nig esrI

lMhat;TI70
eK[ S k3 13 23 33 .... n3
n 2 2 2 2 2
n k 2 3 n
k 1

KNna S CaGnuKmn_n n rYcTajrk lim S


n
n
n

dMeNaHRsay
KNna S CaGnuKmn_n n
n
n k 2 12 22 32 n2
Sn k 2 3 .... n
k 1 3 3 3 3 3
k2
tag tk kcMeBaHRKb;
3
k 1

(k 1)2 k 2 2k 1
eK)an 3t k 1 t k
3 k
k
3 3k
2k 1
yk Tk 3t k 1 t k
3k
2k 3 2k 1 2
eK)an 3Tk 1 Tk
3k

3k
k
3
b 2
3( 3t k 2 t k 1 ) ( 3t k 1 t k ) k
3
b 9t k 2 6t k 1 t k k
3
2

edayeKman
9t k 2 6t k 1 t k 9(t k 2 t k 1 ) 3(t k 1 t k ) 4t k

eroberogeday lwm pln TMBr 233


sIVtncMnYnBit nig esrI

eKTaj t k
1 1 9
.
2 3k 4
( t k2 t k 1 )
3
4
(t k 1 t k )
1 n 1 9 n
eK)an S k
2 k 1 3 4 k 1
3 n
k t k 2 t k 1 t k 1 t k
4 k 1
1
1 n
1 3 9 (t 3
. n 2 t 2 ) (t n 1 t1 )
6 1 1 4 4
3
1 1 9 3 9 3
(1 n ) t n 2 t n 1 t 2 t1
4 3 4 4 4 4
2
eday tk k
k
3
(n 1)2 (n 2) 2
eK)an 1
3
4
t1 ; t 2 ; t n 1 n 1 ; t n 2
9 3 3n 2
1 1 9 (n 2)2 3 (n 1)2 9 4 3 1
Sn n
. n 2 . n 1 . .
4 4.3 4 3 4 3 4 9 4 3
3 1 n 3n 3
2
dUcenH Sn .
2 2 3n
n 2 3n 3
eday lim
n 3 n
0

dUcenH lim Sn
n
3
2
.

eroberogeday lwm pln TMBr 234


sIVtncMnYnBit nig esrI

lMhat;TI71
a 5
2
eK[ a0 2 3 6 nig a n 1 n
2(a n 2)
cMeBaRKb n0 .
2n 3
cUrRsayfa a n cot 2 cMeBaRKb n IN .
3
dMeNaHRsay
2n 3
RsaybBaakfa a n cot 2
3
cMeBa n 0 eKn a 0 cot 2
24

cos 2 cos 2 1 cos
24 24 12
cot
24
sin 2 sin cos sin
24 24 24 12
6 2
1 cos( ) 1
3 4 4 4 6 2

6 2 6 2
sin( )
3 4 4

eroberogeday lwm pln TMBr 235


sIVtncMnYnBit nig esrI
4( 6 2 ) ( 6 2 ) 2
cot
24 4
4( 6 2 ) 8 4 3

4
2 2 3 6
eKTaj a0 2 3 6
2n 3
ehtuen a n cot 2 BitcMeBa n0 .
3
2k 3
sntfavaBitdltYTI k KW a k cot 2 Bit
3
eyIgnwgRsayfavaBitdltYTI k 1 KW
2k 2
a k 1 cot 2 Bit .
3
ak 5
2
2k 3
eyIgman a k 1 eday a k cot 2
2(a k 2) 3
2
2
k3

cot 2 5
3
ena a k 1
2k 3
2 cot
3

eroberogeday lwm pln TMBr 236


sIVtncMnYnBit nig esrI
2k 3 2k 3
cot
2
4 cot 1
a k 1 3 3
2k 3
2 cot
3
2k 3
cot 2
1
3 2
k3
2
2 cot
2
cot 2 a 1
edayeRbIrUbmn cot 2a
2 cot a
2k 2
eKn a k 1 cot 2 Bit .
3
2n 3
dUcen a n cot 2 .
3

eroberogeday lwm pln TMBr 237


sIVtncMnYnBit nig esrI

lMhat;TI72
eK[ xn 22 1
n
cMeBaRKb n 1 , 2 , 3 , ... .
1 2 22 2n 1 1
cUrRsayfa ....
x1 x 2 x 3 xn 3
dMeNaHRsay
1 2 22 2n 1 1
Rsayfa ....
x1 x 2 x 3 xn 3
tag y 2 1 RKb n 1
n
2n

eKn y 2 1 (2 1)(2 1)
n 1
2 n 1 2n 2n

y y x .
n1 n n

1 2 1 2 x 2
eKman n

yn y n1 yn xn y n xn y n
eday x 2 2n
1 2 2 1 yn
2n 2n

1 2 1
eKTaj
yn y n 1 x n
2n 1 2n 2n 1
naM[
xn y n y n1
cMeBa n 1 , 2 , 3 , .... eKn

eroberogeday lwm pln TMBr 238


sIVtncMnYnBit nig esrI
1 2 22 2n 1 1 2n 1 1
....
x1 x 2 x 3 xn y 1 y n1 y 1
eday y 1 22 1 3
1 2 22 2n 1 1
dUcen .... .
x1 x 2 x 3 xn 3

eroberogeday lwm pln TMBr 239


sIVtncMnYnBit nig esrI

lMhat;TI73
eK[sIVt U n 2 2 2 ..... 2 3

(n)

cMeBaRKb n IN * .
k-cUrkMnt U CaGnuKmn_n n .
n

x-cUrbgHajfa U U U ...... U
3 .
1 2 3 n

2 sin
3 .2 n
K-eKBinitsIVt Vn 2 n 2 2 2 ..... 2 3

.
(n)

cUrKNna Vn nig lImIt lim Vn


n
.
dMeNaHRsay
k-kMnt U CaGnuKmn_n n
n

eyIgman U n 2 2
2 ..... 2 3

(n)

eyIgn U 1 3 2 cos
6

U 2 2 3 2 U 1 2 2 cos 2 cos
6 12

U 3 2 2 3 2 U 2 2 2 cos 2 cos
12 24

eroberogeday lwm pln TMBr 240


sIVtncMnYnBit nig esrI

---------------------------------------------------------
eyIgsntfavaBitdltYTI U 2 cos . k k
3.2
eyIgnwgRsaybBaakfavaBitdltYTI (k 1) KW

U 2 cos
k 1 Bit
k 1
3.2
eyIgman
U k 1 2 2 2 2 ... 2 3 2 U k

edaytamkarsnteKman U k 2 cos
3.2 k
eyIgn
U k 1 2 2 cos

4 cos 2
2 cos
Bit .
3.2 k 3.2 k 1 3.2 k 1

dUcen U 2 cos .
n n
3.2
x-bgHajfa U U U ...... U
3
1 2 3 n

2 sin
3. 2 n
2
sin
eyIgman 3.2 n
U n 2 cos n
3.2
sin n
3.2
( rUbmn sin 2a 2 sin a cos a )

eroberogeday lwm pln TMBr 241


sIVtncMnYnBit nig esrI

eyIgn

sin
3
U 1
sin
6

sin
U 2 6

sin
12


2
sin n
3 . 2
U n
sin
3 .2 n

sin
dUcen U 1 U 2 U 3 ...... U n 3 3 .

sin n 2 sin n
3 .2 3.2
K-KNna Vn nig lImIt lim Vn
n

eyIgman Vn 2 n 2 2 2 ..... 2 3

(n)

Vn 2 n 2 2 2 .... 2 3

( n 1 )

Vn 2 n 2 U n 1

eroberogeday lwm pln TMBr 242


sIVtncMnYnBit nig esrI

eday U 2 cos ena U n 1 2 cos
3.2 n 1
n n
3.2
eyIgn

Vn 2 n
2 2 cos n 1
3.2
n 1
2 n 4 sin 2 2 sin
3.2 n 3.2 n

dUcen Vn 2 sin n .
n 1

3.2
2
ehIy nlim

Vn lim 2 n 1 sin n
n
.
3.2 3

eroberogeday lwm pln TMBr 243


sIVtncMnYnBit nig esrI

lMhat;TI74
eK[GnuKmn_ f (x) x 2 Edl x IR
2

k-eKyk U1 f (x) nig U n1 f (U n ) cMeBaRKb n IN * .


cUrbgHajfa U n fn f .......f f (x)....... .
x-RsayfaebI x 2 eKn U n 2 RKb n IN * .
K-eKtag Vn U n U n 2 4 RKb n IN * nig x 2 .
cMeBaRKb n IN * cUrbgHajfa 2Vn1 Vn 2 .
X-sntfa Wn ln Vn ln 2 cMeBaRKb n IN * .
cUrrkRbePTnsIVt Wn .
g-eRbIlTplxagelIcUrTajrkGnuKmn_
Fn ( x) f n f ........f f ( x ) ......... .

dMeNaHRsay
k-bgHajfa U n fn f .......f f (x).......
eyIgman U1 f (x) Bit ( tamsmtikm )
U 2 f ( U 1 ) f f ( x) Bit ( eRBa U n 1 f ( U n ) )
U 3 f ( U 2 ) f f f ( x ) Bit
eyIgsntfavaBitdltYTI k KW

eroberogeday lwm pln TMBr 244


sIVtncMnYnBit nig esrI

U k f k f .......f f ( x)....... Bit


eyIgnwgRsayfavaBitdltYTI k 1 KW
U k 1 f k 1 f .......f f ( x )....... Bit
eyIgman
Uk 1 f U k f fk f ........f f ( x )......
fk 1 f .......f f x .......
dUcen U n f n f .......f f (x)....... .
x-RsayfaebI x 2 eKn U n 2 RKb n IN *
eyIgman U n1 f (U n ) U n 2 2
ebI x 2 ena U1 f (x) x 2 2 2 U1 2 Bit
eyIgsntfavaBitdltYTI k KW U k 2 Bit
eyIgnwgRsayfavaBitdltYTI k 1 KW U k 1 2 Bit
eyIgman U k 1 U k 2 2
eday U k 2 naM[ U k 2 4 U k 2 2 4 2 2
eKTaj U k 1 U k 2 2 2 Bit .
dUcen ebI x 2 eKn U n 2 RKb n IN * .
K-bgHajfa 2Vn1 Vn 2
eyIgman Vn U n U n 2 4
eroberogeday lwm pln TMBr 245
sIVtncMnYnBit nig esrI

eyIgn Vn 1 U n 1 U n 1 4
2
Et U n 1 U n 2
2

Vn 1 U n ( U n 2)2 4
2 2

Vn 1 U n U n 4U n
2 4 2

Vn 1 U n U n U n 4
2 2

2Vn 1 2U n 2U n U n 4
2 2

2Vn 1 Un 2Un Un 4 ( U n 4 )2
2 2 2


2Vn 1 U n Un 4
2
2
Vn
2

dUcen 2Vn1 Vn 2 .
X-rkRbePTnsIVt Wn .
eKman Wn ln Vn ln 2 cMeBaRKb n IN *
eKn Wn1 ln Vn1 ln 2 eday 2Vn1 Vn 2
2
V
Wn 1 ln n ln 2 2 ln Vn 2 ln 2 2Wn
2
dUcen ( Wn )CasIVtFrNImaRtmanersug q2 .
g-rkGnuKmn_ Fn (x) f n f ........f f (x) .........
eday U n f n f .......f f (x).......
eKTajn Fn (x) U n .
tamsRmayxagelIeyIgman ( Wn ) CasIVtFrNImaRtman

eroberogeday lwm pln TMBr 246


sIVtncMnYnBit nig esrI

ersug q 2 .
tamrUbmn Wn W1 q n1 2n1 . W1
eday W ln V ln 2 ln V2
1 1
1

Et V1 U1 U12 4 f (x) f 2 (x) 4
V1 x 2 2 ( x 2 2)2 4
V1 x 2 2 x 4 4x 2
1
x 2 2 x x 2 4 ( x x 2 4 )2
2
2
(x x 2 4 )2 x x2 4
eKn W1 ln ln

4 2
2 2n
x x 4
ehtuen Wn 2 n 1 . ln

2
ln x x 4

2


2 2
V
eday Wn ln Vn ln 2 ln n
2
2n
Vn x x2 4
eKTaj ln ln

2 2
2n
x x2 4
Vn 2
2



mageToteKman Vn U n U n 4
2

eroberogeday lwm pln TMBr 247


sIVtncMnYnBit nig esrI

U n Vn U n 4
2

U n 2U n Vn Vn U n 4
2 2 2

2U n Vn Vn 4
2

V 4 1
2
2
Un n Vn
2Vn 2 Vn
2n
x x 4
2
eday Vn 2
2



2n
eKn x x2 4
Un
1
2 2n
x x 4
2

2

2n
x x2 4
KuNnwgkenam
2


eKn

eroberogeday lwm pln TMBr 248


sIVtncMnYnBit nig esrI
2n
x x 4 2
n
x x2 4
2
2
Un
2 2n 2n
x x 42 x x 4 2

2 2

2n
x x2 4
2n

x x 42 2
Un
2 2n
x x 4
2 2


4
2n 2n
x x 42 x x 4 2
Un
2 2

2n 2n
x x2 4 x x2 4
dUcen Fn ( x )
2



2


.

eroberogeday lwm pln TMBr 249


sIVtncMnYnBit nig esrI

lMhat;TI75
sIVtncMnYnBit (an ) n1 kMNt;eday a1 1 , a2 3
nig an 2 (n 3)an1 (n 2)an , n IN
cUrkMNt;RKb;tm n edIm,I[ an Eckdac;nwg 11 .
dMeNaHRsay
kMNt;RKb;tm n edIm,I[ an Eckdac;nwg 11
eKman an 2 (n 3)an1 (n 2)an , n IN
b an 2 an1 (n 2)(an1 an )
b aan 2 aan1 n 2
n 1 n
(n 2) a ak 1 ( n 2 )
eK)an k2 (k 2)
k 1 ak 1 ak k 1
an an 1
a 2 a1
3.4.5......n eday a2 a1 2
eKTaj)an an an1 n!
n n
ehIy ak ak 1 k!
k 2 k2

an a1 2!3!.... n! eday a1 1 1!
eK)an an 1!2!3!..... n! .
eroberogeday lwm pln TMBr 250
sIVtncMnYnBit nig esrI

-krNITI1 cMeBaH n 11 eKman


a1 1
a2 3
a 3 1!2!3! 9
a4 1!2!3!4! 33 3 11
a5 a4 5! 153
a6 a5 6! 873
a7 a6 7! 5913
a8 a7 8! 46233 4203 11
a9 a8 9! 409113
a10 a9 10! 4037913
eK)an n 4 , n 8 .
-krNITI2 cMeBaH n 11
n
eK)an an a10 (k! )
k 11
n
eday (k! ) Eckdac;nwg 11 ehIy a10 Eckmindac;nwg 11
k 11

enaHcMeBaH n 11 eK)an an Eckmindac;nwg 11 .


dUcenHtm n EdleFIV[ an Eckdac;nwg 11 manEtBIrKt;KW
n4 b n8 .

eroberogeday lwm pln TMBr 251


sIVtncMnYnBit nig esrI

lMhat;TI76
eK[sIVt (un ) kMNt;eday
5
u0 2 , u1 , un 1 un (un 12 2) u1
2
cMeBaH n 1 , 2 ,...
2n ( 1)n
cUrbgajfacMeBaHRKb;cMnYnnKt;viCman n eKman un 2 3

Edl x tagCacMnYnKt;viCmantUcCag x

dMeNaHRsay
2n ( 1)n
bgajfa un 2 3

CadMbUgeyIgKNnatY u2 , u3 , u4 , u5 eK)anlMnaMdUcxageRkam
1 5 1
u 0 2 20 , u1 21 1
20 2 2
5 1 1
u2 21 1 , u 3 23 3
2 2 2
1 1
u 4 25 5 , u5 211 11
2 2
eyIgsegteXIjfakenSam un manragTUeTA un 2Vn V1n
2
Edl (Vn ) CasIVtkMNt;eday (Vn ) : 0 , 1 , 1 , 3 , 5 , 11 , .....
eroberogeday lwm pln TMBr 252
sIVtncMnYnBit nig esrI

tagsIVt (wn ) Edl wn Vn Vn1 RKb; n 0


eK)an ( Wn ) : 1 , 2 , 4 , 8 , 16 , ....naM[ ( Wn ) CasIVtFrNImaRt
mantY W0 1 nig ersug q 2 .
eK)an Wn 2n enaH Vn1 Vn 2n
KuNGgTaMgBIrnwg (1)n1 eK)an
( 1)n 1 Vn 1 ( 1)n Vn ( 1)n 1 2n

eK)an
n 1
( 1) k 1
n 1
Vk 1 ( 1) Vk ( 1)k 1 2k
k

k 0 k 0
( 1)n 2n 1
( 1) Vn V0
n
3
( 1)n 2n 1 2n ( 1)n
eday V0 0 enaH Vn
( 1) .3
n 3
2n ( 1)n 2n ( 1)n

dUcenH un 2 3 2 3

eday 2n (1)n (mod 3 ) enaH 3 | 2n ( 1)n


2n ( 1)n 2n ( 1)n

ehIy 2 3 1 dUcenH un 2 3 .

eroberogeday lwm pln TMBr 253

Potrebbero piacerti anche